Pediatric Clerkship UMed

Ace your homework & exams now with Quizwiz!

Steven, a 5-year-old boy with no significant past medical history, was in his usual state of health until last night when he developed abdominal pain. This morning his mother noticed a red and blotchy rash on his buttocks and lower extremities and his abdominal pain has worsened. Otherwise, he has no other symptoms and except for an upper respiratory tract infection last week, he has been in good health recently. On exam, the presence of palpable purpura and petechiae over the buttocks is confirmed. Laboratory studies are normal and, after a clinical diagnosis is made, he is discharged home the same day and given instructions to return for follow-up. Which of the following is important to measure at the first follow-up visit? A. BP and urinalysis B. Platelet count C. PT/PTT D. White blood cells and hemoglobin E. Signs of intracranial hemorrhage

> A has been selected by the expert. A. BP and urinalysis is the correct answer. This patient likely has Henoch-Schonlein Purpura (HSP). The exact etiology of HSP is unknown, but it is believed to involve an IgA-mediated immune response to infection or other triggers. The incidence is 10 cases per 100,000 children with a peak at ages 4 to 6 years (range of 2 to 17 years). HSP is characterized by a rash consisting of petechiae and palpable purpura. Other findings include a colicky diffuse or periumbilical abdominal pain, arthritis or arthralgia, and renal disease. Given the incidence of renal disease, it is important to check the urine for signs of hematuria or proteinuria; sudden changes in blood pressure can potentially suggest a change in renal function. With abnormal findings, serum BUN and creatinine must be checked. B. Platelet count is normal in HSP since the hallmark rash is a non-thrombocytopenic purpura. Checking this value at follow-up when the initial value is normal is not indicated in a patient with HSP. Idiopathic thrombocytopenic purpura (ITP) is part of the differential for palpable purpura and petechiae. ITP is characterized by low platelet counts (usually < 20,000) with normal WBCs and hemoglobin. Should you find low platelets, you may want to reconsider the differential and think about ITP or possibly leukemia. C. A coagulation disorder is part of the initial differential for a patient presenting with palpable purpura and petechiae. In this case, our patient with HSP has no abnormalities in any coagulation pathway. Instead, the mechanism of petechiae and purpura is secondary to an immune-mediated vasculitis. Therefore, checking the PT/PTT at the first follow-up visit is not indicated in this case. D. Leukemia was in the initial differential for Steven's petechiae and purpura rash. Leukemia is characterized by abnormalities in the different cell lines (WBC, RBCs and platelets) resulting in cytopenias. The clinical presentation of leukemia is associated with bone marrow infiltration and cytopenia. Signs and symptoms include petechia and purpura, bone pain, fever, fatigue, malaise, hepatosplenomegaly, and lymphadenopathy. When leukemia is suspected, it is important to measure WBCs, Hgb and platelets. In HSP, there should not be any abnormalities in any of these cell lines and, therefore, measuring these values at follow-up does not contribute any significant information. E. ICH is a potential but rare complication of idiopathic thrombocytopenic purpura (ITP). The incidence of ICH in patients with ITP is 0.1 to 0.5% of cases. In this case of HSP, ICH is not a potential complication.

A 2-year-old male presents to the ED with a 5-hour history of hyperactivity, fever, and sweating. His BP is 160/90 mmHg, HR 130 bpm, RR 30 bpm. On exam, he has dilated pupils, cool skin, and hyperreflexia. What is his most likely accidental medication ingestion? A. Pseudoephedrine B. Codeine C. Iron pill D. Acetaminophen E. Propranolol

> A has been selected by the expert. A. Choice A is correct. Ingestion of a sympathomimetic like pseudoephedrine stimulates the beta and alpha adrenergic receptors, causing elevated HR, RR, BP and hypothermia along with diaphoresis, dilated pupils, hyperreflexia, and hyperactivity. B. Choice B is incorrect because constricted pupils are seen in ingestions of opioids. Slowed breathing, rather than tachypnea, may also be seen. C. Choice C is incorrect. Iron toxicity would present with severe abdominal symptoms followed by signs of shock. D. Choice D is incorrect. Acetaminophen toxicity initially presents with minimal symptoms, followed by symptoms of liver toxicity such as jaundice and RUQ pain. E. Choice E is incorrect. Beta-blocker toxicity would cause bradycardia, not tachycardia and hypertension.

An 11-year-old male comes to the clinic with a chief complaint of abdominal pain for three months. The pain is not associated with eating. Sometimes he feels full and nauseated, along with the pain, but then it resolves on its own. He denies diarrhea, vomiting, and bloody stools. His mother is primarily concerned because his abdominal pains cause him to miss school quite often now. ROS is otherwise negative and the only pertinent issue is his pain. When you evaluate his growth curves, he is progressing at the 60th percentile for height and weight and you do not notice a change since birth. Through a social history you ascertain that he is quite intelligent and has recently been advanced to 7th grade from 5th grade. Vital signs are within normal limits for his age and physical exam (including rectal and genital) are unremarkable. Stool sample was sent in anticipation of today's visit and was negative for occult blood. What is the most likely cause for his abdominal pain? A. Functional abdominal pain B. Inflammatory bowel disease C. Bacterial gastroenteritis D. Peptic ulcer disease E. Meckel's diverticulum

> A has been selected by the expert. A. Functional abdominal pain would be the most likely diagnosis in this setting at this time. History in this setting is not suggestive of any other diagnosis directly causing his abdominal pain, except a change in his social setting. For better understanding of the nature of this child's pain, it would be best to talk to him alone, without his mother present, to determine if he is having trouble adjusting to school and to assess whether he has a stable home environment. His pain is chronic, with no other symptoms (diarrhea, bloody stools, growth failure), making a functional issue most likely. B. Inflammatory bowel disease (Crohn's disease or UC) could be the cause of this child's pain, but more frequently IBD presents with growth failure, diarrhea, bloody stools, and sometimes fever. Although this could be an early presentation, the absence of arthritis, uveitis, and other constitutional symptoms, make IBD less likely. C. Choice C is incorrect because bacterial gastroenteritis is more of an acute issue and would be unlikely to persist for months. The absence of diarrhea also suggests bacterial gastroenteritis is not the best answer option. D. Choice D is incorrect because peptic ulcer disease (PUD) is relatively uncommon in children and the stool sample was negative for occult blood. PUD would be chronic and recurrent, but should also be associated with eating and would not spontaneously remit every couple of days. E. Choice E is incorrect because Meckel's diverticulum usually presents as painless blood in the stool.

A 2-year-old male is brought into the ED by his mother because of vomiting and altered mental status. He has pinpoint pupils and seems to be drooling and sweating uncontrollably. His heart rate is 60 bpm, his respiratory rate is 45 bpm, and he seems to have difficulty breathing. Which ingestion is the most likely cause of his symptoms? A. Organophosphates B. Tricyclic antidepressant C. Barbiturates D. Codeine E. Pseudoephedrine

> A has been selected by the expert. A. Organophosphates cause cholinergic effects, such as miosis, sweating, lacrimation, salivation, urination, increased gastric mobility (vomiting, diarrhea), muscle twitching, bronchospasm, bradycardia, and seizures. A good mnemonic is SLUDGE (salivation, lacrimation, urination, defecation, GI mobility, emesis). B. Tricyclic antidepressants are part of the group of anticholinergics. Overdose can cause mydriasis, dry and red skin, fever, delirium, seizures, tachycardia, urinary retention, and ileus. C. Barbiturates are part of the group of sedative-hypnotics. Excessive ingestion presents as blurred vision, hypotension, apnea, bradycardia, hypothermia, sedation, delirium, and coma. D. Codeine and other opioids can cause miosis, respiratory depression, bradycardia, hypotension, hypothermia, and depressed mental status. E. Pseudoephedrine is a sympathomimetic, and overdose results in mydriasis, fever, diaphoresis, tachycardia, agitation, and seizures.

A mother brings her 8-year-old son to his primary care physician for pain in his knees and ankles that have been present for the past three days. She also notes that he has had a rash since yesterday, but otherwise feels well. The patient has no chronic illnesses, but he was brought in three weeks ago for an upper respiratory infection. Exam is significant for pain elicited on passive movement of the ankles and knees. Additionally, the patient is found to have an erythematous, slightly raised, non-blanching, maculopapular rash over the legs, buttocks, and posterior portion of the elbows. CBC shows WBC 8.9, Hgb 12.5, Hct 36.1, and Plt 327. Urinalysis is unremarkable. Skin biopsy shows leukocytoclastic vasculitis with IgA deposition. Which of the following is the best next step in management? A. Observation B. Corticosteroids C. Intravenous immunoglobulin (IVIG) D. Intravenous hydration E. Platelet transfusion

> A has been selected by the expert. A. The joint pain, purpuric rash, and IgA deposition on skin biopsy support the diagnosis of HSP. Most cases of HSP resolve within approximately one month and do not require treatment. However, symptomatic treatment for joint pain, initially with NSAIDs, may be indicated. B. Corticosteroids can be useful as a second line agent for treating joint pain. Additionally, corticosteroids can be used as part of an immunosuppressive regimen in HSP patients who have acute kidney failure. There is no suggestion of kidney involvement in this case, and urinalysis was found to be unremarkable. Note that steroids may also be used in HSP patients with severe abdominal pain, but this patient did not present this way. C. IVIG is not used as a mainstay of treatment for HSP. D. There is no evidence of significant dehydration or inability to tolerate enteral nutrition in this patient. Thus, intravenous hydration is not warranted. E. Although patients with HSP have a petechial/purpuric rash, their platelet counts are within the normal range. Therefore, a platelet transfusion would be unlikely to be beneficial.

An 8-year-old healthy obese African American male with no past medical history is found to have a blood pressure of 125/90 mmHg on all four extremities on routine evaluation during an office visit for well-child care. Review of symptoms is negative. A physical exam and screening bloodwork are performed. Both are normal, with the exception of his blood pressure and obesity. What is the most likely diagnosis? A. Primary hypertension B. Renal artery stenosis C. Coarctation of the aorta D. Pheochromocytoma E. Hyperthyroidism

> A has been selected by the expert. A. The sole physical finding is hypertension. Given the mild hypertension and the patient's age, symptoms are unlikely to be present. Other etiologies should be ruled out, but review of symptoms, physical examination, and laboratory studies do not suggest other etiologies. B. Patients with renal artery stenosis are largely asymptomatic, but as the kidney function deteriorates, they may experience edema and dyspnea. On physical examination, patients will have hypertension (as a consequence of elevated angiotensin II and aldosterone) and possibly abdominal bruits. For laboratory studies, the patient will have elevated serum angiotensin II and serum aldosterone, as the kidneys attempt to compensate for a perceived decrease in glomerular filtration rate. C. Assuming sufficient severity of the coarctation, symptoms include chest pain, cold lower extremities, dizziness, syncope, exercise intolerance, failure to thrive, poor growth, headache, and dyspnea, among others. Distal to the coarctation, pulses will be diminished. Blood pressure will be lower in the lower extremities compared to the upper extremities. Murmurs may also be heart on auscultation. Coarctation of the aorta may also be associated with other congenital heart defects (bicuspid aortic valve in 50% of patients) and chromosomal abnormalities (Turner syndrome). D. Symptoms include headache, diaphoresis, palpitations, tremor, nausea, weakness, anxiety, nervousness, irritability, and weight loss, among other symptoms. On physical examination, patients typically present with tachycardia and severely elevated blood pressure. For laboratory studies, the patient will have elevated urinary VMA, urinary HVA, urinary metanephrines, and serum glucose. Pheochromocytoma may be associated with various syndromes, such as multiple endocrine neoplasia and Von Hippel-Lindau disease. E. Symptoms include heat intolerance, frequent bowel movements, increased appetite, diaphoresis, nervousness, restlessness, weight loss, tremor, hair loss, and palpitations, among others. On examination, patients typically present with tachycardia, hypertension, hyperreflexia, and goiter. The patient will have elevated T3, T4, and serum glucose, and TSH may be depressed or elevated depending on the etiology.

When you are at your preceptor's office, you are told to see Amy, a 20-month-old female diagnosed with cerebral palsy (CP) at the last visit. Your preceptor knows that you just had your lecture on CP and tries to have you figure out which type of CP Amy has. She tells you that Amy was born at full term but had severe jaundice and required extensive treatment for hyperbilirubinemia, including an exchange transfusion. She now has slow and uncontrolled movements throughout her body. Her brain MRI shows some atrophy of the basal ganglia. What type of CP does Amy have? A. Dyskinetic cerebral palsy B. Spastic diplegia C. Spastic quadriplegia D. Spastic hemiplegia E. Ataxic cerebral palsy

> A has been selected by the expert. A. This choice is correct because dyskinetic CP is associated with kernicterus, due to hyperbilirubinemia, as well as findings of basal ganglia pathology on imaging. Patients typically have motor abnormalities throughout the body. Dyskinetic CP is also associated with perinatal asphyxia and can involve the thalamus and cerebellum on imaging. B. This choice is incorrect because spastic diplegia is classically associated with premature birth and specific MRI findings of periventricular white matter abnormalities. Patients present with motor involvement that is more prominent in the legs than the arms. C. This choice is incorrect because patients with spastic quadiplegia have spasticity, clonus, and exaggerated tendon jerks throughout their bodies. Imaging would show global brain abnormalities. D. This choice is incorrect because spastic hemiplegia-associated with a stroke damaging a unilateral upper motor neuron tract-should present with spasticity of the contralateral arm and leg, not the entire body. E. This choice is incorrect because ataxic cerebral palsy should show cerebellar abnormalities on imaging, while Amy's MRI shows only basal ganglia involvement.

A 6-year-old male comes to the clinic with a chief complaint of scrotal swelling, recent weight gain, and decreased appetite. Vital signs are stable and there is no evidence of cardiac disease or jaundice. Further workup reveals proteinuria, hypoalbuminemia, and hyperlipidemia, consistent with nephrotic syndrome. Which of the following histological patterns is most likely to be seen on light microscopy? A. Normal glomeruli with minimal increase in mesangial cells and matrix B. Mostly normal glomeruli and mesangial proliferation but with areas of juxtamedullar glomeruli showing segmental scarring in one or more lobules C. Tram-track appearance of the glomerular basement membrane and subendothelial immune complex deposition D. Enlarged, hypercellular glomeruli with neutrophil invasion E. Glomeruli showing diffuse capillary and glomerular basement thickening

> A has been selected by the expert. A. This choice is correct because it describes the histology seen in minimal change disease. Histologically, minimal change disease is characterized by normal glomeruli on light microscopy. Minimal change disease is the most likely diagnosis of nephrotic syndrome between the ages of 1 and 10 years old, accounting for up to 85% of cases. B. This choice is incorrect because it describes the histology of focal segmental glomerulonephritis (FSGS). Segmental sclerosis and scarring as well as hyalinosis can be seen on light microscopy in FSGS. FSGS accounts for roughly 10% of cases of nephrotic syndrome in children. Over time, FSGS can often lead to involvement of all the glomeruli and can lead to end stage renal disease. C. This choice is incorrect because it describes the histology seen in membranoproliferative glomerulonephritis (MPGN). MPGN accounts for less than 5% of cases of nephrotic syndrome in children. D. This choice is incorrect because it describes the histology seen in acute post-streptococcal glomerulonephritis, which is a cause of nephritic syndrome, not nephrotic syndrome. Nephritic syndrome is an inflammatory process leading to hematuria and RBCs in the urine. Nephritic syndrome is associated with azotemia, oliguria, hypertension, and less severe proteinuria (< 3.5 g/day). E. This choice is incorrect because it describes the histology seen in diffuse membranous glomerulonephritis. Diffuse membranous glomerulonephritis is rarely seen in children.

An 18-month-old male comes to the clinic for a well-child check. His mother says he is a happy boy and endorses no complaints. She notes that he was born a few weeks early via emergency C-section and spent two months in the NICU. She says he is feeding well and gaining weight. When you ask about developmental milestones, you find out that that he pulls himself up to stand, but is not walking yet. He has several words and is interactive. There is no family history of disease during infancy. He is up to date on his immunizations. He is one of five children and his parents are currently experiencing difficult financial times. Exam reveals increased tone and hyperreflexia in his lower extremities. What is the most likely diagnosis of his developmental delay (if any)? A. Cerebral palsy B. Normal variant C. Metabolic disorder D. Genetic abnormality E. Psychosocial stress reaction

> A has been selected by the expert. A. This choice is correct. Children with cerebral palsy often present with defects in motor development and are often found to have abnormal neuromuscular exams, including increased tone and reflexes. Other areas of development are less consistently impacted. The cerebral palsy most likely was the result of an hypoxic injury during the perinatal period. B. This choice is incorrect because this child has profound motor delay and abnormal neurologic findings (hypertonia). C. This choice is incorrect because this child's newborn screening exam was normal, he has no family history of metabolic disease, and has not had any indication of any metabolic abnormalities. D. This choice is incorrect because of a lack any characteristic syndromic phenotype and negative family history, and the delay appears to be limited to his gross motor exam. E. This choice is incorrect because of his persistence of developmental delay and lack of behavioral changes (temper tantrum, sleep disturbance, etc.). Temporary loss of developmental achievements can be seen in times of stress.

A 5-year old previously healthy boy is brought to his pediatrician with complaints of intermittent abdominal pain, right ankle pain, and a purpuric rash over his buttocks and lower extremities. His mom says she thinks he may recently have recovered from an upper respiratory infection. Which of the following statements is true? A. CBC would likely reveal thrombocytopenia B. This disease is classified as a small vessel vasculitis C. Urinalysis is not warranted for this patient's work-up D. Treatment options include IVIG E. This disease is equally common in girls and boys

> B has been selected by the expert. A. Choice A is incorrect. Despite the petechiae and purpura seen in children with HSP, the platelet count is normal. This is a key difference between idiopathic thrombocytopenic purpura (ITP) and leukemia (which both commonly present with thrombocytopenia) and HSP, which has a normal platelet count. B. Choice B is correct because HSP is classified as a small vessel vasculitis. The exact mechanism of HSP is unknown; however, it is thought to be an IgA-mediated immune response affecting small vessels (skin, GI tract, joints, kidneys). Approximately 50% of cases follow viral or bacterial URIs. Biopsy of affected organs shows leukocytoclastic vasculitis with IgA deposition. C. Choice C is incorrect because 20 to 50% of individuals with HSP have renal involvement, so UA would be an appropriate study to order in a suspected HSP case. UA will often reveal mild hematuria and/or proteinuria. If either are present, BUN and Cr should be ordered to further evaluate the extent of renal involvement. D. Choice D is incorrect because treatment of HSP is symptomatic and supportive (e.g., NSAIDs for joint pain). HSP usually resolves within four to six weeks and has a 30% recurrence rate. IVIG is not indicated in patients with HSP; it is, however, a treatment option for ITP. E. Choice E is incorrect because HSP occurs in children ages 2 to 17 years, with the majority occurring between ages 4 and 6. Boys are affected twice as often as girls.

You are working overnight call in the ED when Charlie, a 3-year-old male infant, arrives after his parents witnessed an episode of convulsions at home. His parents report that Charlie was in his usual state of good health until three days ago when he developed fever, cough, and rhinorrhea. This evening they found him in bed with his eyes rolled upward, jerking all four of his extremities uncontrollably. He was unarousable from this state, which self-resolved after about two minutes. This has never happened before. Currently, Charlie is sleepy but arousable and complains of nausea. His vitals include T 103.2 F, P 112 bpm, BP 100/60 mmHg, RR 22 bpm, O2 sat 99% on room air. Aside from rhinorrhea and erythematous mucous membranes, the remainder of his physical exam is unremarkable. What is the next best step in management? A. Workup for source of fever B. EEG C. MRI brain D. Abdominal ultrasound E. Administration of valproic acid

> A has been selected by the expert. A. This is the correct answer because Charlie has likely experienced a febrile seizure in the setting of an infection. These are relatively common occurrences in Charlie's age range (6 months to 5 years), and the first priority would be to identify the source of fever and treat it. B. This answer is incorrect because EEGs are indicated in recurrent, focal, or complex seizures. An EEG may eventually be useful to evaluate Charlie's situation, especially if he has another seizure in the ED or if his febrile seizures recur in the future; however, identification of the source of the fever should be the first priority. In addition, interictal EEGs are only positive in 60% of children with epilepsy. C. This answer is incorrect. In the differential diagnosis of a self-limited generalized seizure in the setting of fever in a child of this age group, a febrile seizure is higher on the differential than a brain mass. D. This answer is incorrect. Acute abdomen is low in the differential in the setting of the presenting symptoms of cough, rhinorrhea, and fever. E. This answer is incorrect, because the majority of patients with febrile seizures do not require treatment with anticonvulsants. This is especially true if this is Charlie's first febrile seizure. Anticonvulsants such as phenobarbital or valproic acid may rarely be indicated, but generally these drugs are not recommended in the setting of febrile seizures because they are associated with serious side effects.

A previously healthy 14-year-old female presents to the ED with a one-day history of fever and altered mental status. Vital signs on presentation include: BP 115/70 mmHg, HR 145 bpm, RR 42 bpm, temp 39.7 C, oxygen sat 93%. Physical exam reveals nuchal rigidity, cool extremities, 1+ distal pulses, diffuse petechial rash, and capillary refill > 2 seconds. What is the important first step in management? A. Place IV and start NS bolus B. Order CBC, CMP, PT, and INR C. Start empiric antibiotic therapy with IV ceftriaxone D. Obtain a head CT E. Order blood cultures

> A has been selected by the expert. A. This patient is in septic shock due to meningococcal infection and should immediately be started on IV fluids in order to maintain perfusion to vital organ systems. Although this patient has a normal blood pressure, other vital signs and physical examination point to shock (HR and RR are both significantly elevated), which first and foremost requires fluid resuscitation. B. Although these labs will need to be obtained, the patient must first be managed in order to ensure hemodynamic stability. Drawing blood for lab tests is not the first priority. C. Starting antibiotic therapy is the second step in management after hemodynamic stabilization. While antibiotics should be started as soon as possible, fluid resuscitation is still the classic "next best step" choice when presented with a patient in shock. D. A head CT is not indicated in the initial evaluation of a patient presenting in septic shock, regardless of the source of infection. If the patient were stable, then a head CT may be required to rule out any masses prior to LP. E. Blood cultures would be the step just prior to antibiotics, so as not to contaminate the results. However, cultures still come after fluid resuscitation when a patient is in shock.

A 3-year-old girl comes to the clinic with a limp and a slightly externally rotated right hip. Which of the following signs/symptoms would you expect in the history or exam if a diagnosis of transient synovitis were made? A. History of a recent upper respiratory tract infection B. High-grade fever C. Iridocyclitis D. Knee pain E. ESR of 110 mm/hr

> A has been selected by the expert. A. Transient synovitis of the hip is associated with a low-grade fever and frequently occurs during or after a URI. Between 32% and 50% of children who present with transient synovitis had a recent upper respiratory tract infection. It is also important to remember that transient synovitis is a diagnosis of exclusion, and it is important to rule out other causes of hip pain that may require urgent intervention, such as septic arthritis. B. High-grade fever would be more concerning for osteomyelitis or septic arthritis, and would not be expected in transient synovitis (although low-grade fever is possible). However, fever occurs in only half of cases of osteomyelitis. Other symptoms associated with osteomyelitis include localized bony tenderness, indolent presentation, and refusal to bear weight. Osteomyelitis cannot be ruled out due to absence of fever. C. Iridocyclitis is associated with juvenile idiopathic arthritis (JIA). Other findings associated with JIA include fever and rash. Diagnostic criteria for JIA include children less than 16 years old and at least 6 weeks of arthritis in at least one joint. D. Hip pathology can present as knee pain. However, it is not something that you would use to rule in or rule out transient synovitis (it is not specific for transient synovitis). E. A significantly elevated ESR would be concerning for septic arthritis. In fact, in one study, an ESR > 40 mm/hr was one of the criteria found to be more associated with septic arthritis than with transient synovitis. Other criteria associated more with septic arthritis than with transient synovitis include: refusal to bear weight, erythema, warmth, swelling, elevated CRP, and an elevated WBC count.

You are working in the pediatric ED when a 3-year-old girl, Jenny, presents with altered mental status for the past six hours. Her mother reports that the babysitter called her at work today after Jenny started acting agitated and "looking very sick." The mother reports "she feels so warm, I think she has a fever and has become dehydrated." On exam, the patient is agitated and anxious with dilated pupils. Her skin is warm and dry. Vitals reveal tachycardia and hypotension. You suspect the child may have accidentally ingested one of her mother's medications. An overdose of which of the following medications could cause Jenny's symptoms? A. Tricyclic antidepressant B. SSRI C. Decongestant D. Acetaminophen E. ACE inhibitor

> A has been selected by the expert. A. Tricyclic antidepressant (TCA) is correct. TCA toxicity presents with agitation, tachycardia, hypotension, dilated pupils, and hot, dry skin from the anticholinergic effects of TCAs. B. SSRI is incorrect, because serotonin syndrome will usually present with a fever and sweaty, not dry, skin. Also, serotonin syndrome usually does not present with hypotension. C. Decongestant is not the correct answer. Decongestant overdose will elicit a sympathomimetic response, including agitation and tachycardia. However, unlike TCA overdose, decongestant overdose will cause hypertension, constricted pupils, fever/sweating. Additionally, severe overdose of decongestant may cause seizures. D. Acetaminophen is not the correct answer. Acetominophen overdose will produce mild and nonspecific symptoms and include right upper quadrant pain, with elevation in liver enzymes, resulting in liver failure at toxic levels. E. ACE inhibitor is not the correct answer. The typical adverse event for ACE inhibitors is cough.

A 2-year-old female is brought to the ED by her mother because of increasingly frequent abdominal pain. She has been experiencing this pain on and off for the past year, along with increasing abdominal distention, vomiting, and diarrhea. You chart her height and weight, and find that she is below the 5th percentile for both. IgA tissue transglutaminase (TTG) antibody returns positive. What is the best treatment for this patient? A. Antibiotic treatment B. Gluten-free diet C. Corticosteroids D. Pain management E. Metronidazole

> B has been selected by the expert. A. Antibiotics would be the treatment for some types of bacterial gastroenteritis. The chronic nature of this patient's abdominal pain strongly points away from a diagnosis of bacterial gastroenteritis, which would more likely be an acute disease. Falling off the growth curve also suggests a process that is more chronic than you would expect from bacterial gastroenteritis. Bacterial gastroenteritis typically presents with bloody diarrhea, with or without abdominal pain, and is associated with fecal leukocytes on Wright stain. B. A gluten-free diet is the best way to manage celiac disease. Celiac disease can present with chronic abdominal pain, vomiting, abdominal distention, and diarrhea. Growth failure can result from malabsorption or anorexia. Anemia may also result from occult GI bleeding, although frank blood in the stool is rare. The IgA tissue transglutaminase antibody titer is a very sensitive and specific test for this disease. C. Corticosteroids may be used to treat a flare of inflammatory bowel disease (IBD). Inflammatory bowel disease can present with mild/subacute or severe/acute abdominal pain, along with diarrhea (often bloody). Falling off the growth curve is also common in patients with IBD. The positive IgA tissue transglutaminase antibody, however, suggests that a diagnosis of celiac is more likely in this patient. D. Pain management alone is not sufficient for a patient with abdominal pain if there is no diagnosis. This may be appropriate in patients with Henoch-Schonlein purpura (HSP), if they have joint and abdominal pains. The abdominal pain experienced by patients with HSP is usually acute in onset and preceded by the characteristic palpable purpuric rash. Because of the acute onset of HSP, growth failure is not associated with this disease. In this patient with chronic abdominal pain, no rash, and growth failure, HSP is unlikely. E. Metronidazole is the treatment of choice for giardiasis. Infections with giardia may be acute or chronic, but do not usually lead to growth failure. This patient's clinical picture may fit with a giardia infection, except that the positive IgA TTG antibody titer suggests a diagnosis of celiac disease.

A 6-month-old baby boy is referred to your clinic because he has not been gaining weight appropriately. His mother denies any difficulty with feeding or reduced appetite, yet his weight has still dropped from the 30th to the 3rd percentile. Mother also complains that he has loose, malodorous stools. After a thorough workup, a diagnosis of cystic fibrosis (CF) is made. Which of the following is a TRUE statement regarding CF? A. CF is an autosomal dominant disorder B. CF is caused by a mutation in CFTR, resulting in defective salt balance C. CF is a disease that exclusively involves the respiratory system D. Gene therapy is now the primary source of CF therapy E. It is important to provide calories at a lower level than recommended dietary allowance for a given age in order to prevent GI upset

> B has been selected by the expert. A. CF is autosomal recessive, NOT dominant. Most people with CF do not have a positive family history. B. Correct. The mutation in CFTR gene results in defective salt balance. CF is caused by mutations in a single large gene on chromosome 7 that encodes the cystic fibrosis transmembrane conductance regulator (CFTR) protein. Clinical disease requires disease-causing mutations in both copies of the CFTR gene. C. CF does involve the respiratory tract-leading to chronic cough and recurrent infections-but may also lead to pancreatic dysfunction, infertility, malnutrition, and more. D. Gene therapy is NOT a current treatment option for CF patients. However, there are studies right now that are investigating gene therapy. Current treatment includes a multidisciplinary approach, using treatments such as pancreatic enzyme and vitamin replacement, mucus clearing, possibly transplant, and more. E. It is necessary to provide calories at a HIGHER level than the recommended dietary allowance for age to maintain appropriate nutritional status, as CF patients usually have a malabsorption problem.

The mother of a 5-year-old boy calls your office asking if she should take her son to the emergency room or wait another day. She states that her son suddenly developed a "high fever" and is extremely tired. When you ask about her son's behavior, she states that he also seems very confused. She also noticed he had developed reddish-purplish spots on his extremities. What is the next best step in management of this patient? A. Have the patient make an appointment to come to your office today B. Tell the mother to take her son to an ED immediately C. Have the patient hydrate well over the weekend and follow up with you in a few days D. This patient most likely ingested something. Recommend ipecac to induce emesis and call 911

> B has been selected by the expert. A. Choice A is incorrect because any acutely altered mental status should raise serious concern, and therefore an office visit would probably not be adequate. B. Choice B is correct. This patient is exhibiting signs of sepsis, more specifically, of meningococcemia. Although it is important to replenish this patient's fluids and control his fever, it should not be done in an outpatient setting. This is a medical emergency! Sepsis can lead to altered mental status. Signs and symptoms of sepsis include: fever, nausea, vomiting, diarrhea, apnea/dyspnea, oliguria, pallor, tachypnea, tachycardia, lethargy, irritability, petechiae, purpura, tremors, and seizures. C. Choice C is incorrect. This patient is too ill to remain at home over the weekend with just supportive care. If he is bacteremic and septic he has a high mortality rate over the next 12 hours. D. Choice D is incorrect. There is no indication for inducing emesis, and this can present a risk of aspiration in a child with altered mental status.

A 6-year-old female comes to the clinic with a chief complaint of worsening right knee pain over the past month. On exam, you note generalized lymphadenopathy and splenomegaly. She coughs intermittently throughout the visit, and her mother explains that she is just getting over a cold. You note absence of tenderness, erythema, effusion or warmth over the hip, knee, or ankle joints. Her vitals are unremarkable except for a low-grade fever (100.8 F). Reviewing her chart, you note that she has lost 5 lbs since her visit 2 months ago. She sits with her right leg externally rotated but appears to be in pain despite trying several different positions, refusing to bear weight on that side. What is the most likely diagnosis? A. Reactive arthritis B. Leukemia C. Osteomyelitis D. Transient synovitis E. Septic arthritis

> B has been selected by the expert. A. Choice A is incorrect because reactive arthritis typically follows an infection outside the joint affecting the GU or GI tract (rather than upper respiratory tract), presenting two to four weeks following an infection. Children are commonly afebrile, and pain may involve multiple joints. B. Choice B is correct. Leukemia can present as bone pain due to replacement of bone marrow by leukemic cells. Patients may present with a limp or refusal to walk. Leukemia is associated with systemic symptoms such as low-grade fever, chronic/insidious joint pain, generalized LAD, weight loss, and/or hepatosplenomegaly. C. Choice C is incorrect because osteomyelitis typically presents with point tenderness over the bony joint and signs of joint inflammation. Osteomyelitis is most commonly associated with infection by Staph aureus or Strep pyogenes. The pain is worse upon weight-bearing, and fever is seen in about 50% of cases. D. Choice D is incorrect because this patient is showing systemic symptoms such as weight loss, hepatosplenomegaly, and generalized LAD. Transient synovitis would be high on the differential if this child were otherwise well-appearing with isolated involvement of the joint, most commonly the hip. Transient synovitis may follow an upper respiratory infection and usually resolves on its own within three to seven days. E. Choice E is incorrect because the history and physical are more suggestive of leukemia; however, septic arthritis may present similarly. Erythema, warmth, and swelling of a deep joint may not be readily apparent on exam. While fever is strongly associated with septic arthritis, it would likely be higher (> 38.5 °C) and the patient would have a more acute presentation. A CBC with differential would be useful in this situation. An ultrasound of the joint would also be able to identify an effusion and/or guide joint aspiration if septic arthritis was suspected. Most common organisms responsible for septic arthritis by age include: Staph aureus, GBS, E coli (neonates < 2 months); Staph aureus, Haemophilus influenzae, Strep pneumo (older children); Neisseria gonorrhea (adolescents).

A 7-year-old girl is brought to her pediatrician because of recurrent puffy eyes. She presented one week ago because of the same problem and was diagnosed with allergies. She was started on an intranasal steroid with no relief. Her mother states she has become increasingly tired and mentions that she has recently outgrown all of her shoes. The patient has no other symptoms and is at the 50th percentile for height and weight, is afebrile, and non-toxic appearing. Her heart and lung exam are normal. She has no hepatomegaly and no evidence of rash. What is your next step in diagnosis/management? A. Echocardiogram B. Urinalysis C. Flonase trial for an additional week D. CBC with manual differential E. Liver enzymes

> B has been selected by the expert. A. Choice A is incorrect. While CHF may present as edema, this patient has no complaints of respiratory symptoms or associated findings on physical exam (such as crackles or a murmur). An echocardiogram would not be the next step in diagnosis, even if CHF is expected. B. Choice B is correct. Because of the high suspicion of nephrotic syndrome, a urinanalysis would be the next step in diagnosis. This patient has symptoms of periorbital edema, lethargy, and pedal edema (she quickly outgrew her shoes). This test can be done in an outpatient setting and is not invasive (you do not have to stick her with a needle!). Nephrotic syndrome is defined as proteinuria > 50mg/kg. However, this cannot be detected with a UA. A UA dipstick will show high albumin concentration (graded as 3+ or 4+), and is used as a screening tool. Additional testing will be needed to confirm the diagnosis. C. Choice C is incorrect. Nephrotic syndrome is often misdiagnosed as allergies. This patient had a one-week trial of nasal steroid treatment with persistent and worsening symptoms. Other diagnoses and further tests should be pursued. D. Choice D is incorrect. While a CBC may show an increased hemoglobin and hematocrit secondary to volume depletion and thrombocytosis, it is not a test that will diagnose nephrotic syndrome. However, if the patient presented with edema and was febrile, a CBC may be warranted to rule out sepsis. E. Choice E is incorrect because the patient does not appear jaundiced and does not have hepatosplenomegaly. Liver enzymes would not be used as a screening test for nephrotic syndrome.

A full-term, 6-week-old baby is brought to the family physician for routine follow-up. At birth her weight, height, and head circumference were at the 50th percentile; she is now at the 5th, 10th, and 25th percentiles, respectively. She and her twin sister are exclusively breastfed; their mother has maintained a rigid every-four-hour feeding schedule since birth. Physical exam reveals a thin but otherwise healthy infant. What is the most likely cause of this infant's failure to thrive? A. Congenital heart disease B. Inadequate caloric intake C. Cystic fibrosis D. Milk protein allergy E. Inborn error of metabolism

> B has been selected by the expert. A. Congenital heart disease is incorrect for two reasons. First, CHD is a less common cause of FTT than option B. Second, CHD severe enough to cause FTT would present with associated signs (such as murmur, cyanosis, or hepatomegaly) and symptoms (such as easy fatigability). B. Inadequate caloric intake is correct because it is the most common cause of FTT. Furthermore, we have reason to believe that this particular infant is not receiving adequate calories. Breastfed infants should eat every two to three hours until 3 months of age; this infant eats every four hours and shares the milk supply with her twin. Twins can thrive on exclusive breastfeeding, but it requires that the mother consume extra fluids and calories and ensure that the twins both get enough to eat. C. Cystic fibrosis is incorrect because it is a less common cause of FTT than option B. CF condition affects fewer than 1 in 3000 Caucasians and is much less common in people of other ethnicities. Furthermore, malabsorption due to CF usually results in diarrhea, steatorrhea, and constant hunger. Respiratory conditions such as chronic cough or recurrent pneumonias may be present as well. D. Milk protein allergy is incorrect because, while it is fairly common (frequency of 2 to 3%), this infant has no symptoms. The most common symptoms include diarrhea, vomiting, abdominal pain, and allergic reactions ranging from urticaria to anaphylaxis. E. An inborn error of metabolism is incorrect because this category of conditions is quite rare and definitely less common than option B. Furthermore, the infant would likely have other symptoms (such as lethargy, vomiting, etc.).

Johnny is brought to your office by his parents for his 18-month well child check. His family recently moved into town, and this is the first time you are seeing him. Per mom, he was born at 32 weeks, and the details of his neonatal course are not clear. Parents share that he is not walking and they are very concerned. Johnny has met his social and language developmental milestones. Physical exam reveals spasticity, exaggerated deep tendon reflexes, and clonus in both of his lower extremities. An MRI of the brain is ordered, and the radiologist reports findings of periventricular leukomalacia. What is the most likely diagnosis? A. Niemann-Pick disease B. Spastic diplegia C. Athetoid cerebral palsy D. Autism spectrum disorder

> B has been selected by the expert. A. Neimann-Pick disease is a neurodegenerative disease that can present in children between the ages of 6 months and 2 years. It causes global delays as well as regression of milestones. Other signs and symptoms include hepatosplenomegaly, interstitial lung disease, and a macular cherry red spot. B. Spastic diplegia is a form of cerebral palsy, a non-progressive static encephalopathy characterized by delays in motor development. It may be associated with periventricular white matter abnormalities that are thought to be due to ischemia. These changes can be visualized on MRI. In spastic diplegia, the motor abnormalities are often greater in the legs than in the arms. C. Athetoid cerebral palsy involves motor deficits of the entire body, not just the legs. It is often caused by perinatal asphyxia and kernicterus, both of which damage the basal ganglia, cerebellum, and/or thalamus. D. Children with autism present with delays in language and social interaction, rather than with motor delays. Motor development is typically normal.

At a routine well-child visit, the frantic mother of your 4-year-old male patient states that she thinks her son has some developmental delays based on what she hears from other parents. Although he knows how to do such things as throw a ball and copy a circle, he cannot brush his teeth on his own, tie his shoes, or hop on one foot. According to the AAP's Bright Futures, which of the following are development milestones for typical 4-year-olds? A. Throw a ball overhand, ride tricycle, build tower of 6-8 cubes B. Hop on 1 foot, copy a cross, brush teeth C. Tie a knot, copy squares D. Mature pencil grasp, print some letters and numbers E. Skip, draw a person with 6 or more body parts

> B has been selected by the expert. A. This choice is incorrect. Throwing a ball overhand, riding a tricycle, building a tower of 6-8 cubes, and copying a circle are developmental milestones for 3-year-olds. B. This choice is correct. A a normally developing 4-year-old should be able to hop on 1 foot, copy a cross, pour/cut/mash their own food, and brush teeth. C. This choice is incorrect. These are milestones for 5-year-olds. D. This choice is incorrect. These are milestones for 5-year-olds. E. This choice is incorrect. These are milestones for 5-year-olds.

A previously healthy 4-year-old girl is brought to her pediatrician because her parents have noticed that she has been less active than usual for the past three weeks. Her father explains that it is difficult to get his daughter out of bed in the mornings and that she no longer plays outside with her older brother. Physical examination is notable for a temperature of 38.4 C, heart rate of 125 bpm, pallor, truncal bruising, and diffuse lymphadenopathy. The remainder of the exam, including a thorough neurologic assessment, is unremarkable. Which of the following is the most likely diagnosis? A. Aseptic meningitis B. Kawasaki disease C. Non-accidental trauma D. Acute lymphoblastic leukemia E. Mononucleosis

> D has been selected by the expert. A. Aseptic meningitis is incorrect because it cannot explain this child's pallor, bruising, or lymphadenopathy. While aseptic meningitis can present with lethargy and low-grade fevers, the remainder of her symptoms are not consistent with meningitis. B. Kawasaki disease (KD) is incorrect because this child does not meet the criteria for diagnosis. She has not had fevers for at least five days and has no other symptoms consistent with KD. She has diffuse lymphadenopathy (not unilateral cervical LAN > 1.5cm in size) and also lacks bilateral nonsuppurative conjunctivitis, upper respiratory tract mucosal changes, extremity erythema or swelling, and polymorphous rash. C. Non-accidental trauma is incorrect because it does not account for the systemic inflammatory signs found on physical exam: fever and lymphadenopathy. Do remember, however, that unexplained bruising should always raise the suspicion of child abuse. D. Acute lymphoblastic leukemia is correct. The child presents with evidence of anemia (fatigue, tachycardia, pallor) and thrombocytopenia (unexplained bruising). Failure of two or more hematologic cell lines should always raise suspicion for malignant invasion of the marrow. Furthermore, the child's chief complaint, fatigue, is the most common presenting symptom of acute leukemia. Finally, the incidence of ALL peaks at age 4 years. E. Mononucleosis is incorrect because the diagnosis does not explain the apparent bone marrow failure evidenced by anemia (fatigue, tachycardia, pallor) and thrombocytopenia (bruising). This child's subacute presentation with marked fatigue, low-grade fevers, and lymphadenopathy is suggestive of mononucleosis, and this diagnosis should remain high on the differential. In a young child, however, one must first rule out acute leukemia.

A 2-month-old male is brought to the ED after his mother found him in his crib not breathing. She says he had no color and was still when she found him, but quickly regained his color. While you are examining him he starts having a tonic-clonic seizure and subsequently is found to have a temperature of 96 F, HR 200 bpm, and RR 18 bpm. On exam he cries intermittently, does not track you with his eyes, has a tense, full fontanelle, and decreased tone throughout. You also notice a healing bruise on his left arm. After assessing circulation, airway, and breathing you obtain IV access. What is the next step in your diagnostic workup? A. Skeletal survey B. Lumbar puncture C. Head CT D. Head MRI E. Social work consult

> C has been selected by the expert. A. Although in cases of child abuse a skeletal survey should be included in the diagnostic work up, it is not the initial test of choice. B. Meningitis is on the differential; however, this child is afebrile and consolable making this diagnosis less likely. Other causes of altered mental status and seizures should be ruled out first. C. This choice is correct because head CT is highly sensitive for an intracranial bleed, such as a subdural hematoma, can be quickly carried out in the emergency setting, and may require urgent intervention. D. This choice is incorrect because MRI is less sensitive for an early bleed than CT, and would require sedation. However, MRI can be used as a confirmatory study after the child is stabilized. E. This choice is incorrect because this infant is not stable. However, a consult would be appropriate after the infant is stabilized.

Katie is a 5-year-old girl with 10-day history of swelling of her face, especially around the eyes. Her mother has also noticed that her pants have become too tight for her, and that she has gained nearly 5 pounds despite a decreased appetite. About a week prior to the start of the swelling, her mother recollects an episode of rhinorrhea, cough, and sore throat. Urinalysis shows no red blood cells or casts, but you have no other data from urinalysis due to a lab error. On exam, temperature is 98.8 F, heart rate is 95 bpm, blood pressure is 95/65 mmHg. Her face is diffusely swollen. Heart and lung exams are normal. Abdominal exam shows some abdominal fullness but no masses or organomegaly. Both feet appear slightly puffy. Which of the following is the most likely cause? A. Sinusitis B. Nephritis C. Nephrotic syndrome D. Congestive heart failure E. Allergic reaction

> C has been selected by the expert. A. Periorbital swelling and redness can result from inflammatory edema secondary to ethmoid, frontal, or maxillary sinusitis. Sinusitis or allergic rhinitis could cause more prolonged symptoms, but are usually accompanied by cough or runny nose. Sinusitis would not be associated with an increase in weight, or cause increasing abdominal girth. B. Nephritis classically presents with proteinuria, hematuria, and hypertension. This patient's blood pressure is normal. Although the urinalysis unfortunately was inconclusive on the level of proteinuria, there was no hematuria. C. The constellation of history and physical exam findings, with a recent viral infection and subsequent edema, suggests nephrotic syndrome. Minimal change disease is the most common form of nephrosis in pediatrics. Changes in the podocytes of the glomerular apparatus allow significant proteinuria (> 3.5 g/day), which leads to hypoalbuminemia and eventually interstitial edema and ascites. This would explain the patient's periorbital swelling, increasing abdominal girth (ascites), and weight gain (pure fluid overload). D. Congestive heart failure could cause generalized edema due to poor ventricular function, but the cardiothoracic exam would be expected to be abnormal, with crackles in the lungs and murmur or gallop over precordium. E. Periorbital swelling from an allergic reaction is usually abrupt in onset, and often accompanied by an urticarial rash. This patient has no rash, and the distended abdomen is not explained by an allergic reaction.

A 15-month-old is able to stand on his own, walk backward, and throw objects underhand. He is unable to draw/scribble or grasp markers. He can say one syllable words "ma" and "da" but his words are unintelligible. He holds a sippy cup with help. He does come when called, plays with a ball and waves bye-bye by imitating his parents. Does this baby have developmental delay, and, if so, within which domains? A. Normal variant B. Gross motor and fine motor delay C. Fine motor and language delay D. Language delay and gross motor E. Social and language delay

> C has been selected by the expert. A. Incorrect. A 15-month-old child should be more developed in language and fine motor skills B. Incorrect. This child is able to walk backward and throw objects without difficulty. However, he does exhibit some delay with fine motor skills with his inability to scribble or hold a sippy cup on his own. C. Correct. A 15-month-old child should be able to scribble, use a cup, and/or stack 2 blocks. His word count should be three to six words at this stage of development. D. Incorrect. Although this child does have some evidence of language delay, his gross motor skills are at a normal stage of development. E. Incorrect. Socially this child is within normal developmental range since he comes when called and he imitates others.

Kenny is a 12 year-old male who comes to your clinic with a chief complaint of crampy abdominal pain. His mother tells you that sometimes he wakes up from sleep due to the pain. He also has diarrhea that sometimes has blood in it. When asked about stressors in his life, his mother sighs and tells you that she is recently divorced and had to move Kenny to a new school. On physical exam, he appears small for his age. Abdomen is soft, non-distended, but tender to palpation at the RUQ. On rectal exam, you note anal skin tags and an anal fistula. Skin exam shows red tender nodules on his shins. Labs show a microcytic anemia. What is the next best step in management? A. Reassure and refer to psychiatry B. Start omeprazole and antibiotics C. Colonoscopy D. Start mesalamine E. Obtain IgA endomysial antibody and IgA anti-tissue transglutaminase antibody

> C has been selected by the expert. A. Reassure and refer to psychiatry is incorrect. This is the right management for functional abdominal pain, which is the most common cause of abdominal pain in this age group. However, bloody diarrhea cannot be explained by functional abdominal pain and suggests a more serious diagnosis. B. Start omeprazole and antibiotics is incorrect. This is the correct management of PUD caused by H. pylori. PUD cannot account for the perianal disease, diarrhea, or erythema nodosum. C. Colonoscopy is the best answer. Kenny likely has IBD (Crohn's disease or ulcerative colitis [UC]). He has crampy abdominal pain and intermittently bloody diarrhea, crampy abdominal pain (that wakes him up at night), perianal disease, and erythema nodosum. He also has microcytic anemia, likely from chronic blood loss. Colonoscopy with biopsies will allow you to diagnose Crohn's disease (or UC) prior to treating it. The diagnosis begins with a colonoscopy to obtain tissue biopsies as well as blood tests (p-ANCA, ASCA). D. Start mesalamine is incorrect. Mesalamine is the first-line treatment for Crohn's disease. However, you have not yet established a diagnosis. E. Obtain IgA endomysial antibody and IgA anti-tissue transglutaminase antibody is incorrect. This is the right diagnostic test for celiac disease. However, celiac disease would not present with blood in the stool or other extraintestinal signs. Celiac can be associated with dermatitis herpetiformis, an itchy, papulovesicular rash.

A 10-month-old male is brought to the emergency room by his very concerned and frantic grandmother. Earlier that day, she retrieved the child from his mother's new boyfriend, who had been watching him while his mother was at work. The grandmother makes it very clear she does not approve of this new boyfriend, and she is concerned that he is rough with her grandson. She demands that her grandson be worked up for injuries and that a restraining order be placed against the boyfriend. Which of the following finding does NOT indicate that a child is being physically abused? A. Retinal hemorrhages on fundoscopy B. A concaved, crescent-shaped mass on head CT C. A spiral fracture of the tibia D. Two posterior rib fractures E. A metaphyseal fracture of the wrist

> C has been selected by the expert. A. Retinal hemorrhages are commonly found in victims of shaken baby syndrome. Of these patients, 65% to 90% typically present with diffuse, bilateral retinal hemorrhages. This typically occurs in younger infants. B. Subdural hematomas occur when bridging vessels are torn during the act of shaking or throwing a baby. It is achieved by extreme rotational cranial acceleration force to the brain. Diffuse axonal injury to the neurons also occurs. CT scans and MRIs can detect the bleed. C. Also called a "toddler's fracture," fracture of the tibia is a commonly occurring fracture in young, ambulatory kids. It is not a sign of abuse. Toddler's fracture is described as a subtle, non-displaced oblique fracture of the distal tibia in kids aged 9 months to 3 years. The child will usually present with acute onset of limp and refusal to bear weight on one leg. It usually occurs when a toddler falls while twisting, or gets a foot caught and falls while trying to free the foot. D. Posterior rib fractures are associated with shaken baby syndrome. As the adult squeezes the child's thorax in the process of shaking the child, the posterior ribs are at risk of being fractured. E. Metaphyseal fractures, also called "bucket handle" or corner fractures, are caused by torsional force on the limb, or by violent shaking. While these can occasionally occur in older children who test the limits of their limbs, this would be unlikely in an 10-month-old. You can never be reassured by a metaphyseal fracture, and should always have child abuse on your differential when this type of fracture is apparent.

A 3-year-old female is at the pediatrician's office for continued right knee pain after a ground-level fall six weeks ago. The patient is UTD on all immunizations, has no significant PMH, and no recent illnesses. Mom reports the patient complains of pain mostly in the morning when going to daycare but doesn't seem to be bothered by it while playing outside in the afternoon. On exam the patient's vitals are all within normal limits. Her physical exam reveals a well-appearing toddler who walks stiffly and avoids bending her right knee. The knee has a mild effusion but no obvious erythema. There is pain with passive flexion and extension of the right knee. During the exam the girl tells you her left ankle also hurts, which mom had forgotten about but says started hurting the same time as the right knee. Her CBC is normal, while her ESR and CRP are mildly elevated. Which of the following is the most likely cause of this patient's condition? A. Septic arthritis B. Leukemia C. Juvenile idiopathic arthritis D. Transient synovitis of the hip E. Bacterial osteomyelitis

> C has been selected by the expert. A. Septic arthritis is incorrect because the patient has been generally well for the past six weeks and presents with mild swelling but no erythema of her joint. She is afebrile with no leukocytosis on CBC. While each of these alone would not rule out septic arthritis, the combination of all these factors together make septic arthritis unlikely. Patients with septic arthritis are often febrile. Joint fluid analysis often reveals elevated WBCs and predominant neutrophils (although a joint tap in this scenario is not necessary based on the presenting symptoms and exam). Bacteria may also be cultured from the joint fluid. Additionally, her CRP and ESR are only mildly elevated. One would expect to see a greater elevation of these markers of inflammation in septic arthritis. B. Leukemia is incorrect because the patient does not present with other systemic symptoms such as fever, weight loss, lymphadenopathy, hepatosplenomegaly, petechiae, or bruising. This patient also has pain that improves with activity, as opposed to a more chronic pain that does not improve with position or movement that would be expected with leukemia. Additionally, a normal CBC without thrombocytopenia or anemia makes this answer unlikely. C. Juvenile idopathic arthritis (JIA) is correct for several reasons. Pauciarticular juvenile arthritis is the most common type of JIA (60% of JIA) and causes pain in four or fewer joints for six or more weeks. This patient is generally well even after six weeks of pain, which would be unlikely if this patient had septic arthritis. Her pain improves with activity, and the ESR/CRP are only mildly elevated. On exam, she has a mild effusion but no obvious erythema. In cases of systemic JIA, patients may have a rash which lasts only a few hours (evanescent) that is also macular and salmon, and high-spiking and appears periodically (once or twice a day); however, this form of JIA is not consistent with this patient's history. D. Transient synovitis of the hip is incorrect because this condition, while acute, generally resolves in three to four days and this patient has had pain for six weeks. Transient synovitis typically affects the hip, but patients may also report knee or inner thigh pain. Commonly, patients may have a history of a recent URI. Additionally, patients with transient synovitis do not typically present with a joint effusion as is seen in this patient. E. Bacterial osteomyelitis in children, usually hematogenous in origin, presents with acute onset of bone pain and fever and may involve a contiguous joint. This patient is well appearing and has no leg tenderness.

An 11-month-old boy is brought to the ED by ambulance. His father called 911 after the patient's eyes deviated to the left as his arms and legs were twitching. During this time he was unresponsive. He has had a tactile fever for three days, and parents mention that he has not been as playful as usual during this time as well. His parents have not had him vaccinated due to personal beliefs. In the ED his vital signs are T 39.1°C, HR 155 bpm, RR 28 bpm, BP 100/65 mmHg, O2 100% (on RA). He does not cry but whimpers during most of your physical exam (including when you look in his ears). You order a CBC and metabolic panel, which are significant for a leukocytosis with a left shift and mild acidosis. Urinalysis and blood/urine cultures are pending. Which of the following additional studies would you obtain? A. Chest x-ray B. Toxicology screen C. Lumbar puncture D. Electroencephalogram (EEG)

> C has been selected by the expert. A. The history given is not suggestive of any pathologic lung process occurring in this patient (no cough, respiratory distress, or desaturation). While it is possible that pneumonia could be causing his fever, a chest x-ray is not indicated at this time. B. It is important to consider toxin ingestion in a patient with altered level of consciousness and seizure. Most toxicology screens, however, are limited to drugs of abuse and may not detect many of the toxins that children commonly ingest. Furthermore, this patient's fever and leukocytosis make infection a much more likely diagnosis. C. In a young child with fever and altered level of consciousness we should always have a high suspicion for meningitis. This patient's parents expressed concerns about his behavior at home before his seizure, and his mental status during your examination is not normal. While very few patients presenting with febrile seizure actually have meningitis, this patient's lack of immunizations put him at increased risk. Furthermore, clinical signs of meningitis in patients under 12 months of age can be very subtle, and so a high level of suspicion is important. A lumbar puncture will help rule in or out meningitis and guide treatment. Note that in some cases the clinician will request a head CT prior to performing a lumbar puncture if there are concerns about increased intracranial pressure. A head CT in itself may not be helpful in the evaluation of a patient with a seizure, although it may be useful in cases where trauma is suspected, or to look for calcifications (such as with cytomegalovirus infection or tuberous sclerosis). D. For children with focal, recurrent, or complex seizures, an EEG to establish a diagnosis of epilepsy. Sixty percent of children and infants may have an interictal epileptiform abnormality. In this case it will not help to guide management acutely.

A 3-year-old boy presents for a follow-up visit after being diagnosed with iron deficiency anemia. He is currently receiving oral iron supplements, 2 mg/kg of elemental iron daily. He has a dietary history of eating mostly sweet, bland, low-texture foods. What strategies may be used to improve his diet? A. Continue bottle-feeding B. Encourage eating small amounts of food throughout the day (grazing) C. Gradually introduce new foods and slowly decrease his old favorites D. Bargain and cajole with the child E. No change is needed; bland, low-texture foods are optimal for a child this age

> C has been selected by the expert. A. This choice is incorrect because the child should stop bottle-feeding now to improve his diet. Children usually stop requesting the bottle a few days after it has been discarded. B. This choice is incorrect because the child should be encouraged to restrict eating to 3 meals and 2 snacks per day, instead of "grazing" throughout the day. C. This choice is correct, because gradually introducing new foods and slowly decreasing his old foods will likely ease the transition to healthier diet choices and encourage long-term adjustment. D. This choice is incorrect because bargaining and cajoling is unlikely to be effective. He should be presented with healthy options only, and dessert should not be used as an incentive for healthy eating. E. This choice is incorrect because the child's current diet is not optimal. He should be eating a varied diet with the recommended servings of fruits and vegetables per day.

A 12-day-old baby girl is brought to the ED by her foster mother due to fussiness and tactile fevers. The baby's teenage biological mother did not receive prenatal care and delivered her baby at home. On further questioning, you find out that the patient has had only two wet diapers per day and two loose green stools per day. On exam, the patient is irritable and her anterior fontanelle is tense. Which of the following diagnoses are of emergent concern at this time? A. Down syndrome B. Fetal alcohol syndrome C. Group B strep sepsis/meningitis D. Meconium ileus E. Poor weight gain

> C has been selected by the expert. A. This choice is incorrect, because although the patient is likely to have missed newborn screening for Down syndrome, her most emergent concern is her fever and possible sepsis. B. This choice is incorrect. Although the patient is at risk for fetal alcohol syndrome (since her mother did not have prenatal care and may not have had counseling regarding abstinence from alcohol), her most urgent concern is her fever in a less than 1-month-old infant. C. This choice is correct. The patient's mother did not have prenatal care and likely did not have screening for group B strep during pregnancy. She also delivered at home and would not have had access to antibiotics during delivery. Group B strep is a common and serious cause of sepsis and meningitis in newborns. D. This choice is incorrect because the patient reportedly has passed stool since delivery. E. This choice is incorrect. Infants may lose weight during the first few days of life, but are expected to regain their birth weight by 2 weeks of age.

A 3-year-old boy described by his mother as a picky eater comes in for a regularly scheduled well-child visit. His mother complains that he has had less energy than usual for the past few months. There is a high clinical suspicion he is anemic. Which of the following is most correct? A. The most cost-effective test to diagnose anemia is a CBC. B. Lead screening is never warranted since a 3-year-old is usually not mouthing objects. C. The most likely cause of anemia in the question is picky eating resulting in low iron intake, which would cause microcytic anemia. D. The most common cause of anemia in this situation is folate deficiency. E. If anemia is due to poor nutrition, restarting the bottle will help the child recover the most.

> C has been selected by the expert. A. This choice is incorrect. The most cost-effective method to diagnose anemia is a screening hemoglobin. B. This choice is incorrect. Although most 3-year-olds stop mouthing objects, they may have been exposed to lead in the past but were not symptomatic at the time. C. This is the correct choice. The most likely cause of anemia in the question is picky eating, which can result in insufficient iron intake. Low iron intake causes a microcytic anemia. A girl with menometrorrhagia would present with iron deficiency, and her MCV also would be indicative of microcytic anemia. D. This choice is incorrect. Iron deficiency is the most common cause of anemia in this scenario. Note that folate deficiency may be associated with a goat's milk diet. E. This choice is incorrect. It is best to stop the bottle by age one year. Solid foods provide more complete nutrition, including iron. An additional concern of prolonged bottle usage is the development of dental caries.

An 8-year-old obese male comes to the clinic with a chief complaint of right knee pain with the right foot medially rotated. On an exam the right knee is neither swollen nor erythematous but he is noted to have a limited ROM of the right hip. In addition, when he lifts his right leg, it externally rotates. The patient did not have a URI or any trauma preceding the onset of pain. The vital signs are normal at the time of the visit and he is well appearing and afebrile. What is/are the best next step(s) in management? A. AP and lateral x-ray followed by casting and crutches B. Bone scan C. AP and lateral x-ray followed up by internal reduction of the femoral head D. Aspiration of the knee E. Observation and weight reduction counseling

> C has been selected by the expert. A. This choice would be correct if we suspected Legg-Calves-Perthes disease, or avascular necrosis of the capital femoral epiphysis. But the knee pain and physical exam findings are not consistent with this diagnosis. B. This choice is incorrect because the bone scan is used to diagnose an acute hematogenous osteomyelitis. This child is not febrile and has no localized bone pain and is therefore less likely to have osteomyelitis. C. This choice is correct because the AP and lateral x-rays are needed to diagnose a slipped capital femoral epiphysis, which is considered an emergency. This patient's age group, his obesity, and the description of the external rotation of the right leg when the hip is flexed all suggest this diagnosis. D. This choice is incorrect because the aspiration and drainage via arthroscopy is done for a septic joint. Based on the physical exam and vital signs, the patient is unlikely to have an infection of the bone or joint. E. This choice is incorrect because observation is not optimal when the patient is likely to have an urgent condition that needs to be fixed. Weight reduction counseling might be a good option after his problem is addressed.

Jane is an 8-year-old girl who presents to your clinic for follow-up after being hospitalized for status asthmaticus. She has just completed a 10-day course of systemic steroids. Given her history of moderate persistent asthma, her outpatient regimen includes Advair, a combined steroid and bronchodilator. She was also diagnosed with ADHD one year ago and was started on Concerta, 18 gm PO once a day. Her BMI today is at the 83rd percentile for her age, and her blood pressure is at the 98th percentile for her age. What is the most likely cause of her stage I hypertension? A. Obesity B. The blood pressure cuff is too big C. Medications D. Renal insufficiency E. Neurofibromatosis 1

> C has been selected by the expert. A. While obesity is a risk factor for hypertension, Jane is not overweight. A child is considered overweight when his or her BMI is between the 85th and 95th percentile for age. Obesity is considered > 95th percentile. B. Blood pressure would be falsely decreased if the cuff was too big, and, inversely, falsely elevated if the BP cuff was too small. C. Both steroids and amphetamines can cause increases in blood pressure, especially when used in combination. Steroids increase blood pressure by mimicking endogenous cortisol and the sympathetic fight or flight response. Amphetamines mimic norepinephrine, stimulating alpha and beta adrenergic receptors, causing an overall increase in blood pressure. D. Renal insufficiency can be a cause of secondary hypertension, but is unlikely in this patient. Risk factors that warrant investigation of renal causes for hypertension include recurrent UTIs, umbilical arterial/venous lines placed while a child was in the ICU, and a family history of renal disease. E. NF-1 can be associated with hypertension as a result of vascular malformations that affect blood supply to the kidneys; however, this child does not present with any signs of NF-1 (café au lait macules, neurofibromas, optic gliomas, lisch nodules).

An 18-year-old mother with her 3-month-old son arrives at urgent care clinic with a chief complaint of "my baby stopped breathing!" She and her baby are rushed into a triage room, where her son is noted to be very lethargic with increased work of breathing. As vital signs are being obtained, the mother reports "my baby stopped breathing in the car coming here, and didn't start again until I reached over and jostled his car seat!" Mom then shared that "my boyfriend said he rolled off the couch last night. Could it be related?" Mom also stated that her son hasn't been as active as usual, and has been vomiting occasionally. Physical exam is notable for a respiratory rate of 70 bpm with intercostal retractions and crackles in the right lower lung fields posteriorly. You admit this patient with the diagnosis of pneumonia for empiric antibiotics and support, pending additional evaluation. CXR subsequently demonstrates a RLL infiltrate and faint, vertical lines on several posterior ribs bilaterally. What is the best next step in management? A. Obtain a PTH level B. Sweat chloride testing C. Skeletal survey (more x-rays) D. Anticipatory guidance about appropriate car seat usage E. Head ultrasound

> C has been selected by the expert. A. While there is certainly the possibility of increased risk of fracture in infants who have with primary hyperparathyroidism, this is an extremely rare disorder. While it may lead to skeletal demineralization, this would more likely be seen on x-ray as decreased bone density. Importantly, this pattern of posterior rib fractures, which may be caused by squeezing the infant's thorax, should lead the clinician to suspect abuse. B. Sweat chloride testing for the diagnosis of cystic fibrosis should be done in an infant with pneumonia and failure to thrive. This one episode of pneumonia and no evidence of FTT does not warrant a workup at this time. C. In this case a skeletal survey is essential. Posterior rib fractures are always concerning findings. While treatment of the patient's pneumonia has been initiated, a complete skeletal survey will screen for other worrisome findings, including multiple fractures in different stages of healing, fracture of the femur or tibia in a non-walking child, and skull fractures. D. It is true that this mother needs to be better informed about car seat use, because the infant car seat should never be in the front seat of a car. This needs to occur prior to discharge, but does not have the same urgency as (C) above. E. Although head trauma (from falling from the couch) could result in an intracranial bleed and lethargy, the best test for such a bleed and/or skull fracture would be a CT scan, not an ultrasound.

A young couple presents to the ED with their 2-month-old son complaining of excessive sleepiness and difficulty arousing him after his nap. Per the parents, the PMH and prenatal course are unremarkable, except that the patient has always been very fussy and would often cry despite being held and cradled. He is cared for during the day by his babysitter. Today he had been in his usual state of fussiness when the babysitter arrived, and they returned to find him napping quietly in his cradle but could not arouse him from sleep when it came time for his feeds. He finally opened his eyes after several minutes of gentle nudging but seemed to quickly fall asleep again. On exam, patient is afebrile with poor tone and is only mildly responsive to painful stimuli. Eye exam shows dilated pupils and an ophthalmology consult reveals retinal hemorrhages. What is the most likely diagnosis? A. Bacterial meningitis B. Infant botulism C. Intoxication D. Closed head injury E. Metabolic disorder

> D has been selected by the expert. A. Bacterial meningitis is more common during the first month of life than at any other point and is usually secondary to GBS infection. While meningitis could account for some of the symptoms seen in our patient, such as lethargy and abnormal tone, fever is the most common symptom and is notably lacking in our patient. The retinal hemorrhages noted are also inconsistent with this diagnosis and point more towards a closed head injury secondary to child abuse. B. Infant botulism results from ingestion of C. botulinum spores that germinate in the infant's gastrointestinal tract leading to constipation, symmetric weakness, feeding difficulties, drooling, irritability and weak cry. It is commonly believed to be associated with ingestion of honey but can also be caused by ingestion of environmental dust containing C. botulinum spores. This diagnosis, while on the differential, is unlikely given the lack of symmetric weakness noted in the history and presence of other physical exam findings more consistent with child abuse. C. Intoxication caused by accidental ingestion of medications or toxins could result in symptoms similar to those seen in our patient such as lethargy and poor tone but is unlikely given the absence of any medication exposure noted in the history. The patient is also too young, at one month of age, to be able to accidentally obtain and ingest any medications or substances which may have fallen on the floor without an adult noticing-unlike children who are capable of crawling or walking-making this a less likely diagnosis. D. Closed head injuries and retinal hemorrhages in infants and children are secondary to violent shaking or throwing, resulting in tearing of the bridging vessels. Retinal hemorrhages, as seen in our patient, are pathognomonic for shaken baby syndrome. Other signs and symptoms include stiffness, constant crying, seizures, difficulty to arouse, decreased appetite and excessive sleeping. Victims often have no other signs of physical abuse (e.g., bruises). E. Most inborn errors of metabolism are rare and caused by single enzyme deficiencies leading to disruption of the metabolic pathway. While errors of metabolism could account for some of the symptoms seen in our patient, such as lethargy and abnormal tone, a negative newborn screen makes this diagnosis less likely. The retinal hemorrhages noted are also inconsistent with this diagnosis and point more towards child abuse.

Casey's parents just learned that Casey's screening test for CF was abnormal. What other signs and symptoms might you expect this patient to develop? A. Bloody stools B. Precordial thrill C. Reflux D. Greasy stools E. Diaphoresis during feeds

> D has been selected by the expert. A. Bloody stools would be expected in a child who had gastroenteritis or intussusception, or other intestinal anomaly. You might also expect mucus in the stool in cases of gastroenteritis or intussusception. B. A precordial thrill would be indicative of a cardiac problem, but cardiac issues are not associated with CF. C. There is no association between gastresophageal reflux disease and CF. D. CF causes exocrine insufficiency and ultimately results in fat malabsorption. This causes frequent foul-smelling and greasy stools. E. Diaphoresis during feeds would raise concern for congestive heart failure. The classic associated signs include tachypnea, tachycardia, and hepatomegaly.

Billy, a 7-year-old boy, presents to the clinic with complaints of headaches and episodes of feeling sweaty and flushed. He also reports that at times he feels as if his heart is racing. Billy was full term, had an uncomplicated birth, and has been otherwise healthy until now. On exam his BP is 120/80 mmHg and is the same in his upper and lower extremities. His weight and height are in the 50th percentile for his age. What is a likely cause of Billy's hypertension? A. Coarctation of the aorta B. Renal vascular disease C. Renal insufficiency due to renal scarring D. Catecholamine excess E. Primary hypertension

> D has been selected by the expert. A. Coarctation of the aorta should be suspected in a child with elevated BP (usually > 99th percentile), little family history of HTN, and a discrepancy between upper and lower extremity BPs. Some children with coarctation of the aorta may go undetected until presenting with hypertension at a school-age visit. It is important to pay special attention to the femoral pulses and to document BP measurement in a lower extremity. Billy's BPs in his upper and lower extremities are the same, making this diagnosis less likely. B. Umbilical arterial or venous lines as neonate (most often in premies) can predispose a child to renal vascular disease. Billy had an uncomplicated birth and did not go the neonatal ICU, making it less likely that his hypertension is due to renal vascular disease secondary to an umbilical arterial or venous line as a neonate. C. Recurrent urinary tract infections in childhood are one of the leading causes of hypertension and renal insufficiency later in life due to renal scarring following infections. UTIs are more common in girls. Billy does not have a stated history of urinary tract infections, and his palpitations and flushing are not consistent with hypertension secondary to UTIs. D. Catecholamine excess (pheochromocytoma or neuroblastoma) should be suspected in a child who is hypertensive and has episodes of sudden sweating, flushing, or feels that his heart is racing. Billy is exhibiting these signs and a urine catecholamine testing would be appropriate in this case. E. Most hypertension in children over 6 years of age, and in adolescents, is due to primary HTN. Obesity is an important correlate. Billy's flushing and racing heart would not be fully explained by primary hypertension. Also, Billy is an appropriate weight, making this diagnosis less likely.

George is a 7-year-old boy frequently in trouble at school for being disruptive and inappropriately talkative in class, not following directions set by his teacher, and not working well with classmates during group activities. His mother relates that at home George is always on the go, sleeping only 6 to 7 hours a night. He does not follow her rules all the time either, including not doing his homework, and sometimes putting himself in danger by doing things she tells him not to do, such as running away unaccompanied. Which of the following is the most likely diagnosis? A. Bipolar mood disorder B. Anti-social personality disorder C. Conversion disorder D. ADHD E. Rett syndrome

> D has been selected by the expert. A. Depression may be responsible for the inattention this child exhibits in school. It is not uncommon for childhood depression to lead to bipolar disorder, in which hyperactivity and impulsivity comprise the manic phase of the disorder leading to a decreased need for sleep. However, the symptoms of depression and mania present in separate phases, not concurrently. The mnemonic commonly used for mania is DIGFAST (distractability, irresponsibility, grandiosity, flight of ideas, agitation, sleep decrease, talkativeness). The mnemonic for depression is SIGECAPS (sleep disturbance, loss of interest, guilt, energy loss, concentration impairment, appetite changes, psychomotor retardation, suicidal ideation). B. Symptoms of anti-social personality disorder include inability to conform to societal norms, disregard of the rights of others, and often criminality. These individuals often exhibit impulsiveness due to their lack of consideration of the consequences of their actions. Males are affected more than females. However, a diagnosis of anti-social personality disorder can be made only in individuals older than 15 years, earlier than which a diagnosis of conduct disorder is appropriate. C. Conversion disorder is a subtype of somatoform disorder. Somatoform disorder would be suspected if no cause could be identified for reported physical symptoms. The drive is unconscious on the part of the patient, and symptoms are not intentionally produced or faked. Conversion disorder is more common in adolescents and involves a sudden loss of sensory or motor functioning. When the patient consciously creates physical symptoms this is referred to as factitious disorder. D. ADHD is characterized by the triad of impulsivity, hyperactivity, and inattention. Other symptoms include motor impairment and emotional labiality. ADHD is typically diagnosed before the age of 7 but persists into adulthood. Intelligence is usually normal, but individuals with ADHD commonly perform more poorly academically than would be expected for their IQ. E. Rett syndrome is an X-linked pervasive developmental disorder seen only in females; affected males die in utero or at birth. The characteristic symptoms involve regression of language and development, intellectual disability, ataxia, and hand-wringing. This disorder is typically diagnosed earlier, at about age 1-4.

8-year-old Jenny presents complaining of intermittent, crampy abdominal pain that has persisted over the last three months. The pain is nonspecific, nonfocal, and not associated with any other systemic symptoms such as fever, chills, weight loss, nausea, vomiting or diarrhea. The pain also seems to occur more frequently during the week and not as often on weekends. The abdominal exam is normal. Jenny is given a diagnosis of functional abdominal pain and scheduled for a one-month follow-up. Six months later, she returns to the clinic complaining of more frequent, more severe abdominal pain that is waking her up at night. She also reports a week of diarrhea containing mucus and blood without associated fever or vomiting. Review of her growth chart demonstrates a slowing of weight gain and a drop in height velocity. What is the most likely diagnosis? A. Irritable bowel syndrome B. Giardiasis C. Celiac disease D. Crohn's disease E. Henoch-Schonlein purpura (HSP)

> D has been selected by the expert. A. Irritable bowel syndrome is a symptom-based diagnosis with chronic abdominal pain, discomfort, bloating, and alteration of bowel habits. It is often a diagnosis of exclusion, with no known organic cause. Other symptoms include diarrhea or constipation. B. Giardiasis is a parasitic disease caused by flagellate protozoan that inhabits the digestive tract leading to decreased appetite, diarrhea, hematuria, loose or watery stool, stomach cramps, bloating, excessive gas, and burping. Many people with giardia infection are asymptomatic. Untreated symptoms may last six weeks or longer. Diagnosis is made via stool microscopy, ELISA, or entero-test with gelatin capsule and thread. C. Celiac disease is an autoimmune disorder of the small intestine characterized by diarrhea, failure to thrive, and fatigue. A rash known as dermatitis herpetiformis (an autoimmune cutaneous eruption) can also occur. D. Crohn's disease is most consistent with this presentation, as it affects GI tract from mouth to anus, leading to abdominal pain, diarrhea (can be bloody), vomiting, or weight loss. Extraintestinal symptoms include skin rashes, arthritis, and fatigue. Fever, fistula, and perianal complications are also common. E. Henoch-Schonlein purpura (HSP) presents with abdominal pain, palpable purpura on the lower extremities, joint pain, and often kidney involvement. It is an acute systemic vasculitis characterized by deposition of immune complexes containing IgA. Abdominal pain is colicky often with nausea, vomiting, constipation, or diarrhea.

A 4-year-old child is refusing to walk over the course of a week. Her mother recalls that she fell off her bike yesterday. On exam, she is afebrile, but has decreased ROM of her hip. You review her file and note that she is up-to-date on her immunizations and she was last seen three weeks ago for a self-limited episode of diarrhea that she developed while visiting family in rural Mexico. Aspiration of her affected hip joint reveals slight increase in inflammatory cells but normal chemistries and a negative gram stain. Culture is pending. Which of the following is the most likely diagnosis? A. Osteomyelitis B. Trauma C. Transient synovitis D. Reactive arthritis E. Septic arthritis

> D has been selected by the expert. A. Incorrect. The gradual onset of refusing to walk is consistent with osteomyelitis, and only half of patients present with fever. However, this patient is not acutely ill as one would expect with hematogenously acquired osteomyelitis. While this is a possible diagnosis, it is further down on the differential diagnosis. B. Children who present with limp secondary to trauma may not have any findings visible on x-ray for a few weeks. Though a fall from a bike could certainly aggravate a weakened bone or joint, this patient's limp developed over the course of a week. C. While transient synovitis could certainly present with pain, it is more likely to present acutely in the absence of other complaints. A recent URI, not gastroenteritis, is most consistent with a transient synovitis. D. Correct. The patient likely had a recent case of mild to moderate gastroenteritis in Mexico, which may have been secondary to an bacterial enteritis such as shigella, or campylobacter. In reactive arthritis, joint inflammation occurs a few weeks later because antibodies made during the illness are attacking the joint. While several inflammatory cells would be seen in the aspirate, importantly, the cultures will turn out to be negative. E. Although on exam it may be difficult to distinguish septic from reactive arthritis, in septic arthritis the culture is often positive. These patients usually have acute onset of pain, fever, and limp, and the hip or knee is commonly affected. It is caused by an infection, typically bacterial, in the joint space. While definitely high on the differential for this patient, it is not the best answer choice given the history of diarrhea and the benign finding on joint aspiration.

A 2-year-old female with normal birth and developmental history presents with increased agitation and decreased arousability. Her father suffers from chronic pain secondary to a back injury, and her mother found an open container of pills on the bed. Vitals reflect bradycardia, bradypnea, hypotension, and slight hypothermia. On physical exam, she exhibits somnolence, constricted pupils, hypoactive bowel sounds, and hyporeflexia. What substance was most likely ingested? A. Iron B. Amitriptyline C. Insecticides D. Hydromorphone

> D has been selected by the expert. A. Iron is incorrect. Iron toxicity tends to cause severe abdominal symptoms followed by signs of shock. The child would be at risk for coagulopathies, gastrointestinal hemorrhage, and/or metabolic acidosis. B. Incorrect. Amitriptyline is an anticholinergic medication that belongs to the tricyclic antidepressant family. Anticholinergic toxicity can cause fever, dry, and flushed skin, urinary retention, hypertension, tachycardia, mydriasis, and/or decreased gastric motility. C. Incorrect. Ingestion of insecticides, which typically are organophosphates, can induce increased lacrimation, salivation, sweating, urination, bronchorrhea, bronchospasm, muscle twitching, muscle weakness, bradycardia, miosis and blurred vision, and/or increased gastric motility. D. Hydromorphone is correct. Opioids such as hydromorphone can cause respiratory depression, bradycardia, hypotension, hypothermia, constipation, nausea, vomiting, sedation, confusion, and/or miosis.

A 6-year-old boy presents to the ED with three days of diffuse muscle aches and occasional chills. Today, he had a headache and abdominal pain. He reports that he does not feel hungry because he feels sick to his stomach. He denies recent cough, congestion, sore throat, joint pains, or sick contacts. His vitals are: T 101.3 F, BP 108/71 mmHg, P 110 bpm, R 28 bpm, O2 sat 100% on RA. On physical exam, you notice blanching, erythematous macules on his ankles and several petechiae on his wrists. Upon questioning, his mother says that the spots on his wrists previously looked like the spots on his ankles. His neck is supple and there is no hepatosplenomegaly or lymphadenopathy. He reports no sick contacts, but recently visited his cousins in North Carolina. What is the best next step in management? A. Give acetaminophen, obtain a Monospot, write a note for activity restriction, and advise his mother to bring him back if he is unable to tolerate fluids B. Perform skin scraping of macules and examine under microscope with KOH prep C. Admit the patient, obtain CBC, blood and CSF cultures, and await culture results to guide antibiotic therapy D. Admit the patient, obtain CBC, blood and CSF cultures, then give loading doses of doxycycline 2.2 mg/kg and ceftriaxone 100 mg/kg/day E. Give acetaminophen and obtain CBC, UA, and BUN/Cr

> D has been selected by the expert. A. This choice is incorrect. Although infectious mononucleosis is a possibility, this presentation is concerning for Rocky Mountain Spotted Fever (RMSF). Of note is the absence of sore throat, lymphadenopathy, and hepatosplenomegaly, which suggest something other than mononucleosis. B. This choice is incorrect. Scabies or a fungal infection such as tinea versicolor would not explain his fever, headache, abdominal pain, petechiae, and myalgias. For scabies, his chief complaint might be intense itching, and he might have an exposure history. C. This choice is incorrect. Given the history of fever, headache, and petechial rash, you cannot yet rule out meningitis and should give empiric antibiotics immediately. Then, once you have established a diagnosis of RMSF, treatment with doxycycline is indicated. D. This choice is correct. Given the patient's abdominal pain, headaches, myalgias, fever, and nausea, followed by blanching erythematous macules, which may be transitioning to petechiae and purpura, this presentation is classic for RMSF. His recent travel to North Carolina also fits with the geographical distribution of RMSF. The treatment of choice is doxycycline. N. meningiditis coverage with ceftriaxone is also necessary given his rash, headaches, and fevers. E. This choice is incorrect. Although Henoch-Schonlein Purpura can present with abdominal pain and a rash on the lower extremities, it usually begins with petechiae and purpura rather than a blanching rash. Arthralgias are also absent.

A 5-year-old girl comes into your office for a well-child visit. The mother says that child is overall very healthy, but she highlights "occasional colds" and recently more frequent temper tantrums. She does well in preschool, is toilet trained, and enjoys eating mostly pasta, bread, and milk. She lives with her mother and father in a home built in 1985. Lab studies were significant for a mild anemia with a hemoglobin of 10.0 g/dL. You note that her hemoglobin was in the normal range at her 3-year-old visit. Which of the following is the most likely cause of her anemia? A. Chronic blood loss B. Lead poisoning C. Chronic illness D. Iron deficiency E. Hemoglobinopathy

> D has been selected by the expert. A. This choice is incorrect. There was no complaint of melena, and the child is overall healthy. Chronic blood loss would therefore be unlikely. B. This choice is incorrect. Risk of lead poisoning is increased in patients who live in homes built before the 1950s. Additionally, this patient does not complain of other symptoms suggestive of lead poisoning: weight loss, lethargy, vomiting, and learning difficulties. C. This choice is incorrect. The patient has no significant past medical history, and the review of systems is unremarkable. Chronic illness is unlikely. D. This choice is correct. Given the patient's age and preference for pasta and milk, the most likely cause of anemia would be iron deficiency. Treatment would include oral iron supplementation and increased dietary iron intake. E. This choice is incorrect. If she had a hemoglobinopathy, she would have been expected to have been anemic at her previous visit as well.

An 8-year-old boy presents with tea-colored urine, oliguria, joint pain, hypertension, and generalized edema. One month ago, he presented with fever and sore throat, headache, abdominal pain, strawberry tongue, papular sandpaper rash. At that time, he did not have runny nose, congestion or cough. He was treated with appropriate antibiotics for his symptoms and made a full recovery. Which of the following diagnostic findings supports his new symptoms? A. ANA positive B. Low anti-streptolysin O titer C. Normal C3 complement D. Low C3 complement E. Gram negative organisms on urine culture

> D has been selected by the expert. A. This is incorrect because ANA is used to screen for lupus glomerulonephritis and will be elevated in lupus, but not in post-streptococcal glomerulonephritis (PSGN). B. This is incorrect because PSGN is associated with high ASO titers, not low titers. This is because the recent strep infection will result in a high level of antibodies against strep antigens, of which ASO is one. C. This is incorrect because PSGN is associated with low C3. Normal C3 is found in IgA nephropathy. D. Correct. C3 is decreased in PSGN. This value will return to normal six to eight weeks after presentation of PSGN symptoms. Although this patient was treated appropriately for his strep throat, treatment does not prevent PSGN. However, timely and appropriate treatment of strep throat will prevent development of rheumatic fever. More details for those really curious: C3 plays a role in bacterial killing. A low C3 > less bacterial killing > more prone to infection. A low C3 is also associated with increased risk for autoimmune diseases. E. This is incorrect. Gram negative organisms on urine culture is associated with UTI, not PSGN.

On your first day rotating in the pediatric clinics, you are assigned to see a patient who is 9 weeks old and was brought into clinic by his worried mother. She states that her son has not gained weight since they left the hospital. His weight is < 5th percentile, and height and head circumference are at the 25th percentile. His mother says he drinks two ounces of milk every two to three hours, and suckling is strong without any spitting up during feeds. He poops more than 10 times a day, but it appears greasy and foul smelling. He had an unremarkable birth history and a normal newborn screen. Cardiac, pulmonary, abdominal, and neurologic exams are all normal. His mother mentions her cousin had trouble gaining weight and would get frequent "lung infections." Which of the following is the best next step in management? A. Increase formula to a higher calorie mixture B. Stool culture and Wright stain C. Swallow study D. Sweat chloride test E. Echocardiogram

> D has been selected by the expert. A. Two ounces of milk every two to three hours is adequate intake. This child may need a higher calorie formula in the future, but not before a diagnosis has been made, as there is likely an organic cause of failure to thrive. B. Stool culture with Wright stain for WBC's could be useful in a patient with diarrhea if you suspect an infectious cause of the diarrhea. This patient however, presents with no fever and no blood in the stool, and a good appetite, all of which are less consistent with an infectious etiology. C. A swallow study would be used to evaluate this child's ability to drink fluids, but since he has not been spitting up and appears to take in milk without difficulty, this test would be less useful at this time. D. The patient's greasy and foul-smelling stool is characteristic of steatorrhea, or fat in the stool. This occurs in patients with cystic fibrosis due to their impaired pancreatic exocrine function. Given the stool history in the face of failure to thrive, a sweat chloride test would be indicated at tis time. Of note, the family history of poor weight gain in a cousin with frequent lung infections also suggests the possibility of cystic fibrosis-related bronchiectasis. Cystic fibrosis should still be suspected in the child who has a normal newborn screen. States differ in the type of test offered to screen for cystic fibrosis, and although great strides have been made in newborn detection, methods are not 100% sensitive. E. An echocardiogram may be useful if you suspect congestive heart failure as the cause of failure to thrive. In this case, however, the cardiac exam was normal and there were no other signs of heart failure, such as tachypnea, tachycardia, hepatomegaly, lower extremity edema, or crackles on lung exam.

A 5-month-old male presents to urgent care with his mother who states that she witnessed her son stop breathing and turn blue for about 25 seconds. Upon physical stimulation by the mother, the patient began to breathe again. This is the first time she has ever witnessed this happening. The patient's birth and past medical history are unremarkable. Physical exam is unremarkable, vital signs are stable and normal, and lab studies are all within normal limits. Which of the following is LEAST likely to be on the differential diagnosis as a cause for this patient's ALTE (apparent life threatening event)? A. Seizures B. Arrhythmia C. Infection D. Gastroesophageal reflux E. Congenital heart disease

> E has been selected by the expert. A. Choice A is incorrect because seizures are a relatively common cause of ALTE. Two case series found seizures to be the underlying cause of an ALTE in 15 to 20% of infants. In a patient with a suspected seizure, the patient may have displayed some or all of the following during symptoms: foaming at the mouth, loss of bowel or bladder incontinence, abnormal posturing, abnormal ocular eye movements, and/or biting of the tongue. However, some seizures may not display any of these symptoms. An EEG may be useful in the evaluation of seizures in a patient presenting with an ALTE. The causes of seizures are vast and include the following: intracranial bleeds, CNS infections, metabolic abnormalities, electrolyte abnormalities, fevers, genetic syndromes and structural abnormalities. B. Choice B is incorrect because up to 5% of ALTEs have been shown to be caused by cardiac arrhythmias. Particular arrhythmias may include prolonged QT syndrome, Wolff-Parkinson-White, or bradycardia secondary to congenital heart block. In patients born to mothers with lupus, the suspicion for congenital heart block as a cause of the ALTE should be high. An EKG would be very useful in evaluation of this patient's ALTE. C. Choice C is incorrect because infectious etiologies are a common cause of ALTE, particularly infection with RSV or B. pertussis. A viral respiratory tract infection can also be associated with increased risk for reoccurrence of an ALTE. Patients with infections or sepsis may present with apnea, pallor, tachycardia, fever or hypothermia. They often have poor oral intake and their muscular tone is frequently diminished. In children less than 1 month of age, common etiologies of infection include Group B strep, E. coli, Streptococcus pneumoniae, listeria, and HSV. A CBC, respiratory virus PCR panel, CXR, and possible blood cultures would be helpful in evaluation of this patient's ALTE. D. Choice D is incorrect because gastroesophageal reflux is diagnosed in up to 30% of patient's presenting with an ALTE. However, there has never actually been a demonstrated link between reflux and apnea. It is thought that reflux triggers laryngospasm, but treatment of the reflux does not necessarily prevent reoccurrence of the ALTE. Reflux as a cause of ALTE is more likely when the infant has gross emesis or regurgitation during the ALTE. If this patient does have gastroesophageal reflux, he may benefit from a therapeutic trial of H2 blocker. He may also benefit from instituting reflux precautions after meals such as leaving the patient upright for 15 to 30 minutes after eating, making sure the baby burps after eating, and tilting the crib up to 30 degrees in the downward direction when the baby sleeps. E. Choice E is correct because it is unlikely for a patient with congenital heart disease to first present with an ALTE. This patient has no past medical history and his birth history was unremarkable. Typically the patient will suffer from acute decompensation within the first few weeks of life. These patients will also have growth problems, difficulty with feeding and a murmur is often appreciated on physical exam. Children with undiagnosed Tetraology of Fallot may have intermittent episodes of cyanosis while blood supply is diverted from the pulmonary vasculature.

A 2-year-old girl is examined as an outpatient. While waiting for the pediatrician, her mother reads her a short book. When they are done, her mother asks her to take the book and return it to a bookshelf in the room. The child is not able to hold a pencil and cannot write her name. She can kick and throw a ball, but cannot jump in place. Which of the following best describes this child's development? A. Delayed language B. Delayed social skills C. Advanced fine motor skills D. Advanced gross motor skills E. Age-appropriate development

> E has been selected by the expert. A. Delayed language is incorrect: A 24-month-old child is expected to use pronouns inappropriately, but should be able to follow two-step commands such as taking a book and returning it to a location in the room. B. Delayed social skills is incorrect: At 24 months of age, children are able not only to listen to short stories, they also engage in parallel play. C. Advanced fine motor skills is incorrect: While a child can hold a pencil at 24 months, the grip is immature and the child imitates pencil strokes. Children can remove their pants and socks at this age, but need help to undress completely. D. Advanced gross motor is incorrect: Being able to jump in place is a 30-month-old milestone. Being able to throw a ball overhand is expected at 24 months of age. E. The child in this vignette is developmentally appropriate for her age.

Alex is a 6-year-old boy who presents to the clinic with a chief complaint of acute onset of bruising. He is afebrile, and his mother reports that he recently had a URI. He was born at full-term and has never been hospitalized. He was circumcised at birth with no problems with bleeding. No one in his family has any chronic medical problems. There have been no serious childhood illnesses or deaths. No one has a history of easy bruising or bleeding. On exam you find that he has a purpuric rash on his buttocks and legs. His urinalysis reveals 15 to 20 RBCs/hpf. Which of the following additional findings would NOT be consistent with the likely diagnosis? A. Elevated serum IgA B. Blood in the stool C. Colicky abdominal pain D. Pain in his knees and ankles E. Low platelets

> E has been selected by the expert. A. HSP is an IgA-mediated vasculitis involving the skin, joints, GI tract, and kidneys. Serum IgA may be elevated, occurring in 50% of patients. B. Since IgA immune complexes in HSP also attack the GI vessels, around 67% of patients have GI bleeding, either occult blood or grossly bloody stool. C. Abdominal pain occurs in 50 to 75% of HSP patients, as a result of the immune attack on the GI vessels, as described above. D. Arthralgias, mainly of the knees and ankles, are seen in about 75% of children with HSP. E. Thrombocytopenia is not characteristic of HSP, but is commonly seen in idiopathic thrombocytopenic purpura (ITP). Decreased platelets are often the delineating finding between HSP and ITP.

A 4-week-old female infant presents to clinic for a well child check. This infant had an uneventful delivery by NSVD at full term and subsequent normal neonatal screen. The nurse reports that her growth is a concern, with weight at 3.0 kg (< 3rd percentile) and weight for height at < 3rd percentile. Mom denies any drinking or drugs since before this pregnancy and says she has been breastfeeding every two to three hours and supplementing with appropriately mixed formula one to two times a day. She does report the baby seems to have issues latching and some possible gasping between suckles. There has been no diarrhea, hematochezia, vomiting, or fevers. The vital signs and exam (apart from a thin infant) are normal. The mother's affect is flat, and she seems anxious when you ask her about her infant. What is the most likely diagnosis for this infant's failure to thrive? A. Malabsorption B. Gastroenteritis C. Milk protein allergy D. Congestive heart failure E. Failure to thrive due to inadequate caloric intake

> E has been selected by the expert. A. Malabsorption typically presents with poor weight gain and loose stools despite good caloric intake. This infant has poor caloric intake with constipation, making this diagnosis unlikely. B. Gastroenteritis typically presents with loose stools, emesis, and fever. This infant has none of these symptoms. C. Milk protein allergy typically causes bloody stools rather than isolated poor weight gain. The patient does not have bloody stools or other signs of allergy or formula intolerance. Milk protein allergy is less common but still occurs in breastfed infants. D. Congestive heart failure is important to consider in any child with failure to thrive, but this child has no signs of CHF such as tiring with feeds, sweating, or tachypnea, tachycardia, or hepatomegaly. E. This infant is likely not consuming adequate calories to grow. The mother or primary caregiver may neglect proper feeding of the infant because of preoccupation with the demands or care of others, her own emotional problems, substance abuse, lack of knowledge about proper feeding, or lack of understanding of the infant's needs. It is also important to assess the mother for post-partum depression. Standardized screening tools, such as the Edinburgh, are used routinely in pediatric office settings for this purpose.

An 8-year-old boy is brought to clinic by his parents because they are concerned that he has not been doing his homework. His teacher recently called the parents to say that their son seems distracted in class, constantly interrupts other children when they are speaking, and is very fidgety. When you speak with the boy, he tells you that he did not know about the homework assignments and that he tries hard to pay attention in class. What is the next best step in management? CORRECT A. Prescribe a stimulant medication for ADHD B. Suggest behavior modification for the child and parenting classes C. Group therapy for the child D. Do nothing, as this child's behavior is normal E. Contact the teacher to find out more about his behavior. Find out more about the child's behavior at home

> E has been selected by the expert. A. Pharmacotherapy is often used in combination with behavioral modification/group therapy for children diagnosed with ADHD. However, to be diagnosed with ADHD, one must have 6 or more symptoms in 2 or more settings for at least 6 months, and several of these symptoms must be present before the age of 12. The symptoms fall within the three categories of inattention, hyperactivity, and impulsivity. B. Behavior modification for the child and parenting classes for the parents are both used as treatment modalities in ADHD. Parents should be counseled on positive reinforcement, firm non-punitive limit setting, and how to reduce external stimuli. However, this child first requires further evaluation. C. Children with ADHD often learn best in group therapy, particularly social skills and self-esteem. Again, this child first requires further evaluation. D. Many school-aged children are easily distractible, impatient, and hyperactive. It is important to distinguish those who truly have ADHD from those who do not. Again, to have a diagnosis of ADHD, symptoms must be present in two or more settings. E. Contacting the teacher to find out more about the child's behavior at school and learning more about his behavior at home are the best ways to determine if 6 of the symptoms are present in 2 or more settings, which is required to make the diagnosis of ADHD. It also will be important to learn more about other aspects of this child's life, as there are several factors that can lead to acting out (including learning disability, hearing disability, family stress, and abuse).

Brian, a 5-year-old boy with swelling around both his eyes and an abdomen that looks "bigger than normal," is brought in by his mother to your preceptor's office. Mom explains that she noticed the puffy eyes and bigger belly starting the week before. It seemed to appear out of nowhere, and Brian has been completely healthy except he had a cold several days before these symptoms developed. When you ask Brian if he has noticed anything else weird, he says that his "pee-pee looks like Coca-cola," at which point his mom scowls and tells him to stop being silly. His blood pressure taken by the nurse right before entering the room is elevated. Based on the above information, which of the following does Brian likely have? A. Periorbital cellulitis B. Viral upper respiratory infection C. Allergic conjunctivitis D. Nephrotic syndrome E. Acute glomerulonephritis

> E has been selected by the expert. A. This choice is incorrect. Periorbital cellulitis is usually unilateral, and it would involve a history of insect bite, trauma, or bacterial infection such as sinusitis. Additionally, periorbital cellulitis, would not explain the larger-appearing abdomen or the tea-colored urine, all suggestive of acute glomerulonephritis. B. This choice is incorrect. Although URI could explain some of the bilateral periorbital edema and the "cold" preceding these current symptoms, URI in itself would not explain the distended abdomen or the darkened urine. C. This choice is incorrect. Allergic conjunctivitis would present with conjunctival injection in addition to the periorbital edema. Additionally this diagnosis would not explain the distended abdomen or darkened urine. D. This choice is incorrect. While nephrotic syndrome could explain the periorbital edema and ascites, it would not explain the darkened urine. Also nephrotic syndrome is more associated with normal blood pressure and Brian's blood pressure is elevated. E. This choice is correct. Acute glomerulonephritis would explain all aspects of Brian's presentation. The disease presents with gross hematuria (which most patients describe as tea-colored or cola-colored), periorbital swelling and ascites due to hypoalbuminemia, and hypertension due to intravascular fluid overload. Additionally, acute glomerulonephritis frequently follows a URI.

A 15-year-old female comes to the clinic with a chief complaint of feeling tired for one month. She has also been complaining of frequent nosebleeds while at school and bruising easily. When further history is elicited, you find out that menarche was at the age of 9 and her periods have always been heavy and irregular. Her mother and grandmother also have histories of heavy periods and easy bruising. You suspect a bleeding disorder and send off some labs including a CBC, INR, PT, PTT, and a von Willebrand panel to confirm your diagnosis. Your suspicion was correct for the most common type of bleeding disorder. How is this bleeding disorder most commonly inherited? A. Autosomal dominant inheritance B. Autosomal recessive inheritance C. X-linked recessive inheritance D. Mitochondrial inheritance

> A has been selected by the expert. A. Autosomal dominant (AD) inheritance is the correct choice. In AD disorders males and females are equally affected within each generation. These include conditions such as von Willebrand's disease, Marfan syndrome, neurofibromatosis, and Huntington's disease. B. Autosomal recessive (AR) is incorrect. In AR diseases female and male offspring of heterozygote carriers have a ¼ chance of being affected. These include Tay-Sachs disease and cystic fibrosis. C. X-linked recessive (XLR) is incorrect. In XLR disorders males are more commonly affected, while females are the carriers and pass the disorder on to their sons. Males cannot pass the disease on to other males. These include conditions such as Duchenne's muscular dystrophy, hemophilia, and Fragile X syndrome. D. Mitochondrial inheritance is incorrect. In mitochondrial disorders the disease is inherited only from the mother, and usually all children are affected. Affected males cannot pass the disorder on to their offspring. These include mitochondrial myopathies such as Leber's hereditary optic neuropathy and mitochondrial encephalopathy.

A 5-year-old African-American male with sickle cell disease presents to clinic with a chief complaint of severe chest pain for the past day. His mother notes that he has been breathing quickly and that she measured his temperature this morning to be 100.5 F. Patient describes pain as an 8/10 on the faces scale. Patient is tachypneic on exam and has an oxygen saturation of 97% on room air. Chest exam reveals normal lung sounds bilaterally, and he has some reproducible tenderness over his chest wall. A chest x-ray is performed and demonstrates clear lung fields and a cardiac silhouette that is within normal limits. What is the most likely cause of the chest pain? A. Acute chest syndrome B. Rib infarction C. Sepsis D. CHF E. Pneumothorax

> B has been selected by the expert. A. Acute chest syndrome (ACS) is incorrect, because there is normal oxygen saturation, and normal lung fields on radiological investigation. Lung markings on chest x-ray suggestive of ACS would include pulmonary infiltrates, atelectasis, and effusion. Causes of ACS include infection, pulmonary fat embolism, or intrapulmonary sickling. Typical therapy may involve supplemental oxygen, pain management, hydration, antibiotics and possibly transfusion. B. Rib infarction is correct because of the chest pain in the setting of a history of sickle cell disease with normal oxygen saturation and clear lung fields on chest x-ray. Osteomyelitis and painful vaso-occlusive crises represent the most common reasons for admission for sickle cell disease patients. Rib infarction may lead to a picture similar to ACS as the pain can lead to hypoventilation, which may result in atelectasis and the characteristic radiographic findings of ACS. However, no radiographic findings are present in this patient. As a result, rib infarction is the most likely etiology. C. Sepsis is incorrect because it would not typically present with isolated chest pain. Sepsis may often happen with sickle cell disease patients, given their susceptibility to infection. If cardiogenic shock follows, cardiomegaly may be appreciated. New infiltrates may occur if ARDS results secondarily from sepsis. However neither cardiomegaly nor new infiltrates were appreciated in this patient. While neither is required for the diagnosis of sepsis, the presence of fever may be the only indication of sepsis in a child with sickle cell disease. As a result, while less likely, sepsis is critical to include in the differential for this patient and to treat empirically until ruled out. D. CHF is incorrect because, like sepsis, it would not typically cause chest pain on its own. Additionally, abnormal lung markings and/or cardiomegaly would be expected in a patient with CHF. Neither is present in the patient in this vignette. E. Pneumothorax is incorrect because, other than rate, no abnormalities were noted on respiratory examination, and there were no supporting findings on chest x-ray. While a pneumothorax may cause chest pain, typically described as sharp and worsening with inspiration, this child's physical exam and radiographic findings make this option much less likely.

You are called down to the nursery to evaluate a newborn girl who is ready to be discharged. The mom is concerned because this 3-day-old has become lethargic and doesn't want to feed. She has vomited twice and is showing no interest in feeding. On physical exam you note a lethargic infant with an enlarged liver and worry about an inborn error of metabolism. Which test would be diagnostic for an ornithine transcarbamylase (OTC) deficiency? A. Hypoglycemia B. Hyperammonemia and elevated urine orotic acid C. Elevated 17-OH progesterone D. Elevated TSH E. Hyponatremia

> B has been selected by the expert. A. Although infants with an inborn error of metabolism may have hypoglycemia, it is not diagnostic of OTC. B. Both hyperammonemia and elevated urine orotic acid are diagnostic of OTC deficiency, an x-linked condition, the most common urea cycle disorder. C. Elevated 17-OH progesterone would be expected and diagnostic of a patient with congenital adrenal hyperplasia (CAH) and would likely be associated with virilization in a female infant. D. Elevated TSH is diagnostic of congenital hypothyroism, not OTC. E. Infants with congenital adrenal hyperplasia may have low sodium, but not patients with OTC deficiency.

You are on the nursery service when your team is called to evaluate a 1-day-old infant. The infant was born via NSVD at 40 weeks' gestation to a 38-year-old G1P1A0 mother who did not have access to prenatal care and did not receive prenatal testing. The infant weighed 7 lbs 12 oz at birth and had Apgar scores of 7 and 8. On exam the infant is sleeping comfortably. She is afebrile with normal vital signs but appears to have low tone on exam. You also notice her ears seem to be lower than her eyes and appreciate mild edema of the hands and feet. Additionally, you note a fluid-filled sac at the base of the neck that does not appear to interfere with breathing. A karyotype performed after birth reveals a chromosomal abnormality. Which of the following is the most likely cause of this patient's condition? A. Down syndrome B. Turner syndrome C. Fetal alcohol syndrome D. Benign neonatal hypotonia E. Cystic hygroma

> B has been selected by the expert. A. Down syndrome is incorrect, because infants with Down syndrome typically have physical findings that include epicanthic folds, flat facial profile, single palmar creases, redundant neck skin, and heart defects. Other findings including a gap between the first and second toes. While infants with Down syndrome can be hypotonic as this infant is, the patient in this scenario does not have the other physical findings consistent with Down syndrome. B. Turner syndrome is correct. Turner syndrome is defined by the karyotype 45 XO. Characteristics of females with Turner syndrome include renal abnormalities, lymphedema (causing edema of hands and feet), low-set ears, congenital heart defects, dental abnormalities-such as narrow or high-arched palates-and cystic hygromas. Other possible physical findings include a webbed neck, widely-spaced nipples, and shield-like chest. As this mother did not have prenatal testing or a karyotype performed previous to her daughter's birth, one should be performed to confirm the diagnosis. C. Fetal alcohol syndrome (FAS) is incorrect. While FAS can cause hypotonia, this infant does not demonstrate the smooth philtrum, thin upper lip, or small palpebral fissures commonly seen in infants with FAS. FAS would not demonstrate a chromosomal abnormality on karyotyping. D. Benign neonatal hypotonia is incorrect because the infant demonstrates facial features that suggest an alternate etiology as well as abnormal karyotype. Typically, hypotonia of the newborn, especially at one day after birth, is abnormal and could reflect illness (such as sepsis), or neurologic dysfunction at either the central or peripheral levels. E. A cystic hygoma can present with a fluid-filled collection around the neck but is not associated with edema elsewhere and would not be associated with a chromosomal abnormality.

A newborn baby boy is born at 30 5/7 weeks' gestation after induction of labor for the severe maternal preeclampsia. He is noted to have subcostal and intercostal retractions, grunting, nasal flaring, persistent cyanosis, and tachypnea 30 minutes after delivery. Apgars were 6 (-2 for color, -1 for breathing and -1 for tone) and 7 (-2 for color and -1 for breathing) at 1 and 5 minutes, respectively. Due to lack of prenatal care and the mother's presentation with severe preeclampsia, betamethasone x 1 was given during induction, but she did not receive a second dose prior to delivery. A chest x-ray is obtained, which reveals diffuse ground-glass appearance and air bronchograms bilaterally. What is the most likely diagnosis? A. Meconium aspiration syndrome (MAS) B. Respiratory distress syndrome (RDS) C. Persistent pulmonary hypertension (PPHN) D. Transient tachypnea of the newborn (TTN) E. Bronchopulmonary dysplasia (BPD)

> B has been selected by the expert. A. This choice is incorrect. Although the presence or absence of meconium was not noted in the case, it is known that from 20 to 34 weeks' gestation, the fetus will pass meconium infrequently. Most cases of MAS are in term or post-term infants. On chest x-ray, we might see overdistention of the lung or other sequelae, such as pneumothorax. B. This choice is correct. The baby boy is preterm, and his mother received only one dose of betamethasone, which puts him at increased risk for developing infant RDS, which is caused by insufficient surfactant. His physical exam and chest x-ray findings are consistent with RDS. C. This choice is incorrect. PPHN generally occurs in babies born after 34 weeks. There are several causes for PPHN: underdevelopment, maldevelopment, and maladaptation. Underdevelopment of the lungs can be secondary to congenital diaphragmatic hernia, oligohydramnios in utero, IUGR, or renal agenesis. The underdevelopment causes increased pulmonary vascular resistance and has a poor prognosis. Maldevelopment involves remodeling of pulmonary vasculature and is associated with post-term delivery and meconium aspiration syndrome. Maladaptation can be caused by infection with GBS. Vasoactive mediators are activated by bacterial phospholipids, causing an increase in pulmonary vascular resistance. D. This choice is incorrect. TTN is a disorder of delayed reabsorption of fluid in the newborn's lungs. Prematurity, delivery by C-section, being large or small for gestational age, or having a diabetic mother are all risks. In order to be diagnosed with TTN, the baby would need to show improvement within several hours. Although this is on the differential for the newborn baby's condition based on clinical presentation, a chest x-ray should have shown perihilar streaking and other evidence of interstitial fluid. E. This choice is incorrect, because BPD is the result of prolonged mechanical ventilation. Our patient is at risk for developing this syndrome if he requires intubation. Chest x-ray may show atelectasis, inflammation, or pulmonary edema. With severe disease, the chest x-ray may reveal fibrosis and hyperinflation.

A 9-year-old female is brought to clinic by her mother because of two days of abdominal pain and vomiting. She has vomited six times today and has had decreased appetite, but no diarrhea, fevers, sick contacts, or changes in diet. Her mom states that she has been otherwise healthy apart from increased thirst and occasional bedwetting over the last few weeks. Of note, patient's maternal grandmother suffers from celiac disease. On exam, patient is afebrile and has a HR of 180 bpm, BP 90/60 mmHg, RR 50 bpm, and O2 saturation of 98%. She is lying in bed appearing slightly drowsy, taking rapid, deep breaths and is slow to respond to questions. Her heart and lung exams are normal apart from being tachycardic, and abdominal exam reveals mild diffuse tenderness to palpation with no rebound or guarding. Which of the following would be the most appropriate next step in management? A. Chest x-ray B. Urine culture C. Fingerstick glucose D. Abdominal ultrasound E. Gastric lavage

> C has been selected by the expert. A. A chest x-ray would be appropriate if bacterial pneumonia were high on your differential. A patient with pneumonia generally presents with fever, cough, tachypnea, and will likely have characteristic lung findings such as crackles on exam. The patient may experience abdominal pain secondary to pleural inflammation; however, vomiting is not a common presentation. The absence of fever, cough, sick contacts, and lung findings on exam make pneumonia a less likely diagnosis. B. Urine culture would be appropriate if pyelonephritis were high on your differential. A patient with pyelonephritis may present with history of fever, dysuria, urinary frequency, CVA tenderness, and vomiting. However, this patient's overall clinical picture does not support the diagnosis, since the patient is afebrile without a history of dysuria or classic CVA tenderness. C. Obtaining a fingerstick glucose is the diagnostic step with the highest yield since the patient's clinical picture is strongly indicative of diabetic ketoacidosis (DKA). DKA is a condition more closely associated with type 1 (rather than type 2) diabetes, and is formally diagnosed if a random glucose is > 200 mg/dL, venous pH is < 7.3 and/or bicarbonate < 15 mEq/L and there is ketonemia or ketonuria. Patients in DKA can present with abdominal pain and vomiting secondary to metabolic acidosis that stems from ketonemia and lactic acidosis. Furthermore, osmotic diuresis from hyperglycemia may contribute to dehydration, which can manifest as tachycardia, hypotension, and altered mental status. In an attempt to compensate for the metabolic acidosis, the patient may also present as tachypneic with characteristic Kussmaul respirations (rapid, deep breaths). This patient's history of polydipsia, enuresis, and family history of autoimmune disease (including celiac disease and Hashimoto's thyroiditis) suggest that the patient has type 1 diabetes. Her current vital signs and general state of lethargy also point towards DKA and should be confirmed with a fingerstick glucose (in addition to other tests). D. Abdominal ultrasound would be indicated if appendicitis were high on your differential. Appendicitis may present with fever, nausea, vomiting, diarrhea, decreased appetite, and abdominal pain that localizes to the RLQ. Physical exam findings may include a positive Rovsing's sign or a positive psoas or obturator sign. Ultrasound may be a useful tool in detecting appendicitis in children. Appendicitis is less likely in this patient since she is afebrile and does not have the classic localizing pain characteristic of this condition. E. Gastric lavage may be the indicated next step of management if toxic ingestion is the confirmed diagnosis. A patient who has ingested toxic substance is usually afebrile and obtunded and may present with vomiting and manifestations of dehydration (secondary to vomiting). In a patient with salicylate toxicity, tachypnea is a common presentation. A thorough history to assess exposure to toxins and sending urine for evaluation of toxins may be appropriate to make this diagnosis. Given this patient's overall picture, DKA is a more likely diagnosis.

A 9-month old baby boy comes to the clinic for a well child visit. The child is at the 50th percentile for weight, length, and head circumference. He is reaching all developmental milestones appropriately. The mother has no concerns at this visit. The child has previously received the following vaccines: 3 doses of DTaP, 3 doses of Hib, 2 doses of HepB, 3 doses of RotaV, 2 doses of IPV and 3 doses of PCV13, and no influenza vaccines. Which vaccines should the child receive at today's visit? A. Influenza, Hep B, IPV, DTaP B. Influenza, IPV C. Influenza, Hep B, IPV D. Hep B, DTaP, IPV E. Hep B, IPV, and MMR

> C has been selected by the expert. A. Influenza, Hep B, IPV, DTaP is incorrect. All three doses of DTaP have been given. B. Influenza, IPV is incorrect. The patient needs the third Hep B shot. C. Influenza, Hep B, IPV is correct. The patient needs a third Hep B, a third IPV, and a yearly flu shot starting at 6 months of age. D. Hep B, DTaP, IPV is incorrect. All three doses of DTaP have been given and the patient now needs a yearly flu shot starting at 6 months of age. E. Hep B, IPV, and MMR is incorrect. The patient also needs a yearly flu shot starting at 6 months of age and MMR is not given before 12 months of age.

An 18-month-old female is brought to her pediatrician by her mother who notes that she has been has been fussy for the past three days and has been pulling on her ears. The child is up to date with her hepatitis B, rotavirus, DTaP, H. influenza type B, pneumococcus, and polio vaccines. Her temperature is 102.2 F. Otoscopic exam of her left ear shows a yellow, opaque, and bulging tympanic membrane. Which of the following organisms is the most likely cause of the child's condition? A. Streptococcus pyogenes B. Candida albicans C. Haemophilus influenzae D. Rhinovirus E. Moraxella catarrhalis

> C has been selected by the expert. A. The child is suffering from acute otitis media (AOM). S. pyogenes is a rare cause of this condition (< 5% of cases). One should not confuse this species of strep with S. pneumoniae, which is a common cause of AOM (25-50% of cases). B. Candida albicans is not a frequent cause of AOM. An infection involving this organism in the middle ear should immediately raise suspicion for an immunocompromised state. C. H. influenzae is a frequent cause of AOM (15-52% of cases). Although the child has been vaccinated against H. influenzae type B, this does not cover the unencapsulated strains of H. influenzae that cause AOM. D. Rhinovirus is a potential cause of AOM, but bacterial AOM occurs with a much higher frequency. E. M. catarrhalis is another common cause of AOM. However, this organism is responsible for only between 3% and 20% of cases, making it a less frequent cause than non-typeable H. influenzae.

A 3-year old girl comes to the clinic with a chief complaint of fever (104F) for over a week. Her mom reports that she has been fussy and inconsolable since she became febrile. She has a red tongue, with large papillae, conjunctivitis, a palmar rash, unilateral cervical adenopathy, as well as swollen feet. Given the most likely diagnosis, what is the most important follow-up for this patient over the next few weeks? A. Ophthalmology follow-up to determine extent of eye damage and determine need for corticosteroids B. Physical therapy follow-up to help prevent long-term joint deformities and ensure long-term functionality C. Cardiology follow-up to rule out presence of rheumatic fever D. Echocardiogram to look for coronary artery aneurysm E. Neurology follow-up to evaluate partial paralysis of lower extremities

> D has been selected by the expert. A. Choice A is incorrect because the patient likely has Kawasaki disease. Early referral to an ophthalmologist is important for patients diagnosed with other conditions, including Stevens Johnson Syndrome (SJS) to determine the degree of eye involvement and if treatment with topical steroids is needed. SJS is a mucocutaneous disorder defined by fever, severe stomatitis (inflammation of the mucous lining of any of the structures in the mouth), conjunctivitis, and a blistering rash. It is typically caused precipitated by medications or infections. Early referral to an ophthalmologist would also be important for patients diagnosed with juvenile idiopathic arthritis, because they can suffer from uveitis, which if left untreated, can lead to long-term problems such as cataracts, glaucoma, or blindness. B. Choice B is incorrect because the patient in this case has Kawasaki disease. Early referral to physical therapy is important for patients diagnosed with systemic juvenile idiopathic arthritis, which is characterized by prolonged or spiking fever, rash, and arthritis. Early treatment and physical therapy can help prevent joint deformities and improve long-term functionality. C. Choice C is incorrect because the patient in this case has Kawasaki disease. Patients who have scarlet fever could develop rheumatic fever or post-streptococcal glomerulonephritis, among other problems. Scarlet fever is characterized by a "erythematous, blanching, sandpaper-like rash" with very fine papules secondary to infection with Group A streptococcus. It may start in the groin, axilla, or neck, before spreading rapidly over the trunk and extremities. Fever can be high, but generally resolves within five days. D. Choice D is correct because children with Kawasaki disease are at high risk for coronary artery aneurysm formation and should receive an echocardiogram within four weeks of the onset of their illness. Use of IVIG for the treatment of Kawasaki disease has decreased the risk of coronary artery aneurysms significantly. Kawasaki disease is diagnosed when there is a fever plus four of the following: changes in oral mucosa (e.g., strawberry tongue), extremity swelling or redness, unilateral cervical adenopathy, conjunctivitis, and rash. Infectious and rheumatologic causes must be excluded in order to make the diagnosis of Kawasaki disease. E. Choice E is incorrect because the patient in this case has Kawasaki disease. Neurologic issues such as paralysis of a lower limb can occur in severe cases of Rocky Mountain Spotted Fever, which is characterized by fever, myalgias, headache, and petechial rash classically starting on wrists and ankles and progressing centrally.

A 17-year-old boy presents for a sports pre-participation physical. He reports that he occasionally gets short of breath and feels light-headed with exercise, and sometimes he experiences chest pain as well. He lost consciousness once last season during a playoff basketball game, but attributed it to feeling sick at the time. His grandfather died suddenly at age 35 of unknown etiology. Which of the following is the most likely diagnosis? A. Hypoglycemia B. Congenital heart block C. Postural hypotension D. Prolonged QT syndrome E. Ventricular septal defect

> D has been selected by the expert. A. Hypoglycemia is very uncommon in healthy children. It is usually associated with diaphoresis, anxiety, tremulousness, and a feeling of hunger. Chest pain and shortness of breath are less likely. B. Congenital heart block can be caused by congenital heart defects and autoimmune disease in pregnant women (lupus). This condition is a rare cause of arrhythmia and is often diagnosed early in life. Many patients require a pacemaker. C. Postural hypotension is a very common cause of dizziness and visual changes in the pediatric population. It is usually triggered by volume depletion and skipping meals. Hypotension is not usually associated with shortness of breath, chest pain, and prolonged (> 5 min) loss of consciousness. D. Prolonged QT syndrome can cause syncopal episodes in late childhood or adolescence. QT intervals are elongated on ECG and lead to arrhythmias, like ventricular fibrillation. This condition is often associated with other abnormalities, including severe congenital sensorineural deafness. E. Ventricular septal defects often have negligible cardiovascular sequelae, or will close up on their own after a number of years. If they are not monitored, however, the shunting can cause hypertrophy of the right heart, called Eisenmeiger syndrome, which could result in pulmonary hypertension, exercise intolerance, and eventual heart failure. It would be unlikely, however, to cause chest pain or syncope. The family history of sudden cardiac death also points to prolonged QT rather than a VSD.

Joe, a previously healthy 11-month-old male with 5-day history of a "cold," is brought to the ED by mom for one day of acute worsening cough and intermittent wheezing. Per mom, the cough was initially dry but has become more "phlegmy," making it difficult for Joe to breathe, particularly when he is feeding or more active. His immunizations are up to date, and he has no known allergies. His family history is significant for a 6-year old sister who was diagnosed with asthma four years ago. On exam, Joe is afebrile, mildly tachypneic with normal O2 saturation. He has prominent nasal flaring and mild subcostal retractions. He has clear rhinorrhea but no evidence of oropharyngeal erythema. Lung exam reveals decreased breath sounds and wheezes on the right. What is the most likely diagnosis? A. RSV bronchiolitis B. Epiglottitis C. Viral URI D. Asthma E. Foreign body aspiration

> E has been selected by the expert. A. Bronchiolitis is a lower respiratory tract infection most commonly caused by RSV, which is characterized by bronchiolar obstruction secondary to mucus plugging, cellular debris, and edema. Patients generally present with fever and URI symptoms which progress to a worsening cough, wheezing and shortness of breath. Although this patient does have wheezing, the unilateral wheezing with decreased breath sounds is not consistent with bronchiolitis. B. Epiglottitis was commonly caused by Haemophilus influenzae type B, but can also be caused by Staph and Strep species. Patients may present with fever, dysphagia, drooling, stridor and significant respiratory distress. Patients are generally seen sitting, leaning forward with the neck hyperextended. Epiglottitis has become less common due to immunization with Hib. This diagnosis is less likely in our patient, since his immunizations are up to date, he is afebrile and not in severe respiratory distress. C. Our patient probably developed a viral URI five days ago. An upper respiratory tract infection in children can manifest as fever, rhinorrhea, cough, sore throat and myalgias, and may be accompanied by wheezing. However, our patient's ausculatation findings cannot be explained solely by a viral URI. D. Asthma is caused by inflammation of airway mucosa, mucus hypersecretion, mucosal edema and reversible bronchoconstriction. It generally presents as cough, wheezing, tachypnea and dyspnea worsened by cold air, exercise, allergies and URIs. The mainstay of treatment involves bronchodilators (beta-2 agonists) and inhaled steroids. Asthma is a possible diagnosis in Joe given the family history of asthma; however, it is less likely since he was previously healthy with no history of recurrent cough or wheezing. Furthermore, asthma does not generally present with focal wheezing as heard on Joe's lung exam. E. Given Joe's age, foreign body aspiration should always be included in the differential diagnosis for acute onset wheezing. The lung findings of asymmetric breath sounds and wheezing support this diagnosis. Foreign body in the airway can be confirmed by bilateral decubitus or inspiratory/expiratory chest films, characterized by decreased deflation on the affected side. If complete obstruction, x-ray will generally reveal atelectasis (whiting out) and signs of volume loss (mediastinal shift towards affected side to compensate for loss of volume).

A 3-week-old baby boy is brought to his pediatrician with a chief complaint of light tan-colored stools and worsening jaundice. His is exclusively breastfed and has 6-8 wet diapers per day. On exam, he appears to have scleral icterus and jaundice. Upon further workup, he is found to have an elevated direct bilirubin. What is his most likely diagnosis? A. Biliary atresia B. Breastfeeding jaundice C. G6PD deficiency D. Physiologic jaundice E. Caput succedaneum

> A has been selected by the expert. A. Biliary atresia can present anytime between birth and 8 weeks of age, but usually occurs after 2 weeks of age. Jaundice is usually the first presenting finding, along with acholic stools, dark urine (from increased bilirubin excretion) and hepatosplenomegaly if the problem goes unrecognized. Laboratory values classically show an increased level of direct or conjugated bilirubin > 2 mg/dL. If biliary atresia is confirmed with further laboratory testing and imaging, surgical intervention must be pursued as soon as possible. B. Breastfeeding jaundice normally occurs within the first week of life, most often because of decreased intake leading to dehydration and increased enterohepatic circulation. This patient is exclusively breastfed, but his jaundice began at approximately day 16 of life. Also, breastfeeding jaundice increases unconjugated bilirubin levels, making this answer choice less likely. C. G6PD is an X-linked inherited disorder. The severity of this disorder is dependent upon the degree of deficiency of the enzyme, but may present with neonatal unconjugated hyperbilirubinemia. However, laboratory findings show hemolytic anemia in the symptomatic state. Hemolysis is usually elicited by drugs, most notably primaquine and dapsone, as well as fava beans. D. Physiologic jaundice peaks at 3-4 days of life and generally resolves within a day or two. This patient is well beyond that age, making this answer less likely. E. Caput succedaneum is caused by an increase in serum above the periosteum of an infant that crosses suture lines (as opposed to cephalohematoma which does not cross suture lines). The increased serum bilirubin does not usually lead to significant hyperbilirubinemia.

A 4-year-old boy who recently emigrated from eastern Europe presents with his mother to your general pediatrics clinic. His mother reports that he has a chronic nonproductive cough during the day and night, mild wheezing for one month and failure to gain weight (his weight has dropped from the 50th to the 10th percentile for his age). His mother denies any high fevers, rhinorrhea, or night sweats. Which of the following are the next best diagnostic tests? A. Chest x-ray and tuberculin skin test B. CT of nasal sinuses C. Spirometry, before and after bronchodilator therapy D. Chest x-ray and methacholine challenge E. None needed, patient likely has habitual cough

> A has been selected by the expert. A. CXR and tuberculin skin test (TST) is the best choice. Signs and symptoms of primary pulmonary tuberculosis are few to none. Toddlers may present with nonproductive cough, mild dyspnea, wheezing, and/or failure to thrive (defined as weight < 5th percentile or drop in two percentile curves for weight). In children, TB can present without systemic complaints (fever, night sweats, and anorexia), severe cough, and sputum production. Regarding diagnostic tests, the TST is a practical tool for diagnosing TB infections. All children with chronic cough (more than three weeks) should be evaluated with a chest x-ray, as other pathology-such as lung abscess or malignancy-can also be detected on CXR. B. Sinusitis is often preceded by a URI, with nasal congestion as a prominent feature, leading to nocturnal cough due to post-nasal drip. These symptoms are not seen in our patient. Furthermore, a diagnosis of sinusitis is made clinically, with CT scan obtained only in complicated cases or cases resistant to treatment. Complications include cavernous sinus thrombosis, meningitis, and epidural abscess. C. Spirometry (pulmonary function testing) before and after bronchodilator therapy is the most specific means of determining whether or not a child has reactive airways. Asthma is a very common diagnosis in pediatrics, and may present with cough that is worse at night and exacerbated by exercise and cold air. Patients with cough-variant asthma present with only cough, typically nonproductive. However, given this patient's failure to thrive, a more serious diagnosis such as TB must be considered. Also, a chest x-ray is needed in all children with chronic cough (more than three weeks). D. Although a chest x-ray is appropriate in all children with chronic cough, a methacholine challenge (for asthma) would be inappropriate in this scenario. Although asthma is a common diagnosis, given the patient's failure to thrive, a more serious diagnosis must be considered. Further, a methacholine challenge is reserved for cases in which asthma is suspected and spirometry is normal or near normal, and should be performed by trained individuals. E. Habitual cough is caused by habitual perpetuation of a cough that begins with a viral URI. Continued coughing further irritates the airway, leading to stronger stimulation to cough. The cough is typically very loud, short, dry, brassy, and spasmodic. This cough is unchanged by exercise or cold air, and classically resolves during sleep. Although the patient in this case has a dry cough, his failure to thrive points to a more serious diagnosis (e.g., TB). All children with chronic cough (persisting longer than three weeks) need a CXR.

A 16-year-old female presents with acute onset of diffuse abdominal pain with periodic sharpness in the right upper quadrant that radiates to her back. She has had some episodes of vomiting and has a fever. She is sexually active and has used alcohol in the past. Which of the following is most likely to present with right upper quadrant pain? A. Pancreatitis B. Urinary tract infection C. Ectopic pregnancy D. Appendicitis E. Ovarian torsion

> A has been selected by the expert. A. Choice A is correct because pancreatitis commonly causes continuous abdominal pain that can localize to the right and left upper quadrants ("band-like pain") as well as radiating to the back. Nausea and vomiting are nearly always present. Lipase is the most sensitive and specific lab test to diagnose pancreatitis. B. Choice B is incorrect because urinary tract infections classically cause painful urination with frequency and urgency. Patients may present with suprapubic tenderness, and flank pain is common when the infection has spread to the kidney. UTIs are common in sexually active females, and fever and vomiting are often seen with pyelonephritis, but sharp right upper quadrant pain would not be expected. C. Choice C is incorrect because although ectopic pregnancy is always a concern in a sexually active female presenting with abdominal pain, the pain is typically located in the lower abdomen. Fever and diffuse abdominal pain are uncommon in uncomplicated ectopic pregnancy. D. Choice D is incorrect because appendicitis classically starts as periumbilical pain that migrates to McBurney's point (1-2" from the anterior superior iliac spine toward the umbilicus). It is also of concern in a teenager with a presentation like this, as acute appendicitis is the most common pediatric condition requiring emergency surgery. Of children presenting with acute abdominal pain, 1-4% have appendicitis. E. Choice E is incorrect. Although ovarian torsion is possible at any age, the abdominal pain is typically described as stabbing and is usually localized to the lower abdomen and pelvis. Nausea and vomiting can also be present.

You have accepted a part-time tutoring job for first-year medical students. One of your students asks if you would please clarify the details of normal fetal circulation. Which of the following best describes the path of the majority of oxygenated blood that enters the right atrium? A. RA > foramen ovale > LA > LV > systemic circulation B. RA > RV > VSD > LV > systemic circulation C. RA > RV > pulmonary circulation > LA > LV > systemic circulation D. RA > RV > ductus arteriosus > LV > systemic circulatio E. RA > RV > ductus arteriosus > systemic circulation

> A has been selected by the expert. A. Correct. In fetal circulation, the foramen ovale connects the RA to the LA, allowing a portion of the blood to bypass the RV and the lungs. Approximately a third of the blood that enters the RA passes through this route (preferentially the most oxygenated which is then delivered to the brain and heart), leaving the majority of the blood to travel into the RV. Closure of the foramen ovale is a normal transition from fetal to extrauterine circulation. B. Incorrect. VSDs are common congenital heart defects, and are not considered a part of normal fetal circulation. They are commonly associated with other conditions such as Trisomy 13, 18, 21, and maternal SSRI use. C. Incorrect. In utero, without ventilation, the pulmonary vasculature is a high-resistance system. As such, only 8 to 10% of the blood that enters the RV flows through the circulation. D. Incorrect. The ductus arteriosus does not empty into the LV, but rather into the descending aorta. E. Incorrect. The majority of the fetal circulation travels this route. Approximately 90-92% of the blood that enters the RV (two-thirds of the blood that enters the RA) travels out and through the ductus arteriosus, bypassing the pulmonary circulation and the left heart, ending up in the descending aorta. This blood is perferentially less oxygenated than that which flows through the foramen ovale. Like the foramen ovale, closure of this bypass is a normal transition from intra to extrauterine life.

You are notified that a 10-day-old patient in your practice had a newborn hemoglobin screen positive for sickle cell disease. Pregnancy and delivery were uncomplicated. Mother is 19 years old and works as a nurses' assistant at a nursing home. When questioned, she says she remembers her grandmother died of chest pain and a lung infection. Which of the following should be ordered next for the baby? A. Antibiotics B. CBC C. Chest x-ray D. H. influenza type b vaccination E. Transcranial Doppler

> A has been selected by the expert. A. Correct. This newborn has sickle cell disease. Her grandmother likely passed away from acute chest syndrome, a common complication of this disease. Due to decreased splenic function and consequent decreased resistance to infection with encapsulated organisms (Streptococcus pneumoniae, Haemophilus influenzae type b, Neisseria meningitidis), infants and young children with sickle cell disease are at increased risk for sepsis. When given to infants with sickling disorders, penicillin significantly decreases the risk of mortality from overwhelming sepsis. Prophylaxis is usually continued until the child is five or six years of age (after this, there is little data to support its use except in patients who have had documented sepsis and bacteremia, or who have had their spleens removed). B. A CBC is not warranted immediately in the absence of symptoms of infection in a newborn. C. A chest x-ray is not indicated in this infant with no respiratory symptoms. D. Vaccination against Haemophilus influenzae and S pneumoniae (with the 13-valent conjugate vaccine Prevnar 13) is indicated at 2, 4, and 6 months of age. The meningococcal vaccine is indicated at 2 years of age for patients with functional asplenia, with a booster at 5-6 years of age. The 23-valent pneumococcal polysaccharide vaccine is indicated at 2 years and between 5 and 8 years of age. This infant is too young for administration of the Hib vaccine. E. A transcranial Doppler may be performed in patients with sickle cell disease to assess for stroke risk but is not indicated in an asymptomatic newborn.

A 19-year-old female in her 38th week of pregnancy goes into active labor. Shortly after birth her baby is noted to have a high-pitched cry, tremulousness, hypertonicity, and feeding difficulties. The baby is otherwise developmentally normal and the remainder of the physical exam also is normal. What is the drug the baby's mother likely used during her pregnancy? A. Heroin B. Alcohol C. Marijuana D. Cocaine E. Tobacco

> A has been selected by the expert. A. Heroin is the correct choice. Opiate use during pregnancy may result in several different symptoms, including CNS findings (irritability, hyperactivity, hypertonicity, incessant high-pitched cry, tremors, seizures), GI symptoms (vomiting, diarrhea, weight loss, poor feeding, incessant hunger, excessive salivation), and respiratory findings (including nasal stuffiness, sneezing, and yawning). B. Alcohol is incorrect. Fetal alcohol syndrome has a distinct pattern of facial abnormalities, growth deficiency, and CNS dysfunction. These infants may also exhibit other neurobehavioral deficits such as poor motor skills and hand-eye coordination and learning problems, such as difficulties with memory, attention, and judgment. C. Marijuana is incorrect. There is limited evidence for a withdrawal syndrome associated with marijuana use. D. Cocaine use during pregnancy is not typically associated with withdrawal symptoms. Cocaine has been linked to subtle deficits appreciated later in childhood, including deficits in cognitive performance, information processing, and attention to tasks. E. Tobacco is incorrect. Smoking is not associated with the withdrawal syndrome described above. Smoking has been linked in a dose-dependent manner with lower weight newborns at birth. There is a two-fold increase in low birth weight even in light smokers (< 10 cigarettes per day). Smoking during pregnancy also has been associated with subtle neurodevelopmental deficits in some exposed children.

A 3-hour-old infant boy, born by C-section at 36 weeks to a 30-year-old G1P1 with Apgars of 8 and 9 at 1 and 5 minutes, respectively, is found to be tachypneic in the newborn nursery. His mother has a history of Type II diabetes that was poorly controlled during her pregnancy. She was compliant with prenatal vitamins and took no other drugs during her pregnancy. Prenatal labs, including GBS, were negative. The mother's membranes ruptured 9 hours prior to delivery, she was afebrile, and the amniotic fluid had no meconium. On physical exam, the infant is large for gestational age. He has good air movement through the lungs bilaterally, without retractions or nasal flaring. He appears well perfused with normal cardiac exam. He is not in a flexed posture and has a weak suck reflex. A screening test at 3 hours of life reveals blood glucose of 39 mg/dL. What is the most likely diagnosis? A. Hypoglycemia B. Transposition of the great arteries C. Transient tachypnea of the newborn D. Neonatal sepsis E. Pneumothorax

> A has been selected by the expert. A. Hypoglycemia is a common presentation in an infant born to a diabetic mother with poor glucose control during her pregnancy. The increase in maternal serum glucose stimulates fetal pancreatic beta cells to increase insulin production, and this hyperinsulinemic state leads to hypoglycemia when the placental glucose supply is discontinued after delivery. At < 4 hours of life, a glucometer reading of < 25 mg/dL without symptoms or < 40 mg/dL with symptoms would require intervention to correct the hypoglycemic state. This infant has signs of hypotonia, with absence of flexed posture and weak suck, and a blood glucose reading of 39 mg/dL, making hypoglycemia the most likely diagnosis. B. Transposition of the great arteries is a congenital heart defect in which the aorta and pulmonary artery are switched, resulting in poorly oxygenated blood pumped into the systemic circulation. The infant with transposition is generally cyanotic and will be in respiratory distress. This congenital defect is usual accompanied by a VSD, and maternal diabetes is a risk factor. This infant appears well perfused on exam, and has no murmurs on cardiac exam, making transposition a less likely diagnosis. C. Transient tachypnea of the newborn (TTN) is a condition characterized by delayed clearance of amniotic fluid from the infant's lung following birth (persistent postnatal pulmonary edema) resulting in respiratory distress. Infants born by C-section and to diabetic mothers are at an increased risk of TTN. X-ray findings include "wet" appearing lungs with significant perihilar streaking, interstitial and alveolar fluid, and fluid in the pleural space and along the fissures. TTN generally resolves within 24 to 48 hours and is treated symptomatically. D. Neonatal sepsis is most commonly caused by GBS, Listeria and E. coli, transmitted from mother to baby. Additional risk factors include premature rupture of membranes (> 18 hours prior to delivery), preterm delivery, and chorioamnionitis. Infants may present with fever, trouble breathing, jaundice, and lethargy. Our infant is premature and tachypneic, but he is afebrile with normal Apgars and no evidence of altered level of alertness. Furthermore, mother was GBS negative, afebrile (no chorioamnionitis), with no premature rupture of membranes, making this diagnosis less likely. E. Pneumothorax is collapse of lung tissue secondary to air accumulation in the pleural space. Risk factors for pneumothorax in an infant include previous intubation or underlying lung disease (such as severe respiratory distress syndrome). Characteristic physical exam findings include asymmetric breath sounds or decrease in breath sounds on one side. This infant has good air movement in bilateral lung fields, making this diagnosis less likely.

You see a 6-year-old male in the ED who presents with a history of a 10-second episode of jerking movements of his extremities with unresponsiveness, observed by both of his parents. His parents claim he has had abdominal pain and small quantities of bloody diarrhea for two days. The child has no significant past medical history, has taken no medications recently, has no pets, and has not traveled outside of California in the past year. He attends kindergarten. Which organism is the most likely cause of the child's symptoms? A. Shigella sonnei B. Rotavirus C. Clostridium difficile D. Enterotoxigenic E. coli (ETEC) E. Vibrio cholerae

> A has been selected by the expert. A. Shigella sonnei causes bloody diarrhea and WBCs in the stool on Wright stain. Rarely, children infected with Shigella can suffer from seizures due to neurotoxin release. B. Rotavirus is the most common cause of infectious gastroenteritis. It is an RNA virus that does not cause WBCs to appear in the stool, and typically does not cause bloody diarrhea. Diagnosis is made using ELISA. Rotavirus is not associated with seizures. C. Clostridium difficile typically causes diarrhea after antibiotic use, which allows the organism to overgrow the normal intestinal flora. It would not be expected to cause diarrhea in an otherwise healthy child who is not taking any medications. Infection can be diagnosed by the presence of C. difficile toxin in the stool. D. ETEC is the strain of E. coli that commonly causes traveler's diarrhea, a noninvasive infection without WBCs in the stool. Diarrhea is typically watery and contains no blood. E. Vibrio cholerae causes massive quantities of watery diarrhea, and patients may present with severe dehydration. Bloody diarrhea would not be expected. It is typically seen in endemic or epidemic form in developing countries, and would not be expected in a child living in the U.S. with no recent travel history.

A 7-year-old boy presents with a five-year history of intermittent vomiting, vertigo, and throbbing unilateral headaches that seem to be induced by emotional stress and when his teacher wears perfume. He reports that the pain is not worsened by long naps or coughing. His mother reports that she has a history of headaches that started as a child and wonders if her son inherited this from her. His neurological exam shows no focal deficits. What is the next step in diagnosis or treatment? A. Trial of prophylactic medication for migraine headaches B. MRI C. NSAIDs D. Referral to an ENT surgeon

> A has been selected by the expert. A. This child is presenting with signs of both typical and atypical migraines and could be started on a trial of prophylactic medication. Tricyclic antidepressants (TCAs) are often used in children for migraine prophylaxis, which is the most likely diagnosis in a child with this constellation of symptoms. B. An MRI would be indicated if an intracranial mass is suspected. An infratentorial mass, which could increase intracranial pressure, might cause headaches in the morning after laying down, or also when valsalva'ing during a cough. An infratentorial mass can also compress the cerebellum and may cause ataxia, dysarthria, and nystagmus. A supratentorial mass is more likely to present with focal motor and sensory deficit. C. NSAIDs are often used for tension headaches. Unlike migraines, however, tension headaches cause pain in a band-like pattern around the head. D. While ENT doctors specialize in vertigo that is caused by inner ear pathology, it is likely that this patient's vertigo is due to an atypical migraine.

A 3-month-old male presents to the ED with a fever that started the previous day. Mother reports that he was fussy and had decreased oral intake. He had had five fewer diaper changes than usual. He had no vomiting, diarrhea, or respiratory difficulty. On physical exam his temperature is 101.6 F, pulse 110 bpm, RR 24 bpm, and BP 95/67 mmHg. The baby seems irritable and is not consolable by the parent. HEENT exam was significant for dry mucous membranes. Other than his irritability, the rest of the physical exam was unremarkable. CBC showed WBC 3.5, but was otherwise normal. BMP was within normal limits. Urinalysis showed positive leukocyte esterase, positive nitrite, and WBCs > 10/hpf. An LP was performed, and urine and CSF culture results are pending. The patient is placed on IV fluids and is started on cefotaxime. What is the next best step in evaluation? A. Renal bladder ultrasound B. Kidney-ureter-bladder (KUB) x-ray C. Intravenous pyelogram D. VCUG E. Oral ampicillin

> A has been selected by the expert. A. This infant has a fever without other respiratory symptoms. Meningitis and UTI must be considered in patients with fever. The only way to rule out meningitis is by lumbar puncture. This patient has a low WBC, suspicious for sepsis, and a UA that is highly suggestive of UTI. Empiric therapy should be started to cover common organisms including E.coli, P. mirabilis, and Klebsiella. Cefotaxime is reasonable empiric therapy. Renal ultrasound is recommended for all infants with pyelonephritis to assess for renal structural abnormalities or signs of obstructive uropathy (hydronephrosis). B. KUB is not recommended for UTI. C. Intravenous pyelogram would expose the patient to radiation and would not be recommended to screen for renal abnormalities. D. VCUG screening is recommended only for recurrent UTI or when there is abnormal renal ultrasound. E. The patient is already on parenteral antibiotics, so oral antibiotics would not be necessary. Also, ampicillin would not provide empiric coverage.

A mother brings her 20-day-old male infant to your clinic for the child's first visit. You learn that the infant was born at home to a 28-year-old G1P1, and the infant has not yet received newborn screening. During your history, you learn that the infant has been vomiting 2 to 3 times per day, and the mother reports that her son seems fussier than her friends' infants. On exam, you note an eczematous rash and a musty odor to the infant's skin and urine. Which enzyme deficiency would you expect the infant to display? A. Phenylalanine hydroxylase B. Cystathionine synthase C. Sphingomyelinase D. Alpha-L-iduronidase E. Glucose-6-phosphatase

> A has been selected by the expert. A. This infant likely has phenylketonuria (PKU), an autosomal recessive disorder of amino acid metabolism caused by a deficiency in the enzyme phenylalanine hydroxylase. Affected infants are normally detected by newborn screening, but can present with vomiting, hypotonia, musty odor, developmental delay, and decreased pigmentation of the hair and eyes. The best developmental outcomes occur if a phenylalanine-restricted diet is initiated in infancy. B. A defect in cystathionine synthase occurs in homocystinuria, a disorder of amino acid metabolism. Homocystinuria is inherited in an autosomal recessive pattern. Individuals display Marfanoid body habitus, a hypercoaguable state, and possible developmental delay. The condition can be diagnosed by testing for increased methionine in a patient's urine or blood. C. A defect in sphingomyelinase occurs in Niemann-Pick disease, a lysosomal storage disease. Children present by six months of age with hepatomegaly, ataxia, seizures, and progressive neurologic degeneration. Fundoscopic exam reveals a "cherry-red" macula. D. A defect in alpha-L-iduronidase occurs in Hurler syndrome, a type of autosomal recessive lysosomal storage disease. Children typically do not display symptoms until one year of age. Symptoms include hepatosplenomegaly, coarse facial features, frontal bossing, corneal clouding, and developmental delay. Affected individuals typically do not live past fifteen years old. E. A defect in glucose-6-phosphatase occurs in Von Gierke's disease, a glycogen storage disease. Von Gierke's disease is inherited in an autosomal recessive pattern. Individuals present with hypoglycemia, hepatomegaly, and metabolic acidosis.

A male infant weighing 3200 grams is born to a G1P1 female at 39 weeks' gestational age via planned C-section. Maternal PMH is unremarkable, and GBS status is unknown. Apgars are 7 and 8 at 1 and 5 minutes of life, respectively. The delivery is uncomplicated, and the infant initially appeared in good condition. However, one hour following delivery the infant develops increasing respiratory distress. RR is assessed as 90 breaths/min. All other vital signs are within normal limits. On exam, the infant is acyanotic with rapid respirations and robust capillary refill. Chest x-ray shows bilateral lung fields with the appearance of "a radio-opaque line of fluid in the horizontal fissure of the right lung." No air bronchograms are noted. What is the most likely etiology of the infant's respiratory distress? A. Transient tachypnea of the newborn (TTN) B. Respiratory distress syndrome (RDS) C. Neonatal sepsis D. Meconium aspiration

> A has been selected by the expert. A. Transient tachypnea of the newborn (TTN) is the most likely underlying etiology. This condition is caused by residual fluid in the infant's lungs following delivery, and usually resolves within several days. It is more common in babies delivered via C-section, as the normal mechanical force of labor that helps expel fluid from the lungs is lacking. Babies with TTN and other forms of respiratory distress are often unable to nurse and require feeding via NG tube until respiratory status stabilizes. B. Respiratory distress syndrome (RDS) is less likely than TTN in this case. RDS is more common in premature infants and infants born to diabetic mothers. On chest x-ray, RDS is characterized by a ground-glass appearance and air bronchograms. C. Neonatal sepsis is possible, especially given the mother's unknown GBS status, but relatively unlikely compared to the other options, especially given the mode of delivery. Sepsis can certainly cause respiratory distress and, if suspected, should be promptly evaluated with screening labs and blood cultures. Neonatal sepsis is also more common with prolonged rupture of membranes (PROM) > 18 hours prior to delivery. D. Meconium aspiration can lead to respiratory distress, but seems less likely in this case given the infant's delivery via C-section. Additionally, meconium aspiration is more common when meconium is found in the amniotic fluid and/or products of conception. No mention of this was made in the above case description.

A 6-month-old female with normal birth and developmental history presents with fever for the past two days, fussiness, and decreased appetite. ROS is negative. No abnormalities are noted on the physical examination. A urinalysis from a bag specimen is positive for leukocytes and nitrite, which suggests the presence of a UTI; a culture from this sample is pending. The patient is ill-appearing, dehydrated, and unable to retain oral intake. She is hospitalized, receives a 20 cc/kg NS bolus and is placed on maintenance IV fluids with clinical improvement. What is the best next step for management of this patient? A. Urinary catheterization B. Renal bladder ultrasound C. Begin parenteral antimicrobials D. Midstream clean catch urine collection E. Increase intravenous fluid administration rate to flush the kidneys

> A has been selected by the expert. A. Urinary catheterization is correct . It is the best method for obtaining a specimen for culture that has not been contaminated by perineal bacteria, and for this ill child, you must determine the cause of the fever with accuracy. B. Renal ultrasound may be indicated if the infant is found to have a urinary tract infection but is not indicated as part of the initial work up. C. With a bag culture pending, you may have a contaminated/confusing culture result and may have to rely solely on clinical criteria to treat this presumed UTI. D. Midstream clean catch specimen is incorrect because it is not ideal in a patient who has not been toilet-trained and cannot void on demand. E. While it is important to provide hydration, this patient has responded well to the initial fluid administration, and there is no evidence that increasing this beyond normal recommendations is helpful.

Mark is a 5-month-old male who is brought to the urgent care clinic with a three-day history of rhinorrhea and non-productive cough. When he was born he was large for gestational age, and his exam then was notable for macrocephaly, macroglossia, and hypospadias. On physical exam now his vitals signs are stable. He has copious nasal discharge, but his lungs are clear to auscultation. On abdominal exam, you palpate an abdominal mass on the right side just below the subcostal margin. It is 7 cm in diameter and does not cross the midline. The abdomen is soft and non-tender with active bowel sounds. What is the most likely cause of his mass? A. Wilms' tumor B. Teratoma C. Renal cell carcinoma D. Hepatoblastoma

> A has been selected by the expert. A. Wilms' tumor is commonly associated with Beckwith-Wiedemann syndrome, a genetic overgrowth syndrome. Other features that may be seen in children with this syndrome include omphalocele, hemihypertrophy, hypoglycemia, large for gestational age, and other dysmorphic features. B. Teratomas are congenital tumors that are present at birth. These benign tumors that are often identified incidentally, or may become symptomatic due to mass effect of the lesion within the abdominal cavity. The aggressiveness of the tumor depends on the degree of differentiation. C. Renal cell carcinomas are much more common in adulthood. Risk factors include cigarette smoking and obesity. D. While children with Beckwith-Wiedemann syndrome can have hepatoblastoma (in addition to other types of tumors), this is not the most common tumor in this genetic condition. Note that hepatoblastoma may also be associated with familial adenomatous polyposis.

A 16-year old female presents to the ED with abdominal pain. Upon questioning, the patient notes that the pain is pretty consistently in the RLQ without radiation. She denies dysuria, hematuria, or blood in the stool. She has a history of multiple sexual partners and inconsistent condom use. She does not use any other contraceptive measures. She believes her last menstrual period was 3 weeks ago, but she is unsure. She has no history of abdominal or pelvic surgeries. Her temperature is 100.8 F, heart rate is 85 bpm, respiratory rate is 12 bpm, and blood pressure is 110/70 mmHg. Her abdominal exam is notable for involuntary guarding, tenderness to palpation in the RLQ without rebound tenderness, and no CVA tenderness. Her pelvic exam is notable for cervical motion tenderness with some discharge. What is the best NEXT step in management? A. Abdominal CT B. Pregnancy test C. Pelvic ultrasound D. Cervical cultures E. Empiric antibiotics

> B has been selected by the expert. A. Abdominal CT may be necessary as a part of the future workup if initial workup is negative. However, the history and physical suggest pelvic inflammatory disease, and CT would not be a part of the initial workup. Importantly, given the history of multiple sexual partners, inconsistent birth control use, and questionable LMP, pregnancy is a possibility, and a CT would be harmful to the fetus. B. A pregnancy test is the best first step in management. Pregnancy is one of the indications for inpatient management of PID, so this is very important information when determining whether to admit the patient from the ED or to provide outpatient treatment. While cervical cultures and empiric antibiotics are obviously a must when you suspect PID, pregnancy test is the first step, and the best answer. C. Pelvic ultrasound is a useful technique to check for tubo-ovarian abscess or pregnancy (including ectopic), but a pregnancy test is still the best first choice. Also, pelvic ultrasound as a diagnostic tool is controversial, and often the diagnosis is made clinically. D. Cervical cultures are typically performed, looking specifically for N. gonorrhoeae or C. trachomatis, but a pregnancy test is still performed first. E. Physicians will often begin empiric antibiotic immediately based on this presentation, as it is classic for PID. However, they will always get a pregnancy test first to determine if admission for IV antibiotics is necessary rather than outpatient treatment.

An 8-year-old girl comes to the clinic with a chief complaint of a "cold" for the past two weeks. On further questioning, she developed a fever of 38.7°C, purulent nasal secretions, malodorous breath, and a nocturnal cough three days ago. Examination of the nose reveals pus bilaterally in the middle meatus, and tenderness over the mid-face. Which of the following is the most likely diagnosis? A. Allergies B. Maxillary sinusitis C. Asthma D. Frontal sinusitis E. Middle ear infection

> B has been selected by the expert. A. Allergies are a very common diagnosis in pediatrics, and nocturnal cough is a frequently associated symptom. However, allergies usually cause clear, thin nasal secretions and are usually not associated with fever and malodorous breath. Other physical exam findings associated with allergies may include allergic shiners, cobblestoning of the posterior pharynx, and edematous turbinates. B. The maxillary and ethmoid sinuses are large enough to harbor infection in infancy. The sphenoid sinuses do not become large enough until the third to fifth year of life, and the frontal sinuses are rarely large enough until the sixth to tenth year of life. Sinusitis is characterized by the findings in the question stem, and is often preceded by a URI. Pus draining from the middle meatus is suggestive of either maxillary, frontal, or anterior ethmoid sinusitis. C. Asthma, another common diagnosis in pediatrics, is often triggered by a URI, and is frequently associated with a nocturnal cough. Fever, purulent nasal secretions, and malodorous breath are not caused by asthma. End-expiratory wheezing, atopic diathesis, and a positive family history would support a diagnosis of asthma. D. Distinguishing between the different locations of sinusitis can be clinically challenging, especially in a 9-year-old patient who likely has mature sinuses. However, frontal sinusitis is characterized by pain over the frontal bone and perhaps facial swelling in an older child or adolescent. It is less likely to cause the symptoms described here. E. Acute otitis media is a common diagnosis in pediatrics. It can follow a URI or occur simultaneously with sinusitis. However, this patient does not complain of ear pain.

Claire is a 16-year-old female who presents for birth control management. Her review of symptoms is unremarkable except for chest pain. When you ask her more questions, she reveals the pains are intermittent, on and off for the past couple months. It is not associated with exertion, sharp, and well localized at the left sternal border. It is very brief, lasting only a few seconds, during which she says she sometimes notices it gets worse when she breathes in. She denies recent URI or viral illness. The family history is negative for early cardiac disease. Her vital signs and physical exam are normal. Which is the next best step in management? A. ECG B. Reassurance C. Referral to a cardiologist D. Fast ultrasound of pericardial window E. Chest x-ray

> B has been selected by the expert. A. An ECG would be next if you suspected a cardiac abnormality. This would be more likely if the patient described something more like angina, a crushing chest pain or pressure, for longer periods of time rather than a few seconds, and aggravated by exercise. B. Based on the history, and assuming your physical exam is unremarkable, this sounds most suggestive of precordial catch syndrome, the most common cause of chest pain in an adolescent. No further workup is needed. C. Associated symptoms of syncope or palpitations would suggest a more severe cardiac abnormality and would prompt a referral to a cardiologist. The vignette states the rest of the ROS is negative and this would not warrant further evaluation now by a cardiologist. D. This would be the management if you suspected pericarditis or a pericardial effusion, but she denies recent URI or viral infection, and your PE is not notable for a pericardial friction rub suggestive of pericarditis. Her vitals are also stable and you might expect fever in pericarditis. E. A chest x-ray would be indicated if you suspected pneumonia that potentially might present with chest pain. However, the vignette didn't mention anything about cough and the patient is afebrile, making this less likely.

A 7-year-old boy is brought by ambulance to the ED with altered consciousness. The EMT said he found the boy in a pool of vomit. He is unable to answer questions coherently and he is alone. Physical exam findings indicate dry mucous membranes, tachypnea, tachycardia, and moaning on palpation of the abdomen. His physical exam is otherwise normal, including a normal blood pressure. What is the most likely cause of his condition? A. Appendicitis B. DKA C. Narcotic overdose D. Non-accidental trauma E. Sepsis

> B has been selected by the expert. A. Appendicitis would rarely present with altered consciousness. The usual history with appendicitis is onset of periumbilical pain that persists over hours, migrating to the right lower quadrant. Vomiting could be present, and tachycardia may be present due to pain or dehydration, but altered mental status would be unusual. On physical exam, peritoneal signs may be present as well as psoas, obturator, or Rovsing's sign. B. DKA typically presents with altered mentation, vomiting, dehydration, and abdominal pain. The history will yield polydipsia and polyuria during the days preceding DKA. Metabolic acidosis causes tachypnea as the body tries to blow off CO2 through a compensatory respiratory alkalosis. C. Although he may have access to narcotics, his presentation does not fit well with this choice. Signs of narcotic overdose include pinpoint pupils, depressed respiratory rate, and altered consciousness. His tachypnea and lack of pinpoint pupils argue against this choice. D. Lack of fractures, bruises, or burns argues against this choice. Trauma resulting in increased intracranial pressure may result in hypertension, bradycardia, and disordered breathing. E. Sepsis can present with altered mental status. This child's presentation is less consistent with sepsis given that he doesn't have fever or other vital sign changes consistent with sepsis syndrome (temperature > 38.5°C or < 36°C, hypotension, along with warm, dry extremities).

A previously healthy and developmentally normal 16-month-old male comes to the urgent care clinic with his father with a chief complaint of his first reported seizure. The child was reported to have dropped to the floor with loss of consciousness and had sporadic twitchy movements of his legs and arms that lasted for five minutes. The child has had URI symptoms for the past two days, with a fever to 103 degrees F without any changes in mental status. Neither parent has a seizure disorder, but the child's mother reports having a single seizure as a young girl once after developing a high fever after a cold. What is the most likely diagnosis? A. Epilepsy B. Simple febrile seizure C. Complex febrile seizure D. Absence seizure E. Cyanotic breath-holding spell

> B has been selected by the expert. A. Choice A is incorrect because epilepsy is a recurring condition, while this seizure is this infant's first episode. In addition, the infant has had URI symptoms with a high fever, likely leading to a febrile seizure. Epilepsy seizures are usually classified as unprovoked. If this is indeed a simple febrile seizure, the child's risk of developing epilepsy in the future is about 0.5-1% above the normal risk. If this child continues to have febrile seizures or starts experiencing complex febrile seizures, the risk for epilepsy in the future is even greater. B. Choice B is correct. Febrile seizures are one of the most common causes of seizures in children. Simple febrile seizures are more common than complex febrile seizures and are characterized by < 15 minutes duration, occurring only once in a 24-hour period, and are generalized (in this patient's case, generalized tonic-clonic). Febrile seizures are usually self-limited events triggered by an acute febrile illness. A positive family history for febrile seizures in the parents makes it a more likely diagnosis in their children. This particular diagnosis fits this patient the best. C. Choice C is incorrect because a complex febrile seizure is characterized by > 15 minute duration, occurring more than once during a 24-hour period, and being focal. Given that this child's seizure was associated with loss of consciousness and tonic-clonic movements lasting five minutes, and occurred only once during a 24-hour period, complex febrile seizure is not the most likely diagnosis. D. Choice D is incorrect because absence seizures are generalized seizures, but children recover consciousness much more quickly, usually within 30 seconds, compared to tonic-clonic seizures, which take much longer. Absence seizures are most often seen in children starting from the age of 3 and are not associated with loss of tone. Potential causes include hyperventilation or photic stimulation. Given that this child's seizure occurred after URI symptoms and high fever and also was characterized by tonic-clonic jerks lasting about five minutes, this is not the most likely diagnosis. E. Choice E is incorrect. Breath-holding spells are most commonly seen in infants from 6 months to 6 years of age. The pathogenesis is not entirely understood but may involve autonomic nervous system dysfunction. In a cyanotic breath-holding spell the infant becomes angry or upset and there is a period of crying, followed by breath-holding in a forced expiration state that may lead the child to lose consciousness and become cyanotic. The child usually recovers soon thereafter with no residual side effects. Given that this child was not reported to have been angry or crying prior to losing consciousness and did not become cyanotic, and because he demonstrated tonic-clonic jerking movements for five minutes, a breath-holding spell would not be a likely diagnosis.

You are seeing a 1-month-old male who is < 3rd percentile for weight. He is breastfed every 2 hours and latches on well. However, he has frequent non-bilious episodes of vomiting that have been increasing over the past week despite his mother taking "reflux precautions." He does not have mucus or blood in his stool. Physical exam reveals a small, olive-sized mass in his abdomen. What is the most likely diagnosis? A. Cleft palate B. Pyloric stenosis C. Cystic fibrosis D. Non-organic failure to thrive E. Munchausen syndrome by proxy

> B has been selected by the expert. A. Choice A is incorrect. Children with structural anomalies of the palate typically present with difficulty feeding. This child appears to be latching on well, but has difficulty retaining the food once ingested. B. Choice B is correct because the history of frequent vomiting, poor weight gain, and the finding of an abdominal mass are consistent with pyloric stenosis. Children with pyloric stenosis often present at 3 weeks of age. C. Choice C is incorrect. Children with cystic fibrosis typically present with failure to thrive secondary to chronic malabsorption, with characteristic loose and malodorous stool. Lab testing reveals elevated sweat chloride. There may be a known family history. D. Choice D is incorrect. Several findings in this patient point toward an organic rather than an inorganic cause of failure to thrive, and organic causes such as pyloric stenosis should be ruled out before an inorganic cause is assumed. E. Choice E is incorrect. Although maltreatment of a child should always be considered, particularly in the youngest children. This child has sufficient findings by history and physical exam to suspect true organic etiology.

A 9-year-old male is brought to the ED in a coma secondary to diabetic ketoacidosis. Which of the following laboratory results would NOT likely be found in this patient? A. Anion gap of 20 mEq/L B. Potassium of 3.3 mEq/L C. Venous pH of 7.1 D. Sodium of 132 mEq/L E. Creatinine of 1.0 mEq/L

> B has been selected by the expert. A. DKA causes a metabolic acidosis from the elevated level of ketones. The elevated level of ketoacids and lactic acid requires buffering by bicarbonate, thus leading to an increased anion gap. B. In diabetic ketoacidosis, the acidosis and lack of insulin cause potassium to leave cells and enter the serum, causing an elevated serum potassium level. However, as the DKA is corrected and insulin is administered, the potassium will re-enter the cells, causing a decreased serum potassium level, so potassium levels should be monitored closely when therapy is initiated. C. The pH would be low due to the metabolic acidosis caused by the elevated level of ketones. D. Hyponatremia is seen in DKA because the hyperosmolarity of the intravascular space from the increased glucose levels causes osmotic movement into the extracellular space. Additionally, there is increased sodium loss from the kidneys. A corrected sodium level should be calculated to adjust for the hyperglycemia, using the following formula: corrected sodium = [{(measured glucose - 100)/100} x 1.6] + measured sodium. E. Patients with DKA are often dehydrated when they present to the ED. This causes a prerenal azotemia, which presents as an elevated creatinine level.

Susie is a 3-year-old girl brought into the clinic by her mother because she has a gradually worsening cough and she has been having trouble breathing. Her mother says Susie sounds like she is barking when she coughs. Susie is up to date with her vaccinations. Susie's mom always watches her when she's playing. On physical exam, you note that Susie has inspiratory stridor. She does not have wheezing, there are no retractions, and she has symmetrical breath sounds. No pseudomembranes are appreciated on physical exam. What is Susie's most likely diagnosis? A. Epiglottis B. Croup (laryngotracheobronchitis) C. Pertussis D. Pneumonia E. Foreign body aspiration

> B has been selected by the expert. A. Epiglottitis is a life-threatening emergency caused by an infection with H. influenzae type B. It is less common now with the advent of Hib vaccine, but in rare cases can occur due to staphylococcal or streptococcal infections. It most often occurs in children ages 2 to 5 years. Children with epiglottitis present with fever, stridor, drooling, dysphonia, dysphagia, and respiratory distress. They frequently appear toxic and sit in the "sniffing position" (sitting, leaning forward, neck hyperextended, chin protruding). A "thumb sign" (thickened epiglottis and aryepiglottic folds) appears on films. Susie is not exhibiting any of these characteristic symptoms and she is up to date with vaccines, making epiglottitis a less likely diagnosis for her cough. B. Croup or laryngotracheobronchitis is due to a viral infection (Parainfluenza type 1). It is most common in the winter, and often occurs in children age 2 to 5 years. Croup can lead to non-specific URI symptoms with some degree of airway obstruction. A barky or seal-like cough and inspiratory stridor (which should be differentiated from expiratory wheezes) is common in croup. C. Pertussis occurs in three phases: The catarrhal stage lasts one to two weeks when children present with URI symptoms. A paroxysmal stage follows and lasts four to six weeks. During this phase, children have repetitive forceful coughing with massive inspiratory effort ("whoops"). Finally, during the convalescent stage, children present with continued cough that may last up to three months. The acellular pertussis vaccine protects against pertussis, and, as Suzie is up to date with her immunizations, this diagnosis is less likely. Her barking cough is more suggestive of croup than the "whooping" cough of pertussis. D. While retractions and cough are present with pneumonia, asymmetric breath sounds and tachypnea would be more specific for a diagnosis of pneumonia. E. Symmetrical breath sounds and the gradual onset of Susie's cough make foreign body aspiration less likely. One might also expect focal wheezing and tachypnea with foreign body aspiration.

A 33-year-old G1P0 female with a history of medically controlled seizures gives birth vaginally to a boy with IUGR at 38 weeks' gestation. The newborn is noted to have dysmorphic cranial features and his head circumference is 28.5 cm (< 5th percentile). What is another associated abnormality you might expect to see in this newborn? A. Hepatosplenomegaly B. Cardiac defects C. Absent red reflex D. Chorioretinitis E. Tremors

> B has been selected by the expert. A. Hepatosplenomegaly in newborns is seen in metabolic diseases, storage diseases, HIV vertical transmission, intrinsic liver disease, and in congenital infections. Maternal anticonvulsant use does not cause hepatosplenomegaly. B. The mother was on an anticonvulsant for her seizures. Taking anticonvulsants during pregnancy may lead to cardiac defects, dysmorphic craniofacial features, hypoplastic nails and distal phalanges, IUGR, and microcephaly. Mental retardation may be seen. A rare neonatal side effect is methemoglobinuria. C. The red reflex is the normal reddish-orange reflection of light from the eye's retina that is observed when using an ophthalmoscope. An absent red reflex in a newborn can be due to congenital cataracts or a retinoblastoma. D. Chorioretinitis in a newborn may be due to congenital toxoplasmosis and CMV infections. It is not caused by maternal anticonvulsant use. E. Tremors in a newborn can be due to maternal substance use. They would not result from maternal use of anticonvulsants.

A 12-year-old girl presents to her PMD complaining of a headache of gradual onset x 3 hours, non-provoked and described as a "big rubber band around my whole head" and a 5 out of 10 on the pain scale. The pain is not throbbing, and there is no associated photophobia, nausea or vomiting. The patient is afebrile, and there are no neurologic deficits during physical exam. Her mother states her pain is typically relieved with ibuprofen, but her mother is concerned that patient may have migraines because she has a few headaches every month after school. The child is otherwise healthy. What is the most likely cause of this girl's headaches? A. Migraine B. Tension-type headache C. Brain tumor D. Sinusitis E. Pseudotumor cerebri

> B has been selected by the expert. A. Migraines are typically described as having "throbbing"-type pain and may also be associated with nausea and/or vomiting with photophobia and phonophobia. Pain is often described at 10/10 and is debilitating. B. Tension-type headaches are often bilateral and involve the forehead, temporal areas, or back of the head. Tenderness of the posterior muscles of the neck may also be present. They should be responsive to NSAIDs. Stress can give rise to a tension headache, and this is consistent with this patient developing headaches after school. C. A brain tumor is unlikely to present without focal neurological findings, and the short duration of symptoms do not warrant further workup for this. In addition, the headache is less likely to be relieved with NSAIDs and would not occur only a few times a month. Headaches of new onset that do not go away and are worse in the mornings and associated with vomiting and personality changes would be most concerning for a brain tumor etiology. D. The patient is otherwise healthy and did not complain of any nasal stuffiness, rhinorrhea, or sore throat with postnasal drip, making sinusitis unlikely. E. Pseudotumor cerebri presents as symptoms of increased intracranial pressure-headache, nausea, blurred vision, diplopia, photophobia, and tinnitus-that occur in the absence of a intracranial mass. CT/MRI will be negative, but opening pressures are elevated on lumbar puncture. It typically occurs in obese women of childbearing age, with the median age of diagnosis around 30 years. While the cause is unknown, other risk factors include OCPs, growth hormone, excess vitamin A, and discontinuing steroids. This girl is probably too young to be given this diagnosis, and tension headache is more likely given the lack of symptoms of intracranial pressure.

Rashid is a 5-week-old baby boy who presents to clinic with 4 days of repeated, forceful, non-bilious, non-bloody vomiting without diarrhea. He has 8 to 9 episodes of vomiting per day immediately following breastfeeding. The episodes started 2 weeks after the entire family suffered from severe viral gastroenteritis. His birth history is uncomplicated (full term, NSVD, unremarkable 30-week ultrasound) and birth weight was 3.6 kg (50th percentile). On exam, his vitals are: T 36.7°C, HR 185, BP 85/45, RR 36, Wt 4.1 kg (25th percentile). On exam, his eyes are moderately sunken without production of tears, his lips are cracked, and his throat is without erythema. His capillary refill is ~3 seconds, and his pulse is thready. What is your first step in management? A. Close observation in the office for 6 hours and encourage PO intake until vitals normalize. B. Intravenous lactated Ringer's solution of 20mL/kg boluses until baseline clinical status is achieved, then 100 mL/kg oral rehydration solutions over next 4 hours. C. Intravenous 20 mL/kg boluses of ¼ normal saline solution until baseline clinical status is achieved, then closely monitor vitals for 6 hours while encouraging PO formula intake. D. Observe for 6 hours with normal PO intake and administer 60-120 mL of oral rehydration solution for every episode of vomiting. E. Administer 75 mL/kg of oral rehydration solution over 3-4 hours and 60-120 mL of oral rehydration solution for every episode of vomiting.

> B has been selected by the expert. A. Observing closely in the office for 6 hours and encouraging PO intake until vitals normalize are not appropriate measures for treating this child's severe dehydration. Signs of severe dehydration include lethargy or unconsciousness on exam, poor PO intake, tachycardia, weak or nonpalpable pulses, deep breathing, deeply sunken eyes, parched mouth and tongue, reduced skin turgor, and cold/cyanotic extremities. In cases like this, the child must be placed on immediate IV fluids with 20 mL/kg boluses until vitals and mental status normalize. B. Lactated Ringer's solution or normal saline in 20 mL/kg boluses until urine output is established and mental status improves, then 100 mL/kg oral rehydration solutions over next 4 hours. This follows current CDC guidelines for treating a severely dehydrated child. Intravenous hydration with 5% dextrose ½ normal saline at twice maintenance fluid rates may be substituted for the oral rehydration solution if the child is not tolerating PO intake. To replace ongoing losses, the CDC recommends 60-120mL of oral rehydration solution per diarrheal/emetic episode (through a nasogastric tube, if necessary). C. One quarter normal saline (1/4 NS) is a hypotonic solution, and would not be ideal for the treatment of dehydration due to emesis. The recommended therapy to correct severe dehydration is to give 20 mL/kg boluses of isotonic solution and to reassess for clinical improvement following each administration. Once the patient is stable and back to baseline, then continue IV hydration with 5% dextrose ½ normal saline at twice maintenance fluid rates OR give 100 mL/kg oral rehydration solution over 4 hours. D. This would not be a recommended treatment for the severely dehydrated child since it relies on normal PO intake. However, the replacement of losses strategy is correct for all patients < 10 kgs. E. IV rehydration is key to initial fluid resuscitation in the severely dehydrated, so this treatment would not be sufficient. However, this plan would be appropriate for mild to moderately dehydrated patients.

A 16-year-old obese Caucasian female with a history of irregular menses presents to the ED with severe abdominal pain and altered mental status. She uses intravenous drugs weekly. She has regular unprotected sexual intercourse with multiple male sexual partners. She has experienced fevers, nausea, vomiting, and right shoulder pain and reports no vaginal bleeding. She has not regularly seen a physician for years. Only bedside studies are performed. Vitals are T 38.0 C, BP 90/60 mmHg, P 120 bpm, R 20 bpm. Qualitative B-hCG is positive, and hemoglobin is 7 g/dL. On exam, she is in apparent distress and has difficulty answering questions. Auscultation of the chest is clear. The abdomen is somewhat rigid with tenderness in the right lower quadrant as well as guarding and rebound tenderness. On pelvic exam, there is cervical motion tenderness but no bleeding or masses noted. What is the most likely diagnosis? A. Pelvic inflammatory disease B. Ruptured ectopic pregnancy C. Fitz-Hugh-Curtis syndrome D. Appendicitis E. Hepatitis

> B has been selected by the expert. A. Pelvic inflammatory disease (PID) is a possibility, as indicated by the patient's sexual history and cervical motion tenderness. Symptoms of PID include lower abdominal or pelvic pain, vaginal discharge, fever, fatigue, nausea, vomiting, diarrhea, dysuria, and dyspareunia, among other symptoms. Although PID may occur during pregnancy, it would not account for the low hemoglobin, peritonitis, or vital signs suggesting fluid loss/bleeding. B. The patient likely has a ruptured right-sided ectopic pregnancy, as indicated by the positive pregnancy test and hemodynamic instability. Hemorrhage into the peritoneum may irritate the peritoneum and cause referred pain to the right shoulder. Cervical motion tenderness may also be found. Patients may experience nausea, vomiting, and fever. C. Fitz-Hugh-Curtis syndrome is a possible complication of pelvic inflammatory disease, a possibility given the patient's sexual history. The abdominal pain would be felt in the right upper quadrant and may be referred to the right shoulder if the peritoneum becomes irritated. Although Fitz-Hugh-Curtis syndrome may occur during pregnancy, Fitz-Hugh-Curtis syndrome would not account for the low hemoglobin, peritonitis, or vital signs suggesting fluid loss/bleeding. D. Appendicitis would present as right lower quadrant pain. Symptoms include nausea, vomiting, fever, diarrhea, and constipation. On physical examination, the patient may have peritoneal signs. It is unlikely that appendicitis would cause low hemoglobin or hemodynamic instability. E. With acute hepatitis, patients may present with anorexia, nausea, vomiting, fatigue, malaise, and fever, among other symptoms. Laboratory studies would reveal elevated AST and ALT. History may be positive for intravenous drug use, immigrant Asian mother, or travel. The pain would be felt in the right upper quadrant and may be referred to the right shoulder if the peritoneum becomes irritated. The patient would unlikely be hemodynamically unstable and severely anemic.

An 11-year old boy presents to clinic with wheezing. Mom states that in the past he has used inhaled albuterol and it has helped with wheezing and shortness of breath. On further history you find out that the patient experiences shortness of breath three times a week and is awakened at night by these symptoms once a week. What is the most appropriate outpatient therapy? A. Only rescue inhaler PRN B. Low dose inhaled corticosteroids C. Medium dose inhaled corticosteroids and course of oral corticosteroids D. Medium dose inhaled corticosteroids, LABA, and course of oral corticosteroids E. Course of oral corticosteroids

> B has been selected by the expert. A. Rescue inhaler (a short-acting beta agonist) i PRN is incorrect because this treatment is indicated in patients with intermittent asthma and have symptoms fewer than two days a week or two nights a month B. Low dose inhaled corticosteroid is correct because this patient has mild persistent asthma. His symptoms occur 3-6 days/week and 3-4 nights/month. C. Medium dose inhaled corticosteroids with a course of oral corticosteroids is incorrect, because it would be indicated in a patient with moderate persistent asthma when symptoms occur daily and more than one night per week. D. Medium dose inhaled corticosteroids, LABA, and oral corticosteroids is incorrect because this patient does not have severe persistent asthma. E. A course of oral corticosteroids alone is incorrect. Asthma needs to be managed long term to prevent exacerbations. An inhaled corticosteroid is indicated.

A 10-month-old infant is brought to the Peds ED by her parents, who say she has been coughing persistently for the last three hours. The parents were watching a movie at home when they first noticed their daughter coughing. Patient is a vaccinated, well-nourished infant in moderate distress with retractions, nasal flaring, and grunting. On auscultation, you immediately notice diminished breath sounds in the right lung with normal breath sounds on the left. What other associated physical exam finding do you expect to hear? A. Stridor B. Asymmetric breath sounds and wheezing C. Rhonchi D. Crackles E. Bronchial breath sounds

> B has been selected by the expert. A. Stridor is due to airway narrowing above the thoracic inlet and can be seen in URI such as epiglottitis. You might expect stridor in laryngomalacia, but this would not have suddenly appeared at 10 months of age. Also, epiglottitis is more rare now in a vaccinated child, and you wouldn't expect to hear unilaterally diminished breath sounds. B. This infant is in respiratory distress from foreign body aspiration, consistent with the history of acute onset of distress and asymmetric breath sounds. Common foreign bodies include peanuts, popcorn, grapes, hard candy and hot dogs. Respiratory distress from foreign body aspiration is usually accompanied by asymmetric breath sounds and wheezes on auscultation. C. Rhonchi are coarse, low-pitched, rattling sounds due to secretions and airway narrowing and are typically heard in the setting of bronchitis or pneumonia. D. Crackles are due to fluid in alveoli or opening and closing of stiff alveoli, not consistent with a foreign body aspiration. You would expect this in either pneumonia or CHF from pulmonary edema, both of which are unlikely in this patient given the abrupt onset and lack of history of cardiac problems. E. Bronchial breath sounds are hollow-sounding and caused by air moving through areas of consolidated lung, such as in the setting of pneumonia.

A 4-day-old baby boy presents for his first pediatric well child visit. His birth history consists of an uncomplicated normal spontaneous vaginal delivery after 7 hours of labor-no vacuum or forceps assistance were used. The patient is the first child to a 30-year-old mother of Mediterranean descent. Mom is very concerned that her baby has started to look "yellow" since leaving the hospital. She has been breastfeeding every 2-3 hours and says that the baby latches on for 1-5 minutes for each feed. He has had few wet diapers, and mom is concerned he is not getting enough to eat. Which of the following would most aid in narrowing the differential diagnoses? A. Newborn screen results B. Fractionated bilirubin C. WBC D. Blood smear E. No further workup is needed, as this is likely physiologic jaundice

> B has been selected by the expert. A. The newborn screen result normally takes a few weeks to process. However, even if the results were available, they might suggest two possibilities in the differential diagnosis of jaundice: hypothyroidism or G6PD deficiency. Even though these diagnoses are important to rule out, the test would not provide more information about the infant's current state of health or the next step in treatment. B. The test that will give you the most information at this juncture is a fractionated bilirubin. With the knowledge of the total serum bilirubin (TSB) and direct serum bilirubin, one will be able not only to narrow the differential (hemolysis vs. obstruction), but also to guide treatment (i.e., indirect serum bilirubin may be above phototherapy level). TSB can also indicate if the situation requires more drastic measures, such as a transfusion exchange. C. A WBC could be helpful if the infant is at risk or showing signs of infection or sepsis. The child did not have a complicated birth history, and now has no signs of fever or distress. Although it may be helpful to check a CBC and know the hemoglobin level, in case of hemolysis, a WBC will not help narrow the differential as much as a fractionated bilirubin given the above presentation. D. A blood smear would be helpful to diagnose hemolysis or RBC membranopathies. However, a blood smear would not be the most useful in narrowing the wide range of possibilities in the current differential, because it would help with identifying hemolytic causes. At this juncture, a fractionated bilirubin would be the most useful test to order first. E. Although physiologic jaundice is the most likely cause of jaundice at this age for an infant, there is not yet enough information to rule out a more serious cause. Because the mother is of Mediterranean descent, it is very possible that the cause of jaundice is G6PD deficiency, so further workup is definitely necessary. At the very least, a fractionated bilirubin is needed, and then likely a CBC.

A 4-year-old male with a history of Down syndrome and no other medical problems is brought to his pediatrician's office by his mother for increasing fatigue, intermittent fever, and decreased appetite for one week. On exam you note conjunctival pallor and hepatosplenomegaly. What is the best next step in the management of this patient? A. Advise the mother to use acetaminophen as needed for fever and discharge home with strict return precautions. B. Send CBC and peripheral smear C. Send patient for chest x-ray D. Send TSH E. Start the patient on a 7-day course of amoxicillin and send home with strict return precautions.

> B has been selected by the expert. A. This choice is incorrect because the patient is likely presenting with signs and symptoms of acute leukemia in a patient at increased risk for childhood malignancy (specifically ALL). B. This choice is correct because fatigue, decreased appetite, anemia, and HSM are all clinical signs and symptoms that may be associated with acute leukemia, for which patients with Down syndrome are at increased risk. A CBC would be helpful both for assessing the white blood cell count as well as the degree of anemia. A peripheral smear would be indicated to further evaluate for leukemia. C. This choice is incorrect because the patient is not presenting with any acute respiratory symptoms to warrant immediate chest x-ray. D. This choice is incorrect because, although Down syndrome may be associated with hypothyroidism, the patient's fever and physical exam findings are suggestive of a separate process. E. This choice is incorrect because the patient is presenting with signs and symptoms concerning for acute leukemia. Although infection may complicate leukemia, an outpatient course of amoxicillin would not be appropriate therapy at this time.

A 19-year-old G1P0 presents in labor to the ED at 38 gestational weeks. On interview it is discovered that the patient had irregular prenatal care, drank a couple of beers every weekend, and smoked 4 cigarettes a day. She delivers a baby boy who is small for gestational age. On exam, it is noted the baby has microcephaly, a smooth philtrum, and a thin upper lip. What do you suspect caused these features in the baby? A. Tobacco exposure B. Alcohol exposure C. Congenital rubella D. Vertically transmitted HIV E. Congenital CMV infection

> B has been selected by the expert. A. This choice is incorrect. While tobacco exposure can cause infants to be small for gestational age they typically do not have any characteristic facial features. B. This choice is correct. Fetal alcohol syndrome has very characteristic facial features, including a smooth philtrum, thinning of the upper lip, and small palpebral fissures. C. This choice is incorrect. Congenital rubella presents with sensorineural deafness, eye abnormalities (retinopathy, cataracts), and patent ductus arteriosus. D. This choice is incorrect. Typically, vertically transmitted HIV does not lead to recognizable symptoms at birth. This diagnosis cannot be completely ruled out without lab testing. E. This choice is incorrect. Symptomatic congenital CMV infection presents with microcephaly, jaundice, hepatosplenomegaly, low birth weight, and petechiae at birth.

A 10-month-old asymptomatic infant presents with a RUQ mass. Work-up reveals a normocytic anemia, elevated urinary HVA/VMA, and a large heterogeneous mass with scant calcifications on CT. A bone marrow biopsy is performed. Which of the following histologic findings on bone marrow biopsy is most consistent with your suspected diagnosis? A. Sheets of lymphocytes with interspersed macrophages B. Small round blue cells with dense nuclei forming small rosettes C. Hypersegmented neutrophils D. Stacks of RBCs E. Enlarged cells with intranuclear inclusion bodies

> B has been selected by the expert. A. This is incorrect, as sheets of lymphocytes with interspersed macrophages are associated with Burkitt lymphoma. B. This is the correct response. In addition to neuroblastoma, other tumors associated with small blue cells include Ewing's sarcoma and medulloblastoma, both of which tumors are seen in children. C. Hypersegmented neutrophils are characteristic of megaloblastic anemia, a condition associated with a vitamin B12 and/or folate deficiency, not malignancy. D. Stacks of RBCs suggest rouleaux formation, a phenomenon seen in multiple myeloma, a condition not seen in young infants. E. This describes the classic "owl's eyes" seen in CMV and other viral infections.

A 16-year-old female presents to clinic complaining of worsening fatigue. Family history is significant for hypothyroidism and heavy periods in the grandmother. Her exam reveals mild tachycardia and oozing around a recent piercing, but is otherwise normal. Labs reveal Hgb 8.5 g/dL, MCV 58, PT 12.5, PTT 44, and low von Willebrand factor activity. Which of the following is the most appropriate treatment for her underlying disorder? A. Blood transfusion and iron supplementation B. Desmopressin C. Factor VIII concentrate D. Cryoprecipitate E. Vitamin K

> B has been selected by the expert. A. This patient has iron deficiency anemia, as indicated by her hemoglobin level and MCV. Nutritional deficiency is the most common cause of iron deficiency anemia, and in these cases iron supplementation would be appropriate. However, a blood transfusion and supplemental iron for a presumed nutritional deficiency without investigating potential ongoing losses is inappropriate, especially in a patient with oozing blood on exam. B. Von Willebrand's disease is the underlying cause of this patient's anemia, as indicated by the low von Willebrand factor activity. This is the most common hereditary bleeding disorder, occurring in roughly 1% of the population. Intranasal or intravenous desmopressin is appropriate treatment for most bleeding problems. Desmopressin works by causing release of von Willebrand's factor from vessel endothelial cells. C. Factor VIII concentrate may be used as a treatment for von Willebrand's disease in the setting of major trauma or surgery. It also may be used for less common subtypes of the disease that do not respond to desmopressin. However, most cases of von Willebrand's disease can be treated with desmopressin, and therefore it is the most appropriate first choice in treating this patient. D. Cryoprecipitate is a blood product containing fibrinogen, factor VIII, factor XIII, and von Willebrand factor. It is most commonly used to replace fibrinogen. It may also be used in certain scenarios for patients with von Willebrand's disease. However, it is not as good as desmopressin as a first choice in this scenario. E. Vitamin K is found in leafy green vegetables and in most multivitamins. It is important in the synthesis of clotting factors. It is given clinically as a one-time IM dose for neonates to assist in clotting factor formation until the gut microbiome is sufficient to provide Vitamin K. It is also used for patients who become supratherapeutic on warfarin therapy. It would not help in this case.

Adam is a newborn male who was just born to a G2P1 mother at 36.2 weeks' gestation via a vaginal delivery. The mother reports that she did not receive prenatal care because she did not have insurance. She says that she thinks her "water broke" about two days ago, but she did not have any contractions after that, so she decided not to come to the hospital. She did not start having contractions until 19 hours before she delivered. After delivery, Adam did not cry vigorously, was tachypneic, cyanotic, and febrile to 100.5 F. Amniotic fluid did not contain meconium. His chest x-ray is normal. Given Adam's birth history, what is the most likely cause of his symptoms? A. Transient tachypnea of the newborn (TTN) B. Sepsis secondary to prolonged rupture of membranes C. Meconium aspiration syndrome D. Hypothermia E. Pneumothorax

> B has been selected by the expert. A. Transient tachypnea of the newborn (TTN) is a benign, self-limited condition caused by delayed clearance of lung fluid after birth. Patients with TTN usually have a classic chest x-ray that shows coarse fluffy densities that represent fluid-filled alveoli and/or fluid in the pleural space and a small amount of fluid in the fissures on the lateral view. Given Adam's normal chest x-ray and fever, it is unlikely that Adam has TTN. B. Prolonged rupture of membranes (PROM) is when the chorioamniotic membrane ruptures before the onset of labor. The main risks associated with PROM are preterm labor and delivery and neonatal sepsis. Adam's mom said that her "water broke" two days ago, which indicates that she had PROM. Adam's mother also did not receive prenatal care; therefore, she did not receive any of the prenatal screening tests that she should have, which increases the likelihood that she has an infection that could have potentially been transferred to Adam after the rupture of her membranes. Adam's history of PROM along with his fever and respiratory distress make this answer choice the best choice. C. Meconium aspiration syndrome occurs when the baby passes meconium in utero and aspirates the meconium either in utero or at the time of delivery. Since it was noted that the amniotic fluid did not contain meconium, it is less likely that Adam is suffering from meconium aspiration syndrome, even though he has symptoms of respiratory distress that can often be seen in meconium aspiration syndrome. D. Hypothermia can cause tachypnea of the newborn, especially premature newborns. However, hypothermia is less likely in Adam given his fever of 100.5 F. E. Pneumothorax is less likely in this case because of Adam's normal chest x-ray, but is important to consider in a tachypneic newborn. The presence of Adam's fever also makes pneumothorax less likely to be the sole cause of his symptoms.

You are seeing a 36-month-old boy for his well-child visit. His parents are anxious about ensuring that his development is appropriate. He passed a hearing screen at birth and, other than a few colds, has been generally healthy. He has never been hospitalized or had any serious illness. He is able to run well, walk up stairs, and walk slowly down stairs. He uses more words than the parents are able to count, but can use them only in short, two or three-word sentences. His speech is understandable. He can draw a circle, but not a cross. Neurologic examination shows normal cranial nerves, normal sensitivity, normal motor reflexes, and no Babinski sign. Which of the following is the most appropriate next step in the management of this patient? A. Perform a brain-stem auditory evoked potential hearing screen B. Perform a screening exam for autism C. Reassure the parents that the boy's development appears normal D. Refer the child to a developmental specialist for comprehensive evaluation E. Refer the child to a specialist for evaluation of his delayed motor development

> C has been selected by the expert. A. A brain-stem auditory evoked potential hearing test (BAER) may be indicated in infants who fail to meet language milestones if they cannot cooperate with other more comprehensive testing. A 36-month-old should be able to cooperate with behavioral audiometry, so a BAER is not indicated. In addition, this child has no evidence of language delay and does not require referral at this point. B. Autism is an increasingly diagnosed cause of developmental delay, but this child is not delayed and no mention is given of any autistic features, such as a lack of symbolic play, repetitive movements, or poor sociability. C. The developmental milestones mentioned in the vignette are within the range of normal for a 36-month-old child. In the absence of any other evidence of significant impairment, there is no indication for referral at this point. D. If there are reasons for concern on developmental screening tests, a referral may be indicated. However, the developmental milestones mentioned in the vignette are within the range of normal for a 36-month-old child. E. This child's motor milestones are not delayed, and no referral is indicated.

A 7-year-old boy with a past medical history of headaches presents with increased frequency and severity of headaches along with new onset vomiting. When the patient was walking into the room, he had a wide stance and nearly tripped twice. Which of the following is the most appropriate next step? A. Daily headache diary B. Computed tomography C. Magnetic resonance imaging D. Lumbar puncture E. Intraventricular pressure monitoring

> C has been selected by the expert. A. A daily headache diary is a valuable clinical tool. It should include: characterization, duration, location, associated symptoms, recent activity, aggravating, and alleviating factors. This particular case includes multiple concerning symptoms and requires a more in-depth workup. B. CT imaging is a faster and more convenient than an MRI and is very valuable in ruling out intracranial hemorrhage. However, it does not visualize the posterior fossa as well as MRI and would not be the modality of choice if MRI is available. C. MRI is more expensive and less readily available than CT imaging. It also frequently requires sedation in pediatric patients. However, it provides the best detail of the posterior fossa, which is the most common location of pediatric brain tumors. D. Lumbar puncture can be very helpful in the diagnosis of meningitis, but would not be appropriate in this case. This patient may have increased intracranial pressure, and lumbar puncture is contraindicated in patients with signs of increased intracranial pressure because it may lead to brain herniation. E. Intraventricular pressure monitoring involves the placement of an intraventricular catheter, which measures pressure inside the skull and sends measurements to a recording device. This option may reveal increased intracranial pressure, but would not be helpful in the diagnosis. A brain MRI would be more appropriate.

An 18-month-old presents with yellow and poorly mobile tympanic membranes. Four months prior he presented then with several days of nasal congestion, cough, decreased eating and ear tugging. His exam then revealed a red, nonmobile tympanic membrane and he was treated with amoxicillin. Based on the history and physical exam, what is the most likely diagnosis now? A. Mastoiditis B. Acute otitis media C. Otitis media with effusion D. Otitis externa E. Viral encephalitis

> C has been selected by the expert. A. Although mastoiditis presents with tympanic membrane changes, the signs of bilateral yellow and poorly mobile tympanic membranes are more suggestive of otitis media with effusion. B. The course of development of this infection over four months makes this diagnosis unlikely. C. The earlier diagnosis of acute otitis media together with current findings of bilateral yellow and poorly mobile tympanic membranes on physical exam make this the most likely diagnosis. D. Although otitis externa presents with otalgia, the sings of bilateral yellow and poorly mobile tympanic membranes are more suggestive of otitis media with effusion. E. Viral encephalitis typically causes neurologic dysfunction with signs that include fever, headache, nausea, vomiting, photophobia, altered level of consciousness, seizures, and possibly focal neurologic signs. This child's presentation is much more consistent with otitis media with effusion.

You are called to the delivery of an infant boy experiencing fetal distress. After a vaginal delivery with vacuum assist, the infant cries spontaneously but remains acrocyanotic, despite supplemental oxygen delivered by mask. The neonate is hypotonic and moves his extremities only in response to noxious stimuli. Physical exam reveals an open mouth with a protruding tongue, upslanting palpebral fissures, low-set ears, and a transverse crease across both palms. You immediately recognize this syndrome, and your attending asks you what is the most common cardiac defect in these patients? A. Aortic insufficiency B. Coarctation of the aorta C. Endocardial cushion defects D. Patent ductus arteriosus E. Conduction pathway defects

> C has been selected by the expert. A. Aortic insufficiency is associated with Marfan syndrome. Aortic dissections are also seen in this condition and can be lethal. B. Coarctation of the aorta is seen in 35% of patients with Turner syndrome. C. Endocardial cushion defects. This patient has physical signs of Down syndrome. Approximately 50% of children with Down syndrome are born with endocardial cushion defects, such as ventricular septal defect, atrial septal defect, or complete atrioventricular canal defect. D. Patent ductus arteriosus is usually seen in infants with prematurity, sepsis, metabolic acidosis, or pulmonary defects not associated with congenital syndromes. E. Conduction pathway defects are not characteristic of patients with Down syndrome but may be seen in infants born to mothers with lupus (heart block).

Sammy is a healthy male child brought into your office by his mother for a well-child examination. As part of your evaluation you assess his developmental milestones. He is able to run, make a tower of 2 cubes, has 6 words in his vocabulary, and can remove his own garments. What would you estimate Sammy's age to be based upon his developmental milestones? A. 12 months B. 15 months C. 18 months D. 30 months E. 36 months

> C has been selected by the expert. A. At age one year, gross motor skills include pulling to stand, standing alone, and perhaps first steps. Fine motor skills including putting a block in a cup and banging 2 cubes held in hands. At this age a child should be able to imitate vocalizations/sounds and babble. The majority of children this age will know 1 or 2 words in addition to "mama" and "dada." Social-emotional milestones at age one year are waving bye-bye and playing pat-a-cake. Running, building towers of blocks, removing clothing, and a 6-word vocabulary are more advanced skills than a 12-month-old would be expected to have. B. At 15 months of age, a child should be able to stoop and recover and walk well, put a block in a cup, have a vocabulary of a few words, wave bye-bye, and drink from a cup. Running, building towers of blocks, removing clothing, and a 6-word vocabulary are more advanced skills than a 15-month-old would be expected to have. C. At 18 months, a child should be able to walk backward, and 50-90% of children can run at this stage. An 18-month-old should be able to scribble, build a tower of 2 cubes, have 3-6 words in her or his vocabulary, and be able to help in the house and remove garments. D. At 2 ½ years of age, kids can jump up and throw a ball overhand. They can build a tower of 6-8 cubes, point to 6 body parts, name 1 picture, put on clothing, and wash and dry their hands. Sammy is only able to build a tower of 2 cubes, can remove his clothing but does not yet put clothing on, and his vocabulary is limited to 6 words-leading us to believe he is not 2 ½ years old. E. At age 3, children can balance on each foot for 1 second, wiggle their thumbs, name 4 pictures, name 1 color, name a friend, and brush their teeth with help. Sammy's vocabulary is only 6 words, he is not able to name a friend, he is only able to stack 2 cubes, and he has just starting running, but is unable to balance on each foot for 1 second.

A 6-month-old male comes to clinic with a chief complaint of several weeks of vomiting after large feedings. The vomiting has become blood-streaked, which is when the mom became concerned and brought him in. The baby's PO intake has been down and he has been losing weight. Abdominal exam is normal, with no masses palpated. What is the most likely diagnosis? A. Pyloric stenosis B. Gastroenteritis C. GERD D. Volvulus E. Intussusception

> C has been selected by the expert. A. Choice A is incorrect because pyloric stenosis is characterized by a pattern of forceful, projectile, non-bilious vomiting, usually in younger infants. Infants are usually hungry and nurse avidly. An oval mass, 5-15 mm in longest dimension can be felt on deep palpation in the right upper abdomen, especially after vomiting. B. Choice B is incorrect. Large watery stools, which this patient did not have, are the hallmark of infectious gastroenteritis. Also, gastroenteritis is a more acute disease, while this patient is having more chronic symptoms. C. Choice C is correct because regurgitation/spitting up may be difficult to distinguish from true vomiting. Infants who reflux with overfeeding may sometimes have forceful vomiting. Severe esophagitis may result in blood-streaked emesis. Pain from reflux or esophagitis may lead to feeding aversion when gastroesophageal reflux is severe. D. Choice D is incorrect because in volvulus, blood may be seen in the stool but not typically in the vomitus. Bowel ischemia from volvulus can cause significant abdominal pain. E. Choice E is incorrect because in intussusception we might see the classic "currant jelly" stools. The abdominal exam in children with intussusception often shows the presence of a sausage-like mass due to the telescoped bowel.

A 9-year-old male presents to the ED in an ambulance after he was found unconscious at a local playground. In the ED he is arousable but extremely obtunded. He is able to minimally verbalize that his head hurts and his stomach feels uncomfortable. He states the pain is constant and non-radiating. He vomits clear liquid twice over the course of 30 minutes. Vital signs are as follows: T 37.6 C, P 66 bpm, BP 155/80 mm Hg, RR 18 bpm. You further notice that his breathing is irregular with brief episodes of apnea. On physical exam you are unable to reproduce the abdominal pain and there is no rebound tenderness or guarding. The rest of the physical exam is unremarkable. What is the most likely diagnosis? A. DKA B. Appendicitis C. Intracranial hemorrhage D. Gastroenteritis E. Small bowel obstruction

> C has been selected by the expert. A. Choice A is incorrect, because in a patient with DKA one would expect increased adrenergic tone leading to tachycardia, not inappropriate slowing of the HR. Secondly, the patient's breathing pattern is more consistent with Cheyne-Stokes respirations, not Kussmaul breathing. Kussmaul breathing is typically characterized by deep breaths that may be rapid, normal or slow in rate without periods of apnea, often associated with metabolic acidosis. Lastly, one would expect signs or symptoms consistent with dehydration such as polyuria, polydipsia, decreased skin turgor, or skin tenting. However, the altered mental status, vomiting, headache, and abdominal pain could be seen in DKA. B. Choice B is incorrect because the lack of fever, inability to reproduce the abdominal pain in the RLQ, and the severely altered mental status argue against the diagnosis. In appendicitis, children will often complain of a migratory pain that beings around the periumbilical region and migrates to the RLQ. Patients often complain of rebound tenderness and demonstrate guarding as well. C. This is the correct choice. Increased ICP can be secondary to epidural or subdural hemorrhage. It is possible the patient may have fallen while playing in the playground. Increased ICP can present as the classic Cushing's triad: hypertension, inappropriate slowing of the heart rate, and irregular respirations (Cheyne-Stokes respiration). A further complication of increased ICP is epigastric discomfort. This is caused by the elevated ICP causing vagal stimulation, resulting in the secretion of gastric acid. Lastly, the patient's headache and non-bilious vomiting can also be ascribed to the increased ICP. D. Choice D is incorrect because gastroenteritis usually presents with fever, colicky abdominal pain, and diarrhea. It would also be atypical for the patient's mental status to be so adversely affected by gastroenteritis. More than 95% of gastroenteritis hospitalizations occur in children younger than 5 years, with the peak incidence between 3 and 24 months of age. The incidence tends to peak in winter. There can be both viral and bacterial causes for gastroenteritis. Classically, viral gastroenteritis will present with diarrhea in which the stool lacks blood or mucus. Bacterial gastroenteritis often causes diarrhea with gross blood or mucus present in the stool. E. Choice E is incorrect because a small bowel obstruction usually presents with bilious vomiting, abdominal distention, inability to pass flatulence, and moderate-to-severe abdominal pain. The pain is often paroxysmal, coming and going in 4 to 5 minute intervals. The patient's lack of a fever, however, is consistent with a GI obstruction. The most common causes of a small bowel obstruction are adhesions from a previous surgery or a hernia.

A woman brings her 8-year-old son to the pediatrician after witnessing him stare blankly into the distance at dinner the previous week. He was unresponsive to her calling his name or any other stimuli, and it lasted for about 10 or 20 seconds. His teacher reports he does seem to daydream often in class but is able to keep up with schoolwork and excels in his studies. She doesn't note him being disruptive or impulsive in class. His mother is concerned about these blank stares and unresponsive episodes. Which of the following is the most likely diagnosis? A. Generalized tonic-clonic seizure B. Atonic seizure C. Absence seizure D. Simple partial seizure E. Complex partial seizure

> C has been selected by the expert. A. Generalized tonic-clonic seizure is the most common type of seizure seen in children. These seizures begin acutely with tonic (rigid) stiffening of all extremities and upward deviation of the eyes. The tonic phase is followed by clonic jerks of all the extremities. The patient then loses consciousness and may have urinary incontinence. The patient in this vignette did not have movement of his extremities and did not have urinary incontinence. B. Atonic seizure, also called drop seizures or drop attacks, consists of loss of motor tone, where patients have a brief lapse in muscle tone. These seizures can occur while walking, standing, or sitting, and have a characteristic head drop (the neck muscles releasing). These seizures last about fifteen seconds. The patient in the vignette did not have symptoms of an atonic seizure. C. The patient is having absence seizures. Absence seizures are characterized by loss of awareness of surroundings ("blank stare" or "in another world") and automatisms (e.g., eye-fluttering or lip-smacking). These patients do not lose consciousness or have loss of tone. Absence seizures should also be differentiated from ADHD, since children with ADHD also can be inattentive or seem to be daydreaming. However, since the patient in the vignette still does well in school and does not have other signs of ADHD, it is most likely an absence seizure. An EEG will confirm the diagnosis. D. Simple partial seizure is a seizure localized to a small region of the brain. Patients do not lose consciousness. Based on the region of the brain the seizure is occurring in, the patient will exhibit focal symptoms (e.g., altered hearing or smelling, labored speech, etc.). If patients have motor symptoms, it is usually localized to one extremity. E. Complex partial seizure is characterized by impairment of consciousness. These seizures are similar to simple partial seizures except patients lose consciousness.

Frank is 16-year-old male brought in by his mother who complains that her son "looks much younger than his age." She states that until about four years ago, she did not notice much difference between Frank and his friends. However, in the past two years, Frank has become the shortest person in his class. Frank's mother is concerned that he has a "hormone problem" and wants to know how she can tell if he has begun puberty. What is usually the first sign of puberty in a male? A. Growth of the penis B. Appearance of pubic hair C. Testicular enlargement D. Growth spurt E. First ejaculations

> C has been selected by the expert. A. Growth of the penis occurs with growth of the scrotum, and usually occurs around 13-14 years of age. It normally follows pubic hair appearance and precedes first ejaculations. B. Pubic hair appearance occurs around 12 years of age, and is usually the second sign of puberty following testicular enlargement. It precedes growth of the penis and scrotum. C. The first sign of puberty in a boy is testicular enlargement. The onset of puberty is quite variable, but usually occurs between 10 and 15 years for boys. It is rare for boys not to have begun puberty by the age of 16. To assess whether or not a male has entered puberty, one must know the order of the appearance of secondary sexual characteristics. D. A growth spurt is usually not appreciated until at least 14 years of age for most boys. It is one of the last secondary sexual characteristics to be noted, following first ejaculations. E. First ejaculations usually occur around 13-14 years of age. These follow growth of the penis and scrotum, and precede a growth spurt.

A 5-year-old female, previously healthy, presents with an erythematous, vesicular rash on the palms and soles and a high fever for several days. Upon examination, she is also found to have ulcers in her mouth. A few days later, the fever and rash resolve. What is the most likely pathogen? A. Herpes simplex virus 1 (HSV-1) B. HIV C. Enterovirus D. Human herpesvirus 6 (HHV-6) E. Group A strep

> C has been selected by the expert. A. HSV-1 causes gingivostomatitis and can sometimes be accompanied by fever and malaise, but lesions on the hands and feet would be uncommon. B. HIV infection can increase the risk of oral lesions secondary to infections by HSV-1 or Candida albicans, but again would be unlikely to cause lesions on the hands and feet. Furthermore, at this patient's age (5 years), she is unlikely to be HIV-positive unless it was vertically transmitted from her mother. C. This presentation is consistent with infection by cocksackie A, an enterovirus. Following an incubation period of three to five days, patients have fever, tender vesicles on their hands and feet, and oral ulcers. Sometimes the rash also occurs on the buttocks and the genitals. The infection resolves spontaneously within three days, and is spread from person to person via saliva, fluid from the vesicles, stool, or nasal discharge. D. HHV-6 is the virus that causes roseola, which manifests as fever followed by a macular or maculopapular rash, but this rash begins on the trunk, eventually spreading to the extremities, and does not cause oral lesions. E. Group A strep infection could cause fever and a rash with scarlet fever, but this rash is described as "sandpaperlike" with small papules, not vesicular, and is also not confined to the hands and feet.

A 5-year-old boy is noted to have a grade II systolic murmur and a widely split S2 murmur on cardiac exam. His vital signs are stable and he has been asymptomatic. Which of the following statement is accurate regarding this patient's presentation and likely condition? A. No further work-up for a presumed venous hum B. Chest x-ray, ECG, and echocardiogram would be indicated as next steps to work up a presumed ventricular septal defect C. This patient's murmur is caused by flow through the pulmonary outflow tract and should be evaluated D. The patient should be scheduled now for cardiac catheterization

> C has been selected by the expert. A. Incorrect. A benign venous hum would not be expected to present with a widely split second heart sound. B. Incorrect. The first steps in the evaluation of presumed structural heart disease would include a chest x-ray, ECG, and echocardiogram. However, the clinical presentation is not consistent with a ventricular septal defect, which usually presents with a holosystolic murmur and a second heart sound that splits variably with respiration. C. Correct. This patient's murmur is likely caused by an atrial septal defect, which causes flow of additional blood through the pulmonary outflow tract and should be evaluated. D. Cardiac catheterization might be indicated to better define this patient's anatomy, and the pressures within each cardiac chamber. However, it would not be indicated to schedule this before a preliminary work-up, including a chest x-ray, ECG, and echocardiogram.

A 45-day-old infant is brought in by his mother due to lethargy, constipation, and yellow skin color noted since birth. The mother and the baby moved to the U.S. from a foreign country that does not screen its newborns. The baby has been fed only formula since birth. Physical exam of the neonate reveals additional findings of large fontanelles, umbilical hernia, a large tongue, and abdominal distension. What is the next best step in diagnosis? A. Phototherapy B. Exchange transfusion C. TSH D. Head ultrasound E. Obtain a family history of jaundice to rule in or out a defect in bilirubin metabolism

> C has been selected by the expert. A. Phototherapy is the treatment for physiologic jaundice, which peaks at three to four days and resolves by the fourth or fifth day of life. Lethargy, macroglossia, and umbilical hernia are not known to be caused by or associated with physiologic jaundice. This constellation of physical exam findings is more consistent with untreated congenital hypothyroidism. B. This choice is incorrect because this is the management of infants with hyperbilirubinemia approaching levels of concern for kernicterus. C. This choice is correct because the constellation of baby's problems is best accounted for by untreated congenital hypothyroidism. Unfortunately, severe mental retardation is unavoidable at this point because this condition should have been treated since birth. In the U.S., it would have been detected on the newborn screen. D. This choice is incorrect as there is no indication of hydrocephalus or concern for bleeding. E. This choice is incorrect because-although defects in biliary metabolism such as Gilbert's syndrome, seen in 5% of the population, can cause harmless jaundice-this patient has many other findings in addition to jaundice. Of the answer choices given, only congenital hypothyroidism fully accounts for the entire constellation of findings.

A 12-year-old male presents to the ED with complaints of anorexia, weight loss, and persistent cough, with nocturnal coughing fits that have been waking him from sleep for the past three weeks. He denies fever, chills, myalgia, sore throat, or rhinorrhea. The patient presented to his primary care physician one week prior with the same complaint, and was treated with amoxicillin and bronchodilator therapy. His chest x-ray was negative for infiltrates at that visit. The patient's symptoms did not improve with this regimen. The cough became more frequent, sometimes causing emesis. Which of the following is the most likely diagnosis? A. Reactive airway disease B. Infection with Bordetella pertussis in the catarrhal stage C. Infection with Bordetella pertussis in the paroxysmal stage D. Atypical pneumonia due to Mycoplasma pneumoniae E. Laryngotracheobronchitis

> C has been selected by the expert. A. Reactive airway disease would most likely have improved with bronchodilator therapy. In addition, you would not expect to see anorexia or weight loss with reactive airway disease. B. The catarrhal stage of pertussis lasts one to two weeks and is often indistinguishable from URI. In this patient the presenting symptoms have been an ongoing problem for more than three weeks. C. The paroxysmal stage of pertussis lasts four to six weeks and is characterized by repetitive, forceful coughing episodes, followed by massive inspiratory effort. This massive inspiratory effort is what results in the characteristic "whoop"-sounding cough. This is consistent with the patient's presentation and duration of illness. The forceful coughing fits in pertussis can even lead to conjunctival hemorrhages and pneumothoraces from the increased intrathoracic and intracranial pressures from Valsalva. The antimicrobial agents of choice for treatment of pertussis are azithromycin, clarithromycin, and erythromycin. Antibiotics given in the paroxysmal phase will reduce communicability but will not alter the clinical course. D. Mycoplasma pneumonia would be expected to be associated with fevers and findings on chest x-ray and lung exam. E. Croup (laryngotracheobronchitis) presents with difficulty breathing and a "seal bark" cough and usually lasts a week or less. Croup also is usually associated with fever.

A 10-year-old male comes to the clinic with a chief complaint of progressive cough for two weeks that began gradually. His cough is described as productive and wet with whitish sputum. His mother denies throat pain, vomiting, and diarrhea in his review of systems. His mother reports that he has been febrile up to 101.5°F daily. She thinks he is fatigued and has not eaten well in the past week. On exam, there is air passage throughout all lung fields, with crackles in the lower right lung field, but no other abnormal sounds. What would you likely find in your workup? A. Response to inhaled beta-agonist B. Hyperinflation in one lung field C. Alevolar consolidation in the RLL D. Positive PCR for pertussis E. Fluffy bilateral infiltrates and a large heart on chest x-ray

> C has been selected by the expert. A. Response to an inhaled beta agonist is a good test for RAD or asthma. Asthma is diagnosed clinically, usually in a school-aged child, with a history of recurrent wheezing. Associated findings might atopic stigmata, such as allergic rhinitis, food allergy, and atopic dermatitis. Without details in the history, asthma is a less likely diagnosis in this setting, and in the absence of wheezing, asthma is even less likely. B. This would be consistent with a foreign body aspiration which could produce cough and fever (if bacterial superinfection occurs). This diagnosis is usually considered in younger children. Constitutional symptoms (fatigue, decreased eating) make pneumonia a better diagnosis in this clinical setting. C. Pneumonia is the most likely cause for his symptoms and a chest x-ray would be a great confirmation of your suspected diagnosis. Eliciting a complete history might reveal history of an upper respiratory infection. Localization of crackles (discontinuous inspiratory sounds) to one lobe makes pneumonia more likely. D. Pertussis can produce a lengthy cough illness, but is not associated with fever or lung findings. E. Pulmonary edema due to CHF is a symmetrical process and less likely to present with a unilateral lung finding. Pulmonary edema should be suspected with crackles, but this clinical setting leaves pulmonary edema low on our differential.

A 39-year-old G2P1 woman with a pre-pubertal 10-year-old boy with intellectual disability comes to the clinic for information on prenatal screening. The 10-year-old boy was born with large ears and long face but no other congenital malformations. The mother is worried that she will have a second child with similar problems. If she were to have a second male child with developmental impairment, what would be the most likely reason if maternal serum testing and fetal ultrasound were both normal? A. Down syndrome B. Trisomy 13 C. Fragile X syndrome D. Turner syndrome E. Klinefelter syndrome

> C has been selected by the expert. A. Risk of Down syndrome is increased with advance maternal age, but prenatal screening would show abnormal maternal serum values with or without abnormal fetal ultrasound findings. B. Patients with Trisomy 13 would not have large ears and long face, but would present with severe impairments at birth. C. Fragile X syndrome is the most common familiar cause of developmental impairment and is due to an abnormal number of trinucleotide repeats. These children present with large ears, long face and mandible and, after puberty, large testicles. D. Turner syndrome is not correct given the child's male sex. Individuals with Turner syndrome have normal IQ. E. Patients with Klinefelter syndrome have IQs that vary around the low-normal range, but typically have no stereotypical physical features other than low muscular bulk due to decreased testosterone levels.

A 3-year-old male presents with fever to 103 F for the past week, injected eyes, and a refusal to walk for the past two days. On physical exam, you note conjunctival injection without pus or exudates bilaterally, prominent papillae of his tongue with redness as well as redness of his hands, and feet. He also has a new non-diffuse maculopapular rash on his torso that gets worse with fever. On examination of the swollen extremities, you are unable to elicit any tenderness or effusions in any joints. Which of the following is the most likely diagnosis? A. Rocky Mountain Spotted Fever (RMSF) B. Bone or joint infection C. Kawasaki disease (KD) D. Scarlet fever E. Systemic onset juvenile idiopathic arthritis

> C has been selected by the expert. A. Rocky Mountain Spotted Fever (RMSF) is a tick-borne disease caused by Rickettsia rickettsii. This tick is commonly found in southeastern parts of the U.S., and patients will often come from or have a history of travel to that region. The disease is characterized by headache, fever, myalgia, and a centrally progressing petechial rash originating on the wrists and ankles. The maculopapular rash, and constellation of other symptoms, as well as lack of any recent travel history, makes this diagnosis less likely in this patient. B. Bone or joint infection should be on the differential given the patient's refusal to walk, as up to 80% of these infections are in the lower extremities. The fever associated with septic arthritis and osteomyelitis typically are not as elevated as the one presented in this case, and the lack of localized symptoms of warmth and tenderness associated with the lower extremity erythema and swelling make this diagnosis somewhat less likely in this patient. C. Kawasaki disease (KD) is one of the most common vasculitides of childhood. For diagnosis, in addition to fever of > 5 days, patient must meet four of the following criteria: rash, conjunctivitis, unilateral cervical lymphadenopathy, changes in oral mucosa, or extremity changes (redness/swelling). Our patient does not have lymphadenopathy, but often this is the least common finding in KD. If children have fever with fewer than four of the five clinical findings, they can have "incomplete KD" if they meet certain laboratory criteria. D. Scarlet fever is caused by erythrogenic toxin produced by Streptococcus pyogenes. Symptoms can include sore throat, fever, "strawberry tongue" and a blanching, erythematous rash with desquamation of the affected areas about six to seven days later as the rash begins to disappear. While our patient does have a "strawberry tongue," fever, and rash, the description of the rash and other physical exam findings are more consistent with KD than with scarlet fever. E. Systemic onset juvenile idiopathic arthritis, also known as Still's disease, is a subset of JIA describing patients with intermittent rash, fever and arthritis. While our patient does present with rash and fever, as well as refusal to walk (potentially a sign of arthritis), systemic onset JIA tends to present with a history of spiking fevers and "salmon" rash occurring when the child is febrile, and disappearing as the fever fades. This is inconsistent with the description of the findings seen in our patient, who does not demonstrate tenderness or effusion in any joint.

Rosy is an 18-month-old previously healthy baby girl who presents to clinic with congestion for three days. Today, her vitals are: T 101.2°F, BP 100/60 mmHg, P 80 bpm, RR 28 bpm. On physical exam, Rosy has clear mucus coming from both nostrils. Both turbinates show erythema. Her oropharynx is erythematous. No crackles or wheezing are heard. Mom reports that acetaminophen aids in bringing down the fever temporarily; however, the fever returns in a few hours. Mom is concerned for possible pneumonia since she was recently was given antibiotics for bronchitis. Her immunizations are up to date. Which of the following is most likely responsible for Rosy's symptoms? A. Strep pnuemoniae B. Group A Strep C. Rhinovirus D. Hemophilus Influenzae type B E. Pertussis

> C has been selected by the expert. A. Rosy's symptoms do not fit a diagnosis of pneumonia, as she does not have significant fever, increased respiratory rate, crackles, or rales. B. Group A Strep is rarely a cause of pharyngitis in young children and notably does not cause the feared sequelae (rheumatic fever) in those < 3 years. C. Rhinovirus causes the common cold and is the most reasonable diagnosis. Rhinovirus is a very common cause of congestion and other cold-like symptoms. Rosy presents with slightly elevated temperature, slight tachypnea, and inflamed turbinates and oral mucosa. Her symptoms all correlate with the common cold. D. Hemophilus Influenzae type B causes pneumonia and epiglottitis. Rosy does not have the typical symptoms of epiglottitis that include difficulty breathing, high fever and drooling. With the advent of vaccinations, Hib infections have decreased significantly. E. The catarrhal phase of pertussis can be indistinguishable from the common cold, but quickly develops into the paroxysmal phase. The paroxysmal phase is characterized by coughing fits and post-tussive emesis, which again Rosy does not have. It typically does not have associated fever.

A two-month-old female presents to clinic for a well-baby checkup. Mom has been happy because the "baby rarely cries and sleeps all the time." On exam, the baby has yellowing of the skin, decreased activity, appears to have decreased tone, and a large anterior fontanel. What is the most likely diagnosis? A. Sepsis B. Congenital adrenal hyperplasia C. Congenital hypothyroidism D. Shaken baby syndrome E. Neonatal lupus

> C has been selected by the expert. A. Sepsis is a blood infection of the infant. Signs and symptoms may include body temperature change, changes in respiration, increased or decreased heart rate, reduced movement, reduced feeding, low blood sugar, seizures, and jaundice. B. Infants with congenital adrenal hyperplasia often have abnormal genitalia (females), poor feeding, vomiting, dehydration, and electrolyte changes. C. Congenital hypothyroidism may not be clinically evident until 6 weeks of age due to circulating maternal thyroid hormone transmitted from the placenta. Signs and symptoms of congenital hypothyroidism include feeding problems, large fontanels, hypotonia, large tongue, coarse cry, and frequently an umbilical hernia. Congenital hypothyroidism should be picked up on routine neonatal screening. D. Shaken baby syndrome may result in significant head trauma, including subdural hemorrhage, hypoxic/ischemic brain injury, and retinal hemorrhage. E. Neonatal lupus should be considered when the infant's mother has lupus. Cutaneous findings may be present at birth or may develop within the first 2 to 5 months of life. These findings include erythematous plaques, telangiectasias, or atrophic lesions. Affected infants may have cardiac abnormalities or conduction deficits. Hematologic disturbances may occur within the first 2 weeks of life.

states that she and the patient's grandmother reached menarche at 13 years of age. The patient is concerned she is behind her friends in terms of development. She is doing well in school and has not had developmental problems in the past. On physical examination, her breasts are elevated without a secondary mound, and curly, coarse pubic hair is present on the labia majora in a triangular shape but does not reach the mons pubis. What Tanner stage would you assign this patient? A. Tanner Stage I B. Tanner Stage II C. Tanner Stage III D. Tanner Stage IV E. Tanner Stage V

> C has been selected by the expert. A. Tanner Stage I breast development consists of no glandular tissue and is prepubertal. Tanner Stage I consists of no pubic hair at all. This is usually around age 10 or younger. B. Tanner Stage II breast development occurs when breast buds form and the areola begins to widen. A small amount of long, downy hair with slight pigmentation appears on the labia majora. This patient's elevated breast buds and pubic hair distribution puts her beyond Tanner Stage II. C. The patient in the vignette is at Tanner Stage III of development. Her breast buds are elevated but do not have the secondary mound characteristic of Tanner Stage IV. Her pubic hair distribution extends more laterally than Stage II but is not adult-like in hair quality and does not extend onto the mons pubis. D. In Tanner Stage IV, breasts are increased in size and elevation and the areola and papilla form a secondary mound that projects from the contour of the rest of the breast, and the pubic hair extends across the mons pubis and spares the medial thighs. E. In Tanner Stage V, breasts reach their adult size and the areola returns to the contour of the surrounding breast while the central papilla remains projecting and the pubic hair extends to the medial surface of the thighs.

A 9-year-old male presents to your clinic with discoloration under his eyes, persistent cough, and skin rashes. He is found to have wheezing on physical exam and increased lung volume bilaterally on chest x-ray. He has struggled with these complaints over the past three years but recently his symptoms have gotten worse, affecting him every other day. He is afebrile. He is found to have wheezing on physical exam and increased lung volume bilaterally on chest x-ray. What would be the most appropriate treatment for him? A. Oral antibiotics B. Short-acting beta agonist PRN C. Short-acting beta agonist PRN with low-dose inhaled corticosteroid D. Short-acting beta agonist PRN with medium-dose inhaled corticosteroid E. Long-acting beta agonist

> C has been selected by the expert. A. The patient's presentation is more consistent with asthma than an infection. The patient has had these complaints for the last few years. His skin rashes and lower eyelid darkening are consistent with allergic processes (atopy), which are associated with asthma. Furthermore, increased lung volumes bilaterally and persistent cough without fever also suggest asthma, thus antibiotics would not be appropriate. B. Cough and wheezing that occur intermittently (< 2 days/week) are consistent with intermittent asthma, which is treated with short-acting beta agonist PRN. C. Persistent cough and wheezing that affect the patient every other day (3-4 days with symptoms/week) are consistent with mild persistent asthma, which is appropriately treated with short-acting beta agonist PRN and low dose inhaled corticosteroid. The swelling under the eyes (allergic "shiners") and skin rash are other signs of atopy, as mentioned above. D. Short-acting beta agonist PRN with medium dose inhaled corticosteroid is the preferred treatment for moderate or severe persistent asthma, which corresponds to daily symptoms or symptoms throughout the day. E. The use of a long-acting beta agonist is reserved for severe persistent asthma, which corresponds to symptoms throughout the day.

John is a 17-year-old presenting today for a pre-participation physical exam. During the interview, he reports a low-grade fever, malaise, and headache for one week. In the past few days, his fever has gotten worse and he complains of a sore throat. He denies cough or chest pain. On physical examination, he is found to have a temperature of 101.3° F, and cervical lymphadenopathy and oropharyngeal erythema with exudate are noted. His participation would be most likely affected by which of the following tests? A. Chest x-ray B. CT head/neck C. EBV serologies D. Throat culture E. No further workup

> C has been selected by the expert. A. This choice is incorrect because there is no evidence of lower respiratory tract signs or symptoms. B. This choice is incorrect because the additional exposure to radiation is not warranted given the availability of other higher priority tests. C. This choice is correct because the patient's symptoms are suggestive of infectious mononucleosis. These include complaints of low-grade fever and malaise and findings of cervical lymphadenopathy and pharyngeal exudate. If testing is positive, the patient should be restricted from strenuous activity or contact sports during his illness due to the risk of splenic rupture. D. An antigen test (and culture, if negative) should be performed for streptococcal pharyngitis, but this clinical picture is more consistent with EBV, and mononucleosis is more likely to preclude participation in sports. After 24 hours of antimicrobials, the patient with streptococcal pharyngitis is no longer contagious. E. This choice is incorrect because the patient's symptoms and findings warrant further evaluation.

A 16-year-old homeless female presents with low-grade fever and abdominal pain. The patient reports recent unprotected sex. Abdominal examination reveals tenderness to palpation in the lower abdominal region, but no masses are appreciated. Pelvic examination reveals whitish cervical discharge and cervical motion tenderness. The discharge is sent for culture, and a pregnancy test is negative. What is the next best step in management? A. Pelvic ultrasound B. Begin oral antibiotics antibiotics and treat her partner C. Arrange for hospitalization D. Abdominal CT E. Surgical consult

> C has been selected by the expert. A. This choice is incorrect. Although pelvic ultrasound is helpful in identifying pelvic disease processes, this patient has signs and symptoms of pelvic inflammatory disease (PID). Treatment of PID can be initiated without further diagnostic workup. B. This choice is incorrect. Although most patients with PID can be discharged home with home antibiotics, this patient is homeless and is therefore at higher risk for failure to complete a coures of antibiotics. C. This choice is correct. This patient has signs and symptoms of PID. Cervical discharge should be tested for gonorrhea and chlamydia and sent for culture. As she is a homeless patient, she is at high risk for failure to complete her antibiotic course. Given the deleterious sequelae of incompletely treated PID, she should be hospitalized in order to ensure a full course of treatment. D. This choice is incorrect. Abdominal CT is not helpful in diagnosing PID. An argument could be made for an ultrasound if ovarian torsions were suspected, or if her symptoms were refractory to treatment, suggesting a tubo-ovarian abscess that could be detected by ultrasound. E. This choice is incorrect. Although the patient exhibits abdominal pain, further workup is required before calling a surgical consult. As the pregnancy test is negative, a ruptured ectopic pregnancy is not an acute concern at this time.

A 2-year-old girl presents to the urgent care clinic with a 7-day history of high fever to 38.5 C, a maculopapular rash that began on the palms and soles of her feet, red eyes without discharge, and unilateral cervical adenopathy. What other symptom/sign might you discover on further history and exam? A. Tonsillar exudates B. Headache C. Erythematous and edematous feet D. White spots on buccal mucosa E. Dysuria

> C has been selected by the expert. A. Tonsillar exudates would be present in strep pharyngitis or tonsillitis. Given the prolonged fever, rash, lymph node involvement, and conjunctivitis, the disease process is more widespread than a simple tonsillitis. You should recognize this constellation of symptoms as Kawasaki disease. B. Headache would be present in Rocky Mountain Spotted Fever. This seems reasonable, given the fever and rash that began on the palms and soles. However, the other findings suggest Kawasaki disease, so this is not the best answer. C. The constellation of symptoms described suggests Kawasaki disease. The other two classic signs not mentioned are erythematous tongue ("strawberry tongue"), and erythema/edema of the extremities, which is the best answer here. D. White spots on the buccal mucosa are also known as Koplik spots, which are pathognomonic for measles. The fever and conjunctivitis could be measles, but there is no cough or coryza. In addition, the rash typically starts at the head and moves downward, rather than starting on the hands and feet. E. Although children with Kawasaki disease can have pyuria, it is not associated with dysuria, a symptom of a UTI, which would be highly unlikely given the other signs.

Devin is a 2-year-old African-American boy with sickle cell disease. Today his parents brought him to the ED because he was not feeling well. His mother reports that he has been very tired for the past week. Vitals show a temperature of 39°C, BP of 120/75 mmHg, RR of 24 bpm, and HR of 104 bpm. On physical examination he is ill appearing. His conjunctivae appear pale, and his sclerae are anicteric. Lungs are clear to auscultation. His abdominal exam is benign (non-tender, non-distended, with no organomegaly). His extremities are non-tender upon palpation. His nailbeds appear pale. Stat CBC reveals Hgb: 4.5 g/dL, Hct 15%, WBC 1800, and platelets 88,000. Mother is concerned as she has never seen him so ill before. What is the most likely cause of his new symptoms? A. Vaso-occlusive crisis B. Splenic sequestration C. Aplastic anemia D. Hemolytic crisis E. Chest crisis

> C has been selected by the expert. A. Vaso-occlusive crisis is a common acute concern in patients with sickle cell disease. This patient, however, does not complain of any pain. Vaso-occlusive crisis would likely present with pain, and is not usually associated with a decrease in all cell lines as seen in this child's CBC. B. Splenic sequestration could potentially cause thrombocytopenia and anemia in a sickle-cell patient; however, there is no organomegaly noted on exam, and it does not explain his fever. C. Parvovirus B19 causes aplastic anemia, especially in children with shortened RBC half-life. A mild anemia associated with parvovirus often goes unnoticed in healthy children. However, in patients with decreased RBC survival, such as sickle-cell patients, anemia can develop very rapidly and severely, along with decreases in the other cell lines (resulting in leukopenia and thrombocytopenia). In the above presentation, the presence of fever in a child with sickle cell disease should also raise concern for significant bacterial infection from encapsulated organisms like pneumococcus. This patient should have a blood culture and be started on empiric antimicrobial therapy pending screen labs and cultures. D. Hemolytic crisis is unlikely as the patient's sclera are anicteric, and thus he is probably not hemolyzing significantly. Hemolysis would also not explain his leukopenia or thrombocytopenia. E. Chest crisis should be considered in all sickle cell patients who present with chest pain, increased work of breathing, or infiltrates on chest x-ray, which may be consistent with either infection or infarction. The patient had no pulmonary symptoms, and the CBC findings are not consistent with chest crisis.

A 6-week-old infant girl whose family recently immigrated from Mexico is brought to clinic for "excessive sleepiness." The mother states the infant is not easily aroused for feedings and is not as active as she was previously. She is also concerned about her daughter's large "outtie" belly button. On exam, the patient is afebrile and jaundiced, with a puffy myxedematous face. The fontanels are large but flat. There is a large umbilical hernia. When asked about the results of a newborn screening exam, mom states that the screening was never performed. What would be an expected abnormal lab value(s) associated with her condition? A. Low sodium, high potassium B. Glucose < 40 mg/dL C. High WBC with bandemia D. High TSH, low T4

> D has been selected by the expert. A. A low sodium and high potassium in a lethargic infant would suggest the diagnosis of congenital adrenal hyperplasia (CAH), characterized by a decreased production of cortisol and aldosterone. Low aldosterone results in decreased stimulation of the H/K exchange in the collecting duct, hence loss of sodium, retention of potassium, and dehydration. In combination with low cortisol levels, patients in adrenal crisis may progress to shock, and death is not treated. The usual age of presentation is 1 to 2 weeks of age. Initial laboratory studies would include serum electrolytes, renin, cortisol and cortisol precursors, androgens, and glucose levels. Note that low cortisol will also impact gluconeogenesis and glycogenolysis. This diagnosis would be less likely in a non-viralized female. B. Symptoms of hypoglycemia in the neonate may include jitteriness, tremors, hypotonia, poor feeding and seizures. Management includes STAT glucose levels and intervention with parenteral or oral glucose, as indicated. The work up will include laboratory studies to rule out hyperinsulinism (IDM, insulinoma, prematurity), increased metabolic demand (polycythemia, sepsis, asphyxia), and inborn errors of metabolism (galactosemia, glycogen storage diseases, maple syrup urine disease). This patient did not present with jitteriness, tremors, or seizures. C. Sepsis should always be considered in lethargic neonates. While septic infants may present with fever or hypothermia, they may also be afebrile. This child did not appear to be acutely ill or toxic in appearance. D. Congenital hypothyroidism may present with poor feeding, constipation, jaundice (longer and more persistent than physiologic jaundice of newborn), mottled skin, large fontanels, hypotonia, hypothermia and an umbilical hernia. Later findings include a hoarse cry, macroglossia, and myxedematous facies. Patients usually remain asymptomatic until after 6 weeks of age, as maternal thyroid hormones may still be in younger infants. Patients with primary hypothyroidism will have high TSH and low T4 levels. The most common cause of primary hypothyroidism will be aplasia or hypoplasia of the thyroid gland, and-much less commonly-inborn errors of metabolism. Secondary or tertiary hypothyroidism (HPA dysfunction) will have both low TSH and low T4, and are relatively rare causes of hypothyroidism in infants.

The parents of 5-month-old Tiffany are concerned about Tiffany's decreasing oral intake over the past 4 days. They report that she has been sleeping more but seems to tire out when feeding; in fact, mom's breasts have become quite engorged and she needs to pump to relieve the pressure. In addition to the sleepiness and poor feeding they report that she has not had a bowel movement in 3 days. She has no fever or respiratory symptoms. You note a weak cry on your exam, and a floppy baby when you try to sit her up. What additional finding are you likely to find on your exam? A. Vesicular rash on her scalp B. Large tongue and umbilical hernia C. III/VI systolic murmur D. Absent deep tendon reflexes E. Cataracts and hepatosplenomegaly

> D has been selected by the expert. A. A vesicular rash may be seen in neonatal HSV infection, which can be a cause of encephalitis. It is less likely in this older age group and would most likely present with fever and possibly seizure. B. Although congenital hypothyroidism can present with lethargy, constipation, and poor feeding, the infant would be less likely to present with these symptoms as late as 5 months of age. C. Congenital heart disease may present with poor feeding, but a large VSD would likely present earlier and would not be associated with constipation and hypotonia. D. This infant likely has infant botulism which usually presents in the first year of life with hypotonia, lethargy, constipation, weak cry and can eventually lead to respiratory failure. These infants will have absent DTRs. E. An inborn error of metabolism can present with lethargy and poor feeding-and hepatosplenomegaly and eye findings may also be present-but this infant presented more acutely and at an older age than would be expected for a metabolic disorder.

A 10-day-old boy is brought to the ED by his mother because of "fever." Mom describes that the baby has been "sleepy" and feeding less vigorously than in the previous two days. She believes his urine output has also decreased. His birth history is notable for prolonged membrane rupture (about 32 hours), and maternal fever at the time of delivery. Prenatal and neonatal ultrasound revealed bilateral hydronephrosis. On exam, the infant is sleepy with a temperature of 38.5 C. A blood sample is sent for CBC, BMP, and culture. Attempts are made to obtain CSF and urine for analysis and culture, but only very small volumes of these fluids are obtained. Volume resuscitation is begun. Chest x-ray is performed with indeterminate results. What is the most appropriate next step? A. Send samples for gram stains and begin parenteral empiric antibiotic treatment B. Send the urine for urinalysis and the CSF for cell count, glucose and protein and begin parenteral antibiotic therapy C. Admit for observation and continue supportive care D. Send samples for culture and begin parenteral antiobiotic treatment E. Attempt to obtain larger samples. Antibiotics should not be started until all needed results are pending.

> D has been selected by the expert. A. Although sending samples for gram stain may give an indication of whether an infection is present, it will not give the same degree of information as would a culture with sensitivities. B. Urinalysis and CSF profiles may help us make the diagnosis, but if positive in the absence of cultures, will commit us to a prolonged course of broad-spectrum and non-specific therapy. C. Delay of therapy would not be indicated. In an infant younger than one month, fever with any suspicion of sepsis, whatever the source, requires immediate evaluation and initiation of antibiotic treatment. D. Given the presentation of fever in a neonate who presents with sleepiness and poor feeding, samples should be sent for culture and the baby started on empiric antimicrobial therapy. This infant is likely to have a urinary tract infection, and urosepsis is certainly a possibility, especially given his known urinary tract anoamlies. We have no way of ruling out meningitis from this presentation, so antibiotics should be initiated at meningitic dosing. In an infant younger than one month, fever with any suspicion of sepsis, whatever the source, requires immediate evaluation and initiation of antibiotic treatment. Because infants at this age have immature immune systems, they do not localize infections as well as older children. An infection of the urinary tract may lead to bacteremia, which in turn may lead to CNS infection. Only cultures will give us the information required to determine the appropriate type length of antimicrobial therapy. E. Given the consequences of significant bacterial infection in an infant this age, delaying therapy to obtain additional laboratory specimens is not appropriate.

A 16-year-old male presents to your office requesting clearance to play football. You begin by taking his medical history. He says that he feels very well, but admits that he recently experienced one episode of syncope that occurred when he trained really hard for football tryouts with his friends. He denies any shortness of breath, or chest pain currently. Family history is significant for an uncle who died of heat stroke at the age of 30 while playing basketball. Physical examination reveals no abnormalities. What is the next best step in management? A. ECG now, and if normal, reassurance B. Medically clear him to play C. Stress test D. ECG and referral to cardiology E. Observe and follow up in 6 months

> D has been selected by the expert. A. An ECG is indicated in all cases of syncope. Neurocardiogenic syncope, also known as vasovagal syncope, occurs when the brain is not being adequately perfused. Predisposing factors may include dehydration, hypovolemia, and standing up too quickly. It is not usually dangerous, and in these cases an ECG would be an appropriate first step. However, in cases of atypical syncope, as above, in which syncope occurs with exertion, and ECG would not be a sufficient work up. B. All medical issues must be resolved prior to clearing a patient to participate in a sport. Syncope with exertion, during exercise, is very concerning, and this patient should be worked up appropriately prior to clearance. C. A stress test is appropriate for evaluating chest pain, particularly in those at risk for atherosclerosis. However, in a patient with known syncope with exertion, a stress test might actually be dangerous. In addition, it only considers vessel disease as a source of pain, and does not take into account structural issues like an outlet obstruction. D. Referral to cardiology is the absolute next best step! The combination of syncope with exertion and a family history of a young death is concerning for something like hypertrophic cardiomyopathy. Don't be fooled about heat stroke. That is a positive family history for sudden death in a young person. This patient must be evaluated by cardiology, even if you don't hear a cardiac murmur! E. Observation is not appropriate here. As mentioned above, the syncopal event and positive family history are concerning. It would be inappropriate to just observe if this patient has hypertrophic cardiomyopathy, a very significant risk factor for sudden death. Cardiology consult is the only appropriate option here.

A 10-year-old boy presents to his pediatrician with a history of hypopigmented non-pruritic "dots," mostly located on his face and neck. His mother complains that lesions get worse during the summer when her son plays outside. On exam, they are slightly scaly, hypopigmented lesions approximately 0.5 cm in diameter. What is the most likely etiology of his rash? A. A pox virus B. Hyperproliferation of keratinocytes C. S. pyogenes and S. aureus D. Decreased number of active melanocytes and decreased number and size of melanosomes E. Ingrown hairs with resultant inflammation

> D has been selected by the expert. A. Answer choice A is incorrect because a pox virus is the cause of molluscum contagiosum. Lesions are dome-shaped, firm, and often with a central indentation. They can appear anywhere on the body, but spare palms and soles. B. Answer choice B is incorrect because hyperproliferation of keratinocytes is characteristic of psoriasis. Psoriasis can present in a variety of ways, but most commonly presents as a plaque. Plaques are often located on the scalp, external ear, and extensor surfaces presenting as a thick silvery scale and sharply demarcated borders. It may or may not be pruritic. C. Answer choice C is incorrect because Staphylococcus aureus and Streptococcus pyogenes cause cellulitis. It is most often "non-bullous" but may also present as "bullous impetigo" or "ecthyma." They may start as papules and then become inflamed vesicles, progressing to pustules with a "honey-colored" crust. Lesions often appear near the nose because of rubbing. D. Answer choice D is correct. Pityriasis alba,common in children 3 to 16 years of age, presents as hypopigmented macules. They most often occur on the face, neck, trunk, and extremities. They have irregular borders, can vary in size, and may have a slight scale. Lesions may become more noticeable after sun exposure because of tanning of the surrounding skin. The etiology of this disorder is unknown, but ultrastructural examination of epidermal cells reveal decreased number of active melanocytes as well as decreased number and size of melanosomes. E. Answer choice E is incorrect because these findings are characteristic of pseudofolliculitis barbarae. Pseudofolliculitis occurs as papules normally located in the beard or hair distribution. It appears similar to acne, but it is more closely located to hair follicles. Development of this disorder is related to trauma due to shaving or plucking.

A concerned mother brings her 7-day-old son to your office after noticing yellowing of his skin for 2 days. She has also noticed he has not been gaining weight since she brought him home from the hospital 5 days ago. This is her first son and she has been trying to do everything perfectly, including breastfeeding him, since she was told that breast milk provides adequate nutrients and other healthy benefits, like antibodies and growth factors. However, upon further questioning, she is feeding him only 6 times a day for 10 minutes each time. She admits her breasts often feel full and are not relieved by nursing. He was born full term by spontaneous vaginal delivery but had a hard time sucking with breastfeeding. Upon exam, he looks dehydrated and appears to have jaundice of the face and chest. He has also lost > 10% of his birth weight. What could be the cause of his jaundice? A. Breast-milk jaundice B. Physiologic jaundice C. Sepsis D. Breastfeeding jaundice E. Crigler-Najjar syndrome

> D has been selected by the expert. A. Breast-milk jaundice is incorrect, because although it is a cause of jaundice associated with breastfeeding, it is NOT a result of low milk volume. In the above vignette, the infant does not appear to be breastfeeding well, which is multifactorial-poor suck and low number of feeds for the infant. Breast-milk jaundice would appear only if the infant were feeding well, although it is not completely understood what causes this form of jaundice. B. Physiologic jaundice is incorrect because this jaundice usually peaks at 3-4 days of life to full-term, healthy infants. This type of jaundice is not associated with breastfeeding but could be from numerous factors such as increased bilirubin production from short-lived RBCs or lack of intestinal flora to metabolize bile. No treatment is required for this type and it typically resolves on its own. C. Sepsis is incorrect because the infant does not clinically appear sick or toxic, and jaundice would be just one sign of a serious infection. Other symptoms may include respiratory distress, lethargy, poor feeding, vomiting, apnea, and temperature instability. Sepsis causes an elevated total and direct bilirubin and is highly unlikely when jaundice is the only symptom. Breastfeeding can have some protection against infection. D. Breastfeeding jaundice is the correct answer because it usually appears early in the first week of life and is caused by various factors, including poor breast milk intake. A decreased milk supply leads to limited enteral intake and can lead to increased enterohepatic circulation. Increased enterohepatic circulation describes the process where unconjugated bilirubin is reabsorbed in to the bloodstream where it binds to albumin and is recirculated. E. Crigler-Najjar syndrome is an incorrect choice because it appears within the first days of life and persists thereafter and is a relatively rare disease. This type of jaundice is caused by decreased bilirubin clearance from deficient or completely absent UDPGT (the enzyme that conjugates bilirubin with glucuronide to make it water-soluble and able to undergo excretion into bile).

During the middle of dinner on your day off, you receive a call from one of your neighbors who remembers that you are a medical student. He is concerned about his 15-year-old daughter who had previously been in her usual state of health and has no significant past medical history. However, over the past 24 hours, his daughter suddenly spiked a fever of 103 F and has "not been herself," acting very lethargic and dazed at times. He also notes that she has been breathing heavily, not been able to eat or drink, and has not urinated over the past 12 hours. He wants your advice about whether she should be taken to the ED. Although you are fairly certain that the best course of action would be to take her to the ED, you contemplate the differential diagnosis of her presentation. Given the limited history, which of the following is highest on your differential? A. Acute cystitis B. Diabetic ketoacidosis C. CNS tumor D. Meningitis E. Hypoglycemia

> D has been selected by the expert. A. Choice A is incorrect because acute cystitis is an infection that is usually limited to the lower urinary tract. While acute cystitis can present with fevers and dysuria, it would likely not present with more systemic systems, like tachypnea, altered mental status, and decreased PO intake. Such a systemic presentation would be more characteristic of bacteremia secondary to pyelonephritis. B. Choice B is incorrect because diabetic ketoacidosis is usually associated with increased (not decreased) urine output secondary to the osmotic effect of ketones and glucose in the urine. Thus, while diabetic ketoacidosis should certainly be on the differential for causes of altered mental status and lethargy, it is not high on the differential given the presentation. C. Choice C is incorrect because a CNS tumor would likely not result in such an acute presentation of altered mental status. While a CNS tumor may result in tachypnea and lethargy depending upon the location of the tumor, the presentation would likely be a more slow and gradual progression. Furthermore, a patient with a CNS tumor would likely not present with such a high-grade fever, which places a CNS tumor lower on our differential. D. Choice D is correct because meningitis is the most likely etiology in our differential given the fever, altered mental status, decreased PO intake, and decreased urine output. While the incidence of meningitis has decreased in this patient's age range due to increased vaccinations against the most common causative organisms of meningitis, it still remains high on our differential given the presentation of this patient. In the ED, we would likely need to obtain a more thorough history and physical exam as well as blood cultures and lumbar puncture to establish the diagnosis of meningitis. E. Choice E is incorrect because hypoglycemia is unlikely to be source of such a high-grade fever (unless the patient had sepsis which resulted in high fever and in hypoglycemia). Furthermore, hypoglycemia would likely not affect urine output over such a significant period of time. While hypoglycemia should certainly be on the differential for causes of altered mental status, the presentation of fever and decreased urine output makes places it lower on our differential.

A 4-day-old male infant was born by vaginal delivery to a 35-year-old G1P1 who declined prenatal screening. The infant has mild hypotonia, epicanthal folds, upslanting palpebral fissures and a flat face. On physical exam, he is in no acute distress, has normal oxygen saturation and has a continuous murmur. Lymphocyte karyotyping showed a particular change in chromosome number. What genetic abnormality is most likely? A. Trisomy 18 B. 45 X0 karyotype C. Trisomy 13 D. Trisomy 21 E. 47 XXY karyotype

> D has been selected by the expert. A. Choice A is incorrect because this describes Edwards syndrome, caused by an extra chromosome 18. These infants present with severe mental retardation, micrognathia (small jaw), low-set ears, short neck, overlapping fingers (digits 2 and 5 overlap medially upon closure of fist), heart defects, renal malformations, limited hip abduction, and "rocker-bottom" feet (the plantar side of foot has a u-shaped curve like a rocking chair). B. Choice B is incorrect because this is characteristic of Turner syndrome. Patients with Turner syndrome are female (males demise) and present with neck lymphedema (can be seen as cystic hygroma in utero), low ears, hand/foot edema, hyperconvex nails, a wide chest with valgus at the elbows. Twenty percent of patients have aorta coarctation. Many do not present classically and may be diagnosed in their early teens when stature is short and no sexual maturation ensues. Most have a normal IQ. C. Choice C is incorrect because this describes Patau syndrome due to an extra chromosome 13. Presentation includes a general theme of shifting facial features toward the midline: microphthalmia (some present with a midline Cyclops eye), microcephaly with severe mental retardation, polydactyly, cleft lip and palate, cardiac and renal defects, umbilical hernias, and cutis aplasia (particularly in the midline of the scalp). D. Choice D is correct because an extra chromosome 21 is indicative of Down syndrome. Patients with Down syndrome will present with the features described in the vignette as well as small ears, redundant nuchal skin, clinodactyly (the fifth digit is shorter and curved toward the radius). Additionally, cleft lip or palate, strabismus, and hypothyroidism may be seen. E. Choice E is incorrect because this describes Klinefelter syndrome. These patients are males who have inherited an extra X chromosome. At birth they usually appear unremarkable without findings. Findings are variable but include testicular atrophy with resultant infertility, eunuchoid body (unusually long limbs compared to the body), and gynecomastia. IQ may be normal or in the low-normal range.

A 5-year-old male comes to the clinic with a chief complaint of four days of progressively worsening fever and that has been minimally responsive to acetaminophen. The patient complains of sore throat and decreased appetite. His sister had a positive rapid strep test and is now being treated with amoxicillin. Your concern is for Group A strep. What is the next best step in management? A. Start antibiotic treatment B. Send blood cultures C. Advise parents to give patient acetaminophen with return precautions D. Rapid strep test with back-up culture if negative E. Chest x-ray

> D has been selected by the expert. A. Choice A is incorrect. Although you may empirically treat this child for infection with Group A strep, a test to diagnose infection should be done prior to initiation of antibiotics. B. Choice B is incorrect. There is no indication for a blood culture at this time. C. Choice C is incorrect. As the patient has a history of being exposed to a sick contact with Group A strep, being sent home with acetaminophen is also not sufficient because the patient has already been treated at home with acetaminophen with no improvement. D. Choice D would provide confirmation of your clinical suspicion and allow for correct diagnosis prior to empiric antibiotic treatment. E. Choice E is incorrect because there is no indication of respiratory symptoms.

A 12-month-old previously healthy girl presents with cough and mild subcostal retractions. She is afebrile, and physical exam reveals asymmetric wheezing. Chest x-ray demonstrates unilateral air trapping. What is the most likely diagnosis? A. Croup B. Pneumonia C. Acute bronchiolitis D. Foreign body aspiration E. Asthma

> D has been selected by the expert. A. Croup involves subglottic inflammation, typically presenting with inspiratory stridor and a "barky cough" (i.e., like a seal). This patient is not noted to have either, and also presents with asymmetric wheezing and air trapping that would not be expected in an individual with croup. B. While this patient presents with cough and increased work of breathing, she is afebrile, and auscultation of the chest does not reveal crackles or decreased breath sounds/area of consolidation, which would be consistent with pneumonia. Additionally, chest x-ray findings are not consistent with a lobar or more diffuse pneumonia. C. Acute bronchiolitis is a good thought, especially as this is the most common cause of wheezing in infant; however, if this were the diagnosis, the patient would most likely be febrile and chest x-ray would demonstrate scattered atelectasis and/or diffuse opacities from bronchial obstruction. D. Features of foreign body aspiration include unexplained wheezing and asymmetric breath sounds, as well as air trapping in one lung indicating unilateral airway obstruction. The right main bronchus is the more commonly obstructed due to anatomy (it is wider and more vertical than the left). The most commonly aspirated foods are hot dogs, nuts, hard candy, grapes, and popcorn. E. While the finding of wheezing is consistent with asthma, this patient has wheezing only on one side. Along those lines, chest x-ray in an asthmatic patient would demonstrate global air trapping with hyperinflated lungs, rather than unilateral findings.

A 4-year-old patient with sickle cell disease presents for a well child evaluation. She has a history of three sickle cell vaso-occlusive crises in the past, including dactylitis and bone pain. She has been symptom-free for a few months and today she is feeling well. She is meeting her developmental milestones. She had an upper respiratory infection recently, but seems to be getting better now. She is up to date on her standard vaccinations up to 2 years including a full course (four doses) of Prevnar. What would you do for her today? A. Echocardiogram and transcranial ultrasound B. Quantiferon gold test C. HPV vaccine D. A dose of pneumococcal polysaccharide vaccine (Pneumovax). E. Chest x-ray

> D has been selected by the expert. A. Echocardiogram and transcranial ultrasound are recommended for patients with sickle cell disease. Echocardiogram screens for evidence of left ventricular hypertrophy and cardiomyopathy. Transcranial ultrasound is used to evaluate risk of stroke by measuring the middle cerebral artery flow. Routine transcranial ultrasound screening for these patients is recommended; echocardiogram is usually recommended for patient older than 21 years old, or for those with cardiopulmonary symptoms. B. A quantiferon gold test assesses previous exposure to TB by measuring for production of IFN by memory T cells from heparinized blood. This test is an acceptable way to measure whether a patient has been infected with TB, including latent infection, but is not indicated in this setting. C. HPV is recommended for children starting at age 11 years old. D. All children should routinely receive the Prevnar vaccine series. Patients with risk factors for pneumococcal sepsis, such as those with sickle cell disease, damaged spleen or asplenia, cochlear implants, CSF leaks, HIV, immunocompromise, chronic hear/lung disease, or those taking immunosuppressive medication should also receive the 23-valent polysaccharide pneumococcal vaccine at age 2 years. E. Chest x-ray is not recommended when there are no current URI symptoms.

A 5-day-old infant presents with a chief complaint of jaundice. As you obtain a careful history and physical examination, which of the following would NOT be a risk factor for jaundice in this infant? A. Mediterranean origin B. Prolonged labor with use of forceps during the delivery C. Mother is type O+ and baby is type B D. Phenylketonuria E. Poor breastfeeding during first few days of life

> D has been selected by the expert. A. Families of Mediterranean origin have higher risks of hereditary diseases that can cause jaundice. Glucose-6-phosphate dehydrogenase (G6PD) deficiency and the thalassemias are more common in Mediterranean families. Both cause hemolysis of RBCs, which results in an increased of jaundice. B. Difficult deliveries and birth trauma may result cephalohematomas, or hemorrhage of blood between the skull and the periosteum. These result from the rupture of blood vessels crossing the periosteum, usually caused by a prolonged second stage of labor or the use of forceps or other instrumentation during delivery. As the cephalohematoma resolves over weeks and the blood is reabsorbed, the breakdown of RBCs from the hematoma can result in increased bilirubin levels. C. ABO compatibility is a common cause of hemolysis in newborns. Since the mother has blood type O, she makes IgG antibodies to A-antigens and to B-antigens, leading to hemolysis of the fetal red blood cells. This can be detected by a positive direct Coombs test. D. Phenylketonuria (PKU) is an autosomal recessive metabolic disorder due to a mutation in phenylalanine hydroxylase, which is required to convert phenylalanine to tyrosine. PKU leads to buildup of phenylalanine in the brain, leading to mental retardation, seizures, and death if not detected and treated early. It is not associated with jaundice. E. Poor breastfeeding during the first few days of life and decreased enteral intake may result in breastfeeding jaundice. This may be caused by the mother who does not make enough milk, or by the infant with inadequate feeding or intake. Decreased intake leads to decreased motility of the GI tract and retention of meconium. The meconium contains β-glucuronidase, which hydrolyzes the conjugated bilirubin to an unconjugated form, which is reabsorbed and re-circulated into the blood through the enterohepatic circulation, increasing bilirubin levels in the blood.

A 3-week-old infant is brought to the pediatrician for failure to thrive (despite adequate, even prolonged, feedings) and respiratory distress (particularly tachypnea). EKG shows high voltage QRS complexes in leads V1 and V2. What other features does this infant most likely have? A. Cyanosis from a right-to-left shunt B. Systolic murmur with a widely split second heart sound C. Continuous murmur that is louder during systole D. Left-to-right shunt

> D has been selected by the expert. A. Incorrect. After the pulmonary resistance of the newborn period diminishes, ventricular septal defects (VSD) present with a left-to-right shunt. If unrepaired spontaneously or surgically, this ultimately leads to increased pulmonary vascular resistance, termed Eisenmenger's syndrome. When this occurs, the pulmonary vascular resistance equals and then exceeds that of the systemic vascular resistance, leading to reversal of the direction of blood flow through the VSD to become a right-to-left shunt. However, this generally occurs if the VSD is allowed to persist for months to years, which is less likely in our 3-week-old infant. B. Incorrect. A widely split, fixed S2 indicates an atrial septal defect (ASD). These are often detected in children when they're between 3 and 5 years old. The systolic murmur is due to the increased blood flow across the pulmonic valve. The widely split fixed second heart sound indicates an ASD is the cause of the murmur rather than an innocent heart murmur. C. Incorrect. A continuous murmur that is louder during systole describes a patent ductus arteriosus (PDA). A PDA, like a VSD, is also discovered during infancy. However, a holosystolic murmur with a palpable thrill at the left lower sternal border is more characteristic of a VSD. The murmur of a VSD is from the left-to-right shunting of blood during systole due to systemic vascular resistance exceeding pulmonary vascular resistance. D. Correct. A heart murmur from a VSD is typically not appreciated in the immediate newborn period, as the pulmonary vascular resistance is still quite elevated. During this time, since the pulmonary vascular resistance equals the systemic vascular resistance, there is no shunting of blood through the open VSD. However, after a few days to weeks after birth, the pulmonary vascular resistance decreases, and the murmur appears, reflecting the shunted flow of blood through the open VSD (from left to right).

A 6-month-old female is brought into the pediatrician's office for three days of high fever, fussiness, and decreased appetite. The patient has not had any upper respiratory tract symptoms, vomiting, diarrhea, or rash. On physical exam the patient is fussy, has a RR of 28 bpm and a pulse of 160 bpm. She is febrile to 102.8 F (rectal). The patient is alert and fully moving all extremities. Apart from her vital signs, no other significant exam findings are noted. A CBC demonstrates leukocytosis of 17.0 cells x 103 / µL with elevated bands. What diagnosis is most likely? A. Measles B. Bacterial meningitis C. Acute otitis media D. Urinary tract infection E. Roseola

> D has been selected by the expert. A. Measles typically begins with a "prodrome" period featuring the "3 Cs" (cough, coryza, conjunctivitis)-none of which this patient has-along with high fever, often > 104 F, and general malaise and anorexia. On the 2nd to 4th day a maculopapular erythematous rash appears starting on the face/upper neck and spreading downward. Although infants receive their first vaccination against measles (the MMR) at 1 year of age, infants are generally protected unless they are exposed to older, unimmunized children who have the disease. B. This answer is incorrect in this situation for several reasons. First, this patient is not toxic appearing, nor is her physical exam positive for any findings suggestive of meningitis (such as bulging fontanel or extreme irritability). Unlike older children or adults, classic meningeal signs will often not be present or will be difficult to appreciate on an infant. The patient's CBC is significant for leukocytosis with elevated bands, suggesting a bacterial infection. On exam the patient is alert and responding well to her environment and does not demonstrate lethargy, respiratory distress or signs of ICP. A definitive lumbar puncture may be ordered if there is more suspicion for meningitis or if the patient's status deteriorates in any way, and caretakers should be given a follow-up appointment as well as clear indications of when to seek care. C. This answer is incorrect because while fever and fussiness can be possible signs of AOM in infants, there is no evidence of infection on physical exam such as inflamed, erythematous tympanic membranes, with bulging of the membrane indicating an effusion. D. UTI, the most common bacterial illness in a female infant, is consistent with her high fever, fussiness, and decreased appetite. Her CBC suggests that she has a bacterial infection (leukocytosis and elevated bands). A sample of her urine should be obtained by catheterization and sent for urinalysis and culture. E. Roseola often presents with a high fever, but also often with a viral prodrome. It is a diagnosis of exclusion at this point and should not preclude obtaining a urine sample in this child.

A 3-year-old girl presents to the ED with sudden onset difficulty walking. She does not have a fever, headache, nausea, or vision changes, but two weeks ago she had a runny nose, a fever, and a rash. Musculoskeletal exam reveal no abnormalities of lower extremities. Neuro exam reveals bilateral horizontal nystagmus, wide based stance and swaying, and bilateral overreaching on finger to nose test. An LP is performed which reveals a normal CSF. Which of the following is the most likely diagnosis? A. Opsoclonus-myoclonus syndrome B. Hydrocephalus C. Infectious cerebellitis D. Post-infectious cerebellitis E. Migraine headache

> D has been selected by the expert. A. Opsoclonus-myoclonus syndrome is incorrect. It is a paraneoplastic syndrome that occurs most often with neuroblastoma in a young child who presents with ataxia and jerking or erratic movements as well as jerky conjugate movements of the eyes. B. Hydrocephalus is incorrect because the presentation tends to be more gradual or insidious in onset and it is usually associated with headache and vomiting. C. Infectious cerebellitis is incorrect because it presents with fever and sometimes mental status changes. Pathogens that cause this may include EBV, mumps, and enterovirus. D. Post-infectious cerebellitis is correct. This typically presents in a younger child with ataxia, nystagmus, vomiting and sometimes dysarthria. It is believed to be an autoimmune response leading to demyelination of the cerebellum occurring several weeks after a viral infection such as varicella or coxsackie virus. E. Migraine headache is incorrect because this would present with a headache. Although basilar artery migraines and hemiplegic migraines may present with acute ataxia, headache is an accompanying symptom.

Luanne is a 15-year-old female with three hours of abdominal pain and two episodes of non-bilious, non-bloody vomiting. She rates her pain at 8/10 and describes it as constant and located mainly in the middle of her belly, but somewhat present throughout her abdomen. It is worse with coughing and moving. She has never had this pain before, and has had no appetite since the pain started. She is sexually active with her boyfriend of three months, always uses condoms, and has not been tested for STIs. Her last menstrual period was two weeks ago. Vitals: 37.9 C, HR 100 bpm, BP 120/85 mm Hg, RR 14 bpm. On exam, she exhibits involuntary guarding, mild rebound tenderness, and tenderness to palpation between her right anterior superior iliac spin and umbilicus. On pelvic exam, she reports tenderness when attempting to palpate her right adnexa, but no masses are appreciated and there is no cervical motion tenderness. Her WBC and CRP are within normal limits. Based on the information above, what is the most likely diagnosis? A. Ovarian torsion B. Pelvic inflammatory disease C. Ectopic pregnancy D. Appendicitis

> D has been selected by the expert. A. Ovarian torsion is more common in the post-menopausal population, though it can present in any age group. It is described as intermittent stabbing pain in the lower abdomen or pelvis. Torsion is often secondary to an ovarian mass, such as a neoplasm or corpus luteal cyst, which may occasionally be appreciated on exam. Nausea and vomiting are very common findings as well. Ultrasound is essential to initial workup. Given Luanne's pain localized around her belly button, her tenderness at McBurney's point, and lack of palpable masses on pelvic exam, ovarian torsion is a less likely diagnosis. B. Pelvic inflammatory disease is definitely a possibility given Luanne's sexual history, lack of STI screening in the past, and adnexal pain. However, this pain is often post-coital and also first occurs during or immediately following menstruation. Another key finding is mucopurulent discharge and cervical motion tenderness, both of which are absent in Luanne. RUQ pain and a fever (present in 50% of patients with PID) are other signs that are not reported in this case. C. Patients with an ectopic pregnancy typically present with painless vaginal bleeding six to eight weeks after their last menstrual period. The pain is described as crampy pelvic pain, and it is often associated with nausea. Diffuse abdominal pain is also present if rupture and intraperitoneal bleeding occurs. Given that Luanne is two weeks from her last menstrual period, she reports no vaginal bleeding, and her pain is intense and located in the middle of her abdomen, an ectopic pregnancy is less likely. D. Appendicitis is the most common condition in children requiring immediate surgical intervention, but often (especially in infants) presents differently than in adults. Aspects of their atypical presentation include lack of migration of pain to the RLQ, negative Rovsing's sign, and involuntary guarding and fever without perforation. School-age children who can articulate the pain often describe pain with movement or coughing (cat's eye sign). Also, rebound tenderness was found to be neither sensitive nor specific in the pediatric population, while in the adult population it is one of the most accurate PE findings (86%). Luanne is of the older pediatric population, and so will present with a more typical appendicitis. Her sudden onset of intense pain at the umbilicus with vomiting, anorexia, and tenderness at McBurney's point are all classic findings. The more atypical signs include diffuse pain centered below the umbilicus, and rebound tenderness that might point to a perforation (more likely, it is part of the atypical pediatric presentation given her normal WBC). Another atypical aspect of her exam is her adnexal pain during the pelvic exam, which could be due to the degree of inflammation and the positioning of her appendix. The key take-away point is to have a high index of suspicion for appendicitis in pediatric patients with abdominal pain given their atypical presentation.

Luanne is a 15-year-old female who presents with three hours of abdominal pain and two episodes of non-bilious, non-bloody vomiting. She rates her pain at 8/10 and describes it as constant, located mainly in the middle of her belly but somewhat present throughout her abdomen. It is worse with coughing and moving. She has never had pain like this before, and has had no appetite since the pain started. She is sexually active with her boyfriend of three months, always uses condoms, and has not been tested for STIs. She is due to start her period next week. Vitals: 37.9 C, HR 100 bpm, BP 120/85 mmHg, RR 14 bpm. On exam, she exhibits involuntary guarding, mild rebound tenderness, and tenderness to palpation between her right anterior superior iliac spine and umbilicus. On pelvic exam, she reports tenderness when attempting to palpate her right adnexa, but no masses are appreciated and there is no cervical motion tenderness. Her WBC and CRP are within normal limits. Based on the information above, what is the most likely diagnosis? A. Ovarian torsion B. Pelvic inflammatory disease C. Ectopic pregnancy D. Appendicitis E. Cholecystitis

> D has been selected by the expert. A. Ovarian torsion is more common in the post-menopausal population, though it can present in any age group. It is described as intermittent stabbing pain in the lower abdominal or pelvis. Torsion is often secondary to an ovarian mass, such as a neoplasm or corpus luteal cyst, which is occasionally appreciated on exam. Nausea and vomiting are very common findings as well. Ultrasound is essential to initial workup. Given that Luanne has periumbilical pain, tenderness at McBurney's point, and no palpable masses on pelvic exam, ovarian torsion is a less likely diagnosis. B. Pelvic inflammatory disease is definitely a possibility given her sexual history, lack of STI screening, and adnexal pain. However, this pain is often post-coital and also first occurs during or immediately following menstruation. Another key finding is mucopurulent discharge and cervical motion tenderness, both of which are absent in Luanne. RUQ pain and a fever (present in 50% of patient) are other signs not seen in this case. C. Patients with ectopic pregnancy typically present with painless vaginal bleeding six to eight weeks after their last menstrual period. The pain is described as crampy pelvic pain, and it is often associated with nausea. Diffuse abdominal pain is also present if rupture and intraperitoneal bleeding occurs. Given that Luanne is three weeks from her last menstrual period, reports no vaginal bleeding, and her pain is intense and located in the middle of her abdomen, an ectopic pregnancy is less likely. D. Appendicitis is the most common condition in children requiring immediate surgical intervention, but often presents differently than in adults (especially in infants). Aspects of their atypical presentation include lack of migration of pain to the RLQ, negative Rovsing's sign, and involuntary guarding and fever without perforation. In school-age children who can articulate the pain, they often describe pain with movement or coughing (cat's eye sign). Also, rebound tenderness was found to be neither sensitive nor specific in the pediatric population, while in the adult population it is one of the most accurate PE findings (86%). Luanne is of the older pediatric population, and so will present with a more typical appendicitis. Her sudden onset of intense pain at the umbilicus with vomiting, anorexia, and tenderness at McBurney's point are all classic findings. The more atypical signs include diffuse pain centered below the umbilicus, and rebound tenderness that might point to a perforation (more likely, it is part of the atypical pediatric presentation given her normal WBC study). Another atypical aspect of her exam is her adnexal pain during the pelvic exam, which could be due to the degree of inflammation and the positioning of her appendix. The key take-away point is to have a high index of suspicion for appendicitis for pediatric patients with abdominal pain given their atypical presentation. E. The pain of cholecystitis is steady, most often present in the RUQ, and can radiate to the right shoulder. It can worsen with ingestion of fat-rich foods, and often causes anorexia with episodes of nausea and vomiting. It is much less common in children than adults. Given Luanne's presentation, age, and exam, cholecystitis is very low on our differential.

A 10-year-old female comes to the clinic for a well child exam. Her mom asks about puberty and wants to know in what order she should expect to see normal developmental changes in her daughter. Which of the following sequences is correct? A. breast bud -> pubic hair -> menarche -> growth spurt B. pubic hair -> breast bud -> growth spurt -> menarche C. pubic hair -> menarche -> breast bud -> growth spurt D. breast bud -> pubic hair -> growth spurt -> menarche E. pubic hair -> breast bud -> menarche -> growth spurt

> D has been selected by the expert. A. This choice is incorrect because a growth spurt occurs before menarche. B. This choice is incorrect because development of a breast bud is the first sign, followed by pubic hair. C. This choice is incorrect because pubic hair is not the first sign. D. This choice is correct because breast buds are the first sign (10-11 years), followed by pubic hair (10-11 years), then a growth spurt (12 years), and then menarche (12-13 years). Most girls reach adult height by approximately 15 years. E. This choice is incorrect because the order is wrong. The correct order is: breast bud -> pubic hair -> growth spurt -> menarche.

Jade is a 2-week-old female who was born at home and received no newborn screenings for congenital disease. Her mother brought her to the pediatrician's office concerned that her daughter appeared to be jaundiced and was constipated, tired, and not feeding well most of the time. Physical exam was notable for enlarged fontanels, jaundice without bruising, hypotonia without tremor or clonus, and an umbilical hernia. There was no sign of virilization, no abnormal facies, and no history of vomiting. Review of systems was otherwise negative except as stated above. Which of the following is the most important next step in Jade's management? A. Glucose and electrolyte supplementation B. Glucocorticoid and mineralocorticoid supplementation C. No treatment needed D. Consult with pediatric endocrinologist and start treatment with 10 to 15 mcg/kg/day of crushed levothyroxine in liquid, and follow up every 12 months E. Empiric antibiotics after collection of blood, urine, and CSF cultures.

> D has been selected by the expert. A. This choice is incorrect because this is the treatment of choice for acutely ill children with dehydration, hypoglycemia, and perhaps infants with congenital adrenal hyperplasia, not congenital hypothyroidism as is most likely in this infant. B. This is the recommended treatment for corticoid and mineralocorticoid deficiency as seen in congenital adrenal hyperplasia. Clinical evaluation of this patient does not show virilization of this female infant, commonly seen in congenital adrenal hyperplasia. However, it would still be important for our patient in this case to undergo newborn screening, which includes screening for this disorder. C. This choice is incorrect because congenital hypothyroidism requires treatment within 2 weeks of onset of symptoms to mitigate severe brain damage and cognitive impairment. No treatment is needed for children born with hypothyroxemia of prematurity without TSH elevation. D. This choice is correct because the American Academy of Pediatrics recommends this treatment regimen for infants age 0 to 6 months old. Dosing is based upon age and weight. It would also be important to consult with a pediatric endocrinologist to evaluate the short and long-term treatment plan. In addition, the specialist could also recommend screening for other autoimmune disorders. E. This choice is incorrect. Although sepsis must always be considered in a neonate with jaundice, there is no indication of bacterial infection in this infant with other signs and symptoms of congenital hypothyroidism.

A 14-month-old female with no significant past medical history presents to clinic with fever to 39.2 C and irritability. According to mom, the patient was initially sick one week ago with a runny nose and cough, but these symptoms had resolved. She started pulling at her ear and becoming increasingly irritable last night, with her fever spiking around 2:00 a.m. this morning. Patient is up to date on immunizations, and has had several prior ear infections. She was most recently treated last month with amoxicillin. When you examine her ears, you observe a red, bulging tympanic membrane with limited mobility in her left ear. The exam of the right ear is normal. You are confident in your diagnosis of acute otitis media. What is your treatment plan? A. Observation B. Anthistamines and decongestants C. High-dose amoxicillin D. Amoxicillin/clavulanate (with high-dose amoxicillin component) E. Tympanocentesis

> D has been selected by the expert. A. This choice is incorrect due to the patient's presentation with a high fever above 39 C. Fifty to 80% of acute otitis media cases will resolve spontaneously without antibiotics; however, the decision to defer treatment with the "observation option" is based on the child's age and illness severity. This option is limited to healthy children between the ages of 6 months to 2 years with non-severe symptoms. Our patient is presenting with severe symptoms and a high-grade fever. B. This choice is incorrect because the FDA has discouraged the use of over-the-counter cough and cold products in children younger than 2 years due to the increased risk/benefit ratio. Also, these medications would be most useful for the upper respiratory symptoms preceding the ear infection. C. High-dose amoxicillin is the most common first-line treatment for acute otitis media due to its general effectiveness against susceptible and partially resistant S. pneumo, in addition to being low cost and having a high safety profile. However, this antibiotic was recently administered, raising concerns for a resistant organism. D. This choice is correct because of the severe symptoms our patient is exhibiting with a high temperature greater than 39 C. Amoxicillin/clavulanate is the treatment of choice for patients with moderate to severe otalgia or high fever, and is used for additional beta-lactamase coverage for Haemophilus influenzae and Moraxella catarrhalis, and when failure with amoxicillin is suspected. E. This choice is incorrect because the patient has not had recurrent episodes of otitis media and has not started antibiotic therapy. Tympanocentesis is recommended as a diagnostic measure to confirm a bacterial etiology after a patient has failed repeated courses of antibiotics or if an unusually resistant organism is suspected.

A 3-year-old male presents to clinic with an annular, well-circumscribed, scaly plaque with a raised erythematous border and central hypopigmentation on the left thigh. The mother reports that the lesion is highly pruritic and that the patient has been exposed to other children with a similar rash at day care. Upon further examination, a similar lesion with boggy borders is also found on the posterior aspect of his scalp. Which of the following is the most appropriate treatment for this child's problem? A. Topical clotrimazole B. Hydrocortisone 1% cream C. Oral prednisone D. Oral griseofulvin E. Selenium sulfide shampoo

> D has been selected by the expert. A. This choice is incorrect. Although topical antifungal therapy would be appropriate for the lesion on the leg (tinea corporis), involvement of the scalp (tinea capitis) necessitates systemic antifungals. B. This choice is incorrect. Steroid therapy is contraindicated in treatment of fungal infections. Treatment with hydrocortisone cream will worsen the fungal infection. C. This choice is incorrect. (See explanation for B.) D. This choice is correct. Topical antifungals are not usually successful in treating tinea capitis, because the infected hair follicles are deep within the scalp. Systemic griseofulvin is the first choice for the treatment of tinea capitis. E. Selenium sulfide shampoo may decrease the likelihood of transmission of the infection to others by decreasing the number of spores shed, but does not treat the primary infection.

A 4-year-old patient presents with several months of cough. Mom also reports a history of red skin patches, which are pruritic, and allergies to peanuts, eggs, and mangoes. Which of the following would be characteristic of the cough that this patient would present with? A. Does not awaken patient from sleep B. Paroxysmal C. Barking cough D. Worse at night E. Associated with crackles on exam

> D has been selected by the expert. A. This choice is incorrect. This patient has asthma, which commonly presents with symptoms awakening the patient from sleep. Habitual cough disappears at night. B. This choice is incorrect. Paroxysmal coughs are associated with bacterial infections such as pertussis, Chlamydia, or mycoplasma. Foreign bodies can also produce sudden onset of cough. History could help to determine if the latter is a cause. C. This choice is incorrect. Barking coughs are associated with croup or other forms of subglottic disease. Foreign bodies can also produce this type of cough. D. This choice is correct. Asthma frequently presents with nighttime exacerbations. The cough often presents with wheezing and is usually a dry cough. E. This choice is incorrect. Diseases associated with crackles usually have intrinsic pulmonary involvement. Crackles can be fine or coarse and usually represent alveolar or small airway conditions.

A 16-year-old previously healthy male comes to the Pediatrics Urgent Care Clinic having "almost fainted" at soccer practice. He says that he had not eaten much earlier in the day and it was very hot and muggy outside. He felt light-headed and sick to his stomach. He denies losing consciousness and did not fall to the ground. He denies any chest pain. When you examine him, his eyes are sunken and he is tachycardic. What would be your next step in his management? A. Electrocardiogram (ECG) B. Measure his blood glucose C. Echocardiogram D. Give fluids and recheck his vital signs E. Stress test

> D has been selected by the expert. A. This choice is incorrect. While an EKG would be useful to assess for possible arrhythmia (long QT syndrome, WPW) or hypertrophic cardiomyopathy, the patient did not have a syncopal event or chest pain. As a result, an EKG would not be the most appropriate next step in diagnosis. B. This choice is incorrect. While the patient has had poor PO intake, hypoglycemia is relatively uncommon in an otherwise healthy young male. In addition, we would expect symptoms including diaphoresis, anxiety, and tremulousness, as opposed to dizziness. C. This choice is incorrect. Obstructive outlet pathology is less likely given the presentation, as no syncopal episode occurred. An echocardiogram may be indicated, but it would not be the next step in the evaluation. D. This choice is correct. The patient is likely dehydrated given the dizziness without loss of consciousness in the setting of poor PO intake, hot weather and exercise. As the symptoms occurred while he was upright, the likely mechanism is vasovagal. His sunken eyes and tachycardia are signs of moderate to severe dehydration. Since this is a clinical diagnosis, fluids should be given with subsequent rechecking of heart rate and blood pressure to confirm the diagnosis. E. This choice is incorrect. A cardiac etiology like hypertrophic cardiomyopathy is less likely given the presentation, and a stress test would not be the next step in the evaluation.

A 6-month-old infant arrives in the ED with a 12-hour history of poor feeding, emesis, and irritability. On exam, she is ill-appearing with T 39.2 C, P 160 bpm, R 40 bpm, BP 80/50 mmHg. CBC shows WBC 11.2, Hgb 13.5, Plt 250. Urinalysis shows > 100 WBC per hpf, positive leukocyte esterase, and positive nitrites. She has no history of prior urinary tract infection. Chest x-ray is negative. Urine and blood cultures are pending. After bringing her fever down, she was still uninterested in drinking, but her exam improved, and you were confident she did not have meningitis, so an LP was not performed. Which of the following is the best next step in management? A. Oral ampicillin B. Oral ampicillin + gentamicin C. Intravenous ciprofloxacin D. Intravenous ceftriaxone E. Intravenous piperacillin + tazobactam

> D has been selected by the expert. A. This patient is too sick for oral treatment, so oral ampicillin would be insufficient. Also, there is rising resistance of E. Coli to ampicillin, so secondary coverage with gentamicin or some other aminoglycoside would be needed unless cultures proved the organism was sensitive to ampicillin alone. B. Although parenteral and oral treatment produce similar outcomes in high quality RCTs, this patient is ill and refuses to drink and so requires parenteral antibiotics. IV ampicillin and gentamicin could be an appropriate choice for empiric coverage. C. Ciprofloxacin could be used for complicated UTIs, but it has the potential for adverse reactions in young children so is reserved for patients > 1 year with complications such as resistant organisms or urinary tract anomalies D. This patient's presentation is suggestive of a UTI. Given the ill appearance, vital signs, and white count, Upper tract disease (pyelonephritis) should be strongly considered. A parenteral (IV/IM) third-generation cephalosporin is the best choice of those listed for pyelonephritis, given its excellent gram negative coverage (except for Pseudomonas). E. IV piperacillin + tazobactam has excellent gram negative coverage with added Pseudomonas coverage, but it is highly expensive and Pseudomonas is unlikely to be the cause of a UTI in a child who is not regularly catheterized.

Adam is a 2-hour-old infant born at 32 weeks' gestational age via spontaneous vaginal delivery to a healthy mother with negative group B streptococcus status. There was no premature rupture of membranes and no meconium in the amniotic fluid. His Apgars were 8 at one minute and 9 at five minutes. Over the last two hours he has become progressively tachypneic. On physical examination he is large for gestational age. His vital signs are respiratory rate 75, temperature 36.5 C and heart rate is 130 beats per minute. His lung exam is remarkable for intercostal and subcostal retractions, grunting, and equal breath sounds. His heart exam reveals normal rhythm, normal S1 and S2, no murmurs, and normal peripheral pulses and capillary refill. Which of the following is the most likely cause of the patient's condition? A. Transient tachypnea of the newborn (TTN) B. Pneumothorax C. Congestive heart failure D. Respiratory distress syndrome E. Sepsis

> D has been selected by the expert. A. Transient tachypnea of the newborn (TTN) is much more common in infants born to diabetic mothers. TTN is unlikely because he is 32 weeks, very premature, and was born via NSVD. RDS is much more likely, although TTN is still a possibility and would need to evaluated with a CXR. B. Pneumothoraces are uncommon, but should always be considered in an infant with respiratory distress. Good bilateral air entry argues against this diagnosis. C. Congestive heart failure is an important cause of tachypnea. Adam has a normal cardiovascular exam, with no murmur, normal pulses, and good capillary refill. Infants with congestive heart failure usually present with the triad of tachypnea, tachycardia, and hepatomengaly. D. Respiratory distress syndrome (RDS) causes tachypnea and is therefore an important consideration in this case. RDS is more common in premature infants. Given the lack of history of maternal diabetes, an NSVD birth, and few risk factors for sepsis other than prematurity, Adam is likely to have RDS. E. Infants may present with respiratory distress from sepsis or from pneumonia. In this case, Baby Adam has a normal temperature, good blood pressure, and normal perfusion. While less likely, this diagnosis should always be considered in infants with respiratory distress.

A 3-year-old child is found to have a dry, pruritic rash on his face. Physical exam is notable for confluent areas of erythema and scaling. There are mild excoriations surrounding some areas and mild lichenification of the extensor surfaces of both elbows. What is the next best step in management of this child's problem? A. Oral clindamycin for 5 days B. Changing detergents C. Topical clotrimazole D. Topical steroids and emollients E. 5% permethrin cream

> D has been selected by the expert. A. Treatment with clindamycin would be appropriate for treatment of cellulitis due to MRSA. The rash described is not consistent with cellulitis. Cellulitis in children is usually caused by S. pyogenes or S. aureus. Treatment with a first-generation cephalosporin (such as cefalexin) or a semi-sythenthetic penicillin with a beta-lactamase inhibitor (amoxicillin-clavulonate) would be approppriate of there is no concern for MRSA. Clindamycin or trimethoprin/sulfamethoxazole would be appropriate treatment for cellulitis caused by MRSA. B. Changing detergents would be an appropriate intervention for irritant dermatitis. The fact that the child has lichenification on the extensor surfaces argues against this diagnosis. Irritant contact dermatitis is often found in the diaper area, face, and extensor surfaces on children secondary to irritating substances. It is typically less pruritic than atopic dermatitis. C. Treatment with topical clotrimazole would be appropriate for tinea corporis. However, the rash described in this vignette is not consistent with this diagnosis. D. Correct. Atopic dermatitis most often presents with dry, itchy skin in addition to erythema, scaling, vesicles, or lichenification in skin flexures. Treatment consists of emollients and topical corticosteroids. E. Treatment with permethrin cream would be appropriate for scabies infection. Scabies is characterized by papules or vesicles accompanied by severe pruritis, especially at night. In addition, burrows may be evident. The wrists, ankles, palms, soles, interdigital spaces, axilla, waist, and groin are among the most common locations for lesions. Finally, there is often a positive sick contact history with similar symptoms.

Johnny is a 25-month-old male who presents to the ED with a 2-day history of vomiting and diarrhea. Dad relays a history of abrupt onset of vomiting that started yesterday around 1 pm. Johnny has had 6 episodes of emesis since yesterday and 3 episodes of diarrhea. The emesis is non-bilious and the diarrhea is described as watery with specks of blood throughout the diarrhea. There are no sick contacts in the home. Vital signs: T 37.1, P 102, R 20, BP 90/60. Physical examination is normal and Johnny has still been tolerating some PO feeds without instant vomiting. What is the most immediate intervention for this patient? A. IV bolus with D5W B. IV bolus with 0.9% saline C. CT scan and surgical consult D. random glucose test E. no immediate intervention is necessary

> E has been selected by the expert. A. An IV bolus with D5W is indicated in cases of confirmed hypoglycemia and is used for maintenance fluids. This is not indicated in this patient. B. With his normal vitals and no obvious signs of dehydration, an IV bolus of 0.9% saline is not indicated in this patient. If there was evidence of dehydration on physical exam or with vitals, then IV fluids would be necessary. C. This vignette does not seem like a surgical case. Physical examination was normal, which makes abdominal pathology less likely. While appendicitis might be in the differential diagnosis, other diagnoses are more likely, making the excessive radiation exposure from a CT scan not necessary. D. With cases of dehydration one must always think about abnormalities in blood glucose levels. This clinical presentation does not suggest signs of hypo or hyperglycemia. E. At this point the patient is most likely suffering from a case of viral gastroenteritis. Because he is still tolerating some PO feeds, has no obvious signs of dehydration, and has normal vital signs, there is no need for aggressive IV fluid administration or diagnostic work up. Strict return precautions should be given and it should be advised that Johnny maintains fluids as much as possible.

A 1-month-old African-American male presents to your office for a check-up. The baby was born at term by NSVD to a 29-year-old G1P0 mother with no complications. Mother states the baby was feeding well until a week ago, when he developed increased sleepiness, prolonged feeding, and greater duration between feeds. His mother notes he stops to take breaks sometimes because he seems to be trying to catch his breath. He has 4 to 6 wet diapers per day and poopy diapers 3 or 4 times per day. Vital signs are: T: 37.6 C, RR: 68 bpm, P: 138 bpm, BP: 88/58 mmHg, and 02 saturation is 98%. The physical examination is notable for increased respiratory effort and retractions, and, upon cardiac examination, a murmur with a hyperactive precordium and no cyanosis. Abdominal exam reveals a liver edge palpable to 4 cm below the right costal margin. Which condition would be least likely to be the cause of the patient's symptoms? A. Aortic stenosis B. Coarctation of the aorta C. Ventricular septal defect D. Patent ductus arteriosus E. Atrial septal defect

> E has been selected by the expert. A. Choice A is incorrect because aortic stenosis is one of the heart defects that present with a murmur and signs of congestive heart failure in infancy. An estimated 10-15% of patients with aortic valve stenosis present when they are younger than one year of age. Neonates with critical stenosis are typically symptomatic and present with symptoms of congestive heart failure-including poor feeding, rapid breathing, poor urine output, and fussiness-as the ductus arteriosus closes and systemic blood flow decreases. B. Choice B is incorrect because coarctation of the aorta is one of the heart defects that present with a murmur and signs of congestive heart failure in infancy. Pediatric patients may present in the first few weeks of life with poor feeding, tachypnea, and lethargy and progress to overt CHF and shock. Symptoms may be subtle at first, and patients may make repeated trips to the physician before being diagnosed. Presentation after the neonatal period is usually consistent with hypertension or a murmur. Patients usually have not developed overt CHF because of the presence of arterial collateral vessels. C. Choice C is incorrect because ventricular septal defect is one of the heart defects that present with a murmur and signs of congestive heart failure in infancy. Signs and symptoms often appear during the first few days, weeks, or months of a child's life. VSD is an acyanotic congenital heart defect, manifesting as a left-to-right shunt. A holosystolic murmur is often appreciated, with larger VSDs causing a parasternal heave. An infant with a large VSD will fail to thrive and become diaphoretic and tachypneic, especially with feeding. D. Choice D is incorrect because a large patent ducutus arteriosus (PDA) is one of the heart defects that present with a murmur and signs of congestive heart failure in infancy. A PDA is more common in premature infants and those with neonatal respiratory distress syndrome. A smaller PDA may not cause any symptoms, but infants can present with tachypnea, poor feeding, tachycardia, shortness of breath, fatigue, diaphoresis, and poor growth. E. Choice E is correct because atrial septal defects (ASDs) do not cause CHF. An ASD malformation is a left-to-right shunt, and-depending on the size of the defect-the patient may or may not present with symptoms. ASDs often go undiagnosed for decades due to subtle physical examination findings and/or a lack of appreciable symptoms. If the defect is large enough, pediatric patients may present with easy fatigability, recurrent respiratory infections, or exertional dyspnea.

An asymptomatic, healthy 9-month-old female is found to have a palpable RUQ mass on exam. After further imaging and lab studies, the mass is diagnosed as a neuroblastoma that has involvement in the bone marrow as well. The mother is worried about the prognosis. Which of the following is true about the prognosis of neuroblastoma in this child? A. Lymph node involvement is a poor prognostic factor B. Prognosis of neuroblastoma is predictable C. Children who are older than 12 months have a better prognosis than younger children D. Favorable histology does not play a role in prognosis E. Non-amplification of the n-myc gene is a favorable prognostic factor.

> E has been selected by the expert. A. Due to the effectiveness of chemotherapy, neuroblastomas with lymph node involvement are still considered favorable, especially in the setting of other favorable factors, such as young age and differentiating histology. Though distant metastasis is a significant poor prognostic factor, regional lymph nodes do not significantly affect the outcome. B. Neuroblastoma has a broad spectrum of clinical courses. Some tumors may spontaneously regress, some may mature to a benign type, and yet other tumors can be very aggressive with metastases. Age plays a role in the prognosis, as most infants have a good prognosis even with disseminated disease, while infants over 18 months of age do not do as well. C. In infants less than one year of age, neuroblastoma tumors may spontaneously regress. Stage 4S neuroblastoma is a special category that is reserved for infants less than 12 months who have resectable primary tumors and metastases to the liver, skin, and bone marrow. Overall survival is over 85 percent for babies over 6 weeks of age with Stage 4S. D. Favorable histology is a good prognostic factor in neuroblastoma, and is based on the differentiation of the cells involved. E. Non-amplification of the n-myc gene is one of the favorable genetics in neuroblastoma.

A 15-month-old boy presents to the ED in January with a 3-day history of diarrhea. His current weight is 11 kg. He was born at 39 weeks, without any perinatal complications. There is no significant history of travel, sick contacts, or recent changes in diet. The mother notes that he has had only 2 diaper changes over the last day. Physical exam is remarkable for an irritable but consolable infant with tachycardia and normal blood pressure. He is crying without tears and his mucous membranes are dry. His abdominal exam is benign. There is no tenting, and capillary refill is 2 seconds. He is diagnosed with gastroenteritis and started on rehydration therapy. Which of the following statements is true? A. The patient is mildly dehydrated and should be managed with oral rehydration (Pedialyte). B. The patient is moderately dehydrated and should be managed with oral rehydration (Gatorade). C. The patient should be rehydrated with clear liquids and then transitioned to a lactose-free diet until his diarrhea resolves. D. The patient is moderately dehydrated and should be bolused with 220 ccs of D5 ½ normal saline for emergency phase correction, to ensure hemodynamic stability. E. ) The work-up for infectious diarrhea for this patient should include a Wright's stain for fecal WBCs, a stool Rotazyme, and a stool sample for culture and sensitivity.

> E has been selected by the expert. A. The patient is likely moderately dehydrated, given the minimal urine output over the last day, the tachycardia (in the face of normal blood pressure), and the preserved skin turgor and capillary refill. If the patient was mildly dehydrated, oral rehydration therapy would be appropriate management, and in low resource settings, oral rehydration has been used very successfully for moderately dehydrated children as well. B. Gatorade would not be recommended as a rehydrating solution for a 15-month-old child. The sugar concentration is high and this may lead to an osmotic diarrhea. C. Current recommendations from the American Academy of Pediatrics suggest reintroduction of the regular diet, as tolerated, and not transitioning first with a lactose-free diet. D. Moderately dehydrated patients should be bolused with 20 ccs/kg of IV fluid to insure hemodynamic stability and adequate perfusion of vital organs. The preferred fluids would be normal saline or lactated ringers. E. In addition to correcting this patient's hydration status, a work-up for the infectious causes of this patient's diarrhea might include a stool Wright's stain for fecal WBCs (which would suggest a bacterial cause if this is infectious diarrhea), a Rotazyme test (given the high incidence of rotavirus in the winter months), and a stool sample for culture and sensitivity. Additional studies might include stool guaiac (for occult blood) and a check for stool C. diff toxin.

A 4-year-old girl with a history of type 1 diabetes mellitus was admitted to a local hospital for treatment of DKA. A few hours after the treatment, she develops grunting, irregular respirations, and has vomited twice. On exam, her left eye is pointing downward and out on straight gaze. Her diastolic blood pressure is 90 mmHg. What is a likely diagnosis? A. Hypoglycemia B. Hypokalemia C. Hyponatremia D. Pneumonia with possible sepsis E. Cerebral edema

> E has been selected by the expert. A. This choice is incorrect because although hypoglycemia can be a complication of DKA treatment, it would be unlikely to cause a cranial nerve palsy. B. This choice is incorrect. Although hypokalemia can be a complication of DKA treatment, it would likely present as an elevated BP, muscular weakness or myalgia, as well as muscle cramps, constipation, and hyporeflexia in severe cases, rather than with the symptoms described in this vignette. C. This choice is incorrect because, while hyponatremia can occur (due to the dilutional effect caused by water shifting from the intracellular to the extracellular compartment because of hyperglycemia and increased plasma osmolarity), it would be corrected with the DKA treatment and would not present with a cranial nerve palsy. D. This choice is incorrect because pneumonia would not present with a cranial nerve palsy, although it might be important to evaluate patients with DKA for signs of intercurrent illness, including pneumonia, UTI, and perinephric abscess. E. This choice is correct. Administration of bicarbonate during DKA treatment increases the risk of cerebral edema. Although symptomatic cerebral edema is rare (less than 1%), it is associated with a high mortality rate (over 20%). The signs of cerebral edema are described in the vignette, and include irregular respirations, headache, vomiting, third nerve palsy, and high blood pressure.


Related study sets

Network+ Quiz #1 (Chapter 1,2,3)

View Set

Careers, Salaries, and Lifetime Income

View Set

electrical signaling by neurons part 1

View Set

Great Depression/New Deal Review

View Set

Writing and Evaluating Expression

View Set

Chapter 12: Health Promotion of the Toddler and Family

View Set

Chapter 18 cost accy dynamic study module

View Set

Unlv Kin 223 Chapter 1 study guide

View Set